Site Loader

Π‘ΠΎΠ΄Π΅Ρ€ΠΆΠ°Π½ΠΈΠ΅

Как Ρ€Π΅ΡˆΠ°Ρ‚ΡŒ 2 Π·Π°Π΄Π°Π½ΠΈΠ΅ Π•Π“Π­ ΠΏΠΎ Ρ„ΠΈΠ·ΠΈΠΊΠ΅, ΠΏΡ€ΠΈΠΌΠ΅Ρ€Ρ‹ Ρ€Π΅ΡˆΠ΅Π½ΠΈΡ (Ростов-Π½Π°-Π”ΠΎΠ½Ρƒ)

Из послСдних КИМов Π•Π“Π­ ΠΏΠΎ Ρ„ΠΈΠ·ΠΈΠΊΠ΅ слСдуСт, Ρ‡Ρ‚ΠΎ Π·Π°Π΄Π°Π½ΠΈΠ΅ 2 относится ΠΊ Ρ€Π°Π·Π΄Π΅Π»Ρƒ Β«Π”ΠΈΠ½Π°ΠΌΠΈΠΊΠ°Β» ΠΈ ΠΌΠΎΠΆΠ΅Ρ‚ ΡΠΎΠ΄Π΅Ρ€ΠΆΠ°Ρ‚ΡŒ расчСтныС Π·Π°Π΄Π°Ρ‡ΠΈ ΠΏΠΎ ΡΠ»Π΅Π΄ΡƒΡŽΡ‰ΠΈΠΌ Ρ‚Π΅ΠΌΠ°ΠΌ: Β«Π—Π°ΠΊΠΎΠ½Ρ‹ ΠΡŒΡŽΡ‚ΠΎΠ½Π°, Π·Π°ΠΊΠΎΠ½ всСмирного тяготСния, Π·Π°ΠΊΠΎΠ½ Π“ΡƒΠΊΠ°, сила трСния».

ΠžΡΠ½ΠΎΠ²Π½Ρ‹Π΅ Ρ„ΠΎΡ€ΠΌΡƒΠ»Ρ‹, ΠΊΠΎΡ‚ΠΎΡ€Ρ‹Π΅ Π½Π΅ΠΎΠ±Ρ…ΠΎΠ΄ΠΈΠΌΠΎ Π·Π½Π°Ρ‚ΡŒ для ΡƒΡΠΏΠ΅ΡˆΠ½ΠΎΠ³ΠΎ Ρ€Π΅ΡˆΠ΅Π½ΠΈΡ задания 2.

Π‘ΠΈΠ»Π° тяТСсти

m — масса Ρ‚Π΅Π»Π°

g=10 ΠΌ/с2 – ускорСниС свободного падСния

Π‘ΠΈΠ»Π° упругости

Ξ”x – ΡƒΠ΄Π»ΠΈΠ½Π΅Π½ΠΈΠ΅ ΠΏΡ€ΡƒΠΆΠΈΠ½Ρ‹

k – коэффициСнт ТСсткости ΠΏΡ€ΡƒΠΆΠΈΠ½Ρ‹

Π‘ΠΈΠ»Π° трСния

Β΅ — коэффициСнт трСния

N – сила Ρ€Π΅Π°ΠΊΡ†ΠΈΠΈ ΠΎΠΏΠΎΡ€Ρ‹

Π‘ΠΈΠ»Π° АрхимСда (Π²Ρ‹Ρ‚Π°Π»ΠΊΠΈΠ²Π°ΡŽΡ‰Π°Ρ сила)

V – ΠΎΠ±ΡŠΡ‘ΠΌ ΠΏΠΎΠ³Ρ€ΡƒΠΆΠ΅Π½Π½ΠΎΠΉ части Ρ‚Π΅Π»Π°

g=10 ΠΌ/с2 – ускорСниС свободного падСния

Π‘ΠΈΠ»Π° притяТСния ΠΌΠ΅ΠΆΠ΄Ρƒ Ρ‚Π΅Π»Π°ΠΌΠΈ (Π·Π°ΠΊΠΎΠ½ ВсСмирного тяготСния)

G = 6,67*10-11 Н*ΠΌ2/ΠΊΠ³2 – гравитационная постоянная

m1 ΠΈ m2 - массы Π²Π·Π°ΠΈΠΌΠΎΠ΄Π΅ΠΉΡΡ‚Π²ΡƒΡŽΡ‰ΠΈΡ… Ρ‚Π΅Π»

r – расстояниС ΠΌΠ΅ΠΆΠ΄Ρƒ Ρ‚Π΅Π»Π°ΠΌΠΈ

Π’Ρ‚ΠΎΡ€ΠΎΠΉ Π·Π°ΠΊΠΎΠ½ ΠΡŒΡŽΡ‚ΠΎΠ½Π°

m – масса Ρ‚Π΅Π»Π°

R – Ρ€Π°Π²Π½ΠΎΠ΄Π΅ΠΉΡΡ‚Π²ΡƒΡŽΡ‰Π°Ρ всСх сил, Π΄Π΅ΠΉΡΡ‚Π²ΡƒΡŽΡ‰ΠΈΡ… Π½Π° Ρ‚Π΅Π»ΠΎ

a – ускорСниС, с ΠΊΠΎΡ‚ΠΎΡ€Ρ‹ΠΌ двиТСтся Ρ‚Π΅Π»ΠΎ ΠΏΠΎΠ΄ дСйствиСм этих сил

ΠŸΡ€ΠΈ Ρ€Π΅ΡˆΠ΅Π½ΠΈΠΈ Π·Π°Π΄Π°Ρ‡ ΠΈΠ· Ρ€Π°Π·Π΄Π΅Π»Π° Β«Π”ΠΈΠ½Π°ΠΌΠΈΠΊΠ°Β» ΠΆΠ΅Π»Π°Ρ‚Π΅Π»ΡŒΠ½ΠΎ ΠΏΡ€ΠΈΠ΄Π΅Ρ€ΠΆΠΈΠ²Π°Ρ‚ΡŒΡΡ ΡΠ»Π΅Π΄ΡƒΡŽΡ‰Π΅Π³ΠΎ Π°Π»Π³ΠΎΡ€ΠΈΡ‚ΠΌΠ° Ρ€Π΅ΡˆΠ΅Π½ΠΈΡ:

1. Π‘Π΄Π΅Π»Π°Ρ‚ΡŒ рисунок, Π½Π° ΠΊΠΎΡ‚ΠΎΡ€ΠΎΠΌ ΡƒΠΊΠ°Π·Π°Ρ‚ΡŒ Π²Π΅ΠΊΡ‚ΠΎΡ€Π° всСх сил, Π΄Π΅ΠΉΡΡ‚Π²ΡƒΡŽΡ‰ΠΈΡ… Π½Π° Ρ‚Π΅Π»ΠΎ.

2. Если Ρ‚Π΅Π»ΠΎ двигаСтся с ускорСниСм, ΡƒΠΊΠ°Π·Π°Ρ‚ΡŒ Π½Π°ΠΏΡ€Π°Π²Π»Π΅Π½ΠΈΠ΅ этого ускорСния. Если Ρ‚Π΅Π»ΠΎ покоится ΠΈΠ»ΠΈ двигаСтся Ρ€Π°Π²Π½ΠΎΠΌΠ΅Ρ€Π½ΠΎ, Π΅Π³ΠΎ ускорСниС a=0.

3. Π‘ΠΎΡΡ‚Π°Π²ΠΈΡ‚ΡŒ ΡƒΡ€Π°Π²Π½Π΅Π½ΠΈΠ΅ двиТСния (Π²Ρ‚ΠΎΡ€ΠΎΠΉ Π·Π°ΠΊΠΎΠ½ ΠΡŒΡŽΡ‚ΠΎΠ½Π°) для рассматриваСмого Ρ‚Π΅Π»Π° Π² Π΅Π³ΠΎ Π²Π΅ΠΊΡ‚ΠΎΡ€Π½ΠΎΠΌ Π²ΠΈΠ΄Π΅.

3. Π’Ρ‹Π±Ρ€Π°Ρ‚ΡŒ систСму ΠΊΠΎΠΎΡ€Π΄ΠΈΠ½Π°Ρ‚ ΠΈ ΡΠΏΡ€ΠΎΠ΅Ρ†ΠΈΡ€ΠΎΠ²Π°Ρ‚ΡŒ ΠΏΠΎΠ»ΡƒΡ‡Π΅Π½Π½ΠΎΠ΅ ΡƒΡ€Π°Π²Π½Π΅Π½ΠΈΠ΅ Π½Π° Π²Ρ‹Π±Ρ€Π°Π½Π½Ρ‹Π΅ оси ΠΊΠΎΠΎΡ€Π΄ΠΈΠ½Π°Ρ‚.

4. Π Π°ΡΡˆΠΈΡ„Ρ€ΠΎΠ²Π°Ρ‚ΡŒ нСизвСстныС Π²Π΅Π»ΠΈΡ‡ΠΈΠ½Ρ‹, вошСдшиС Π² ΡƒΡ€Π°Π²Π½Π΅Π½ΠΈΠ΅ двиТСния.

5. Π Π΅ΡˆΠΈΡ‚ΡŒ ΠΏΠΎΠ»ΡƒΡ‡Π΅Π½Π½ΡƒΡŽ систСму ΡƒΡ€Π°Π²Π½Π΅Π½ΠΈΠΉ.

Π—Π°Π΄Π°Π½ΠΈΠ΅ 2 – это расчётныС Π·Π°Π΄Π°Ρ‡ΠΈ Π±Π°Π·ΠΎΠ²ΠΎΠ³ΠΎ уровня слоТности, ΠΈ для Ρ€Π΅ΡˆΠ΅Π½ΠΈΡ Π½Π΅ΠΊΠΎΡ‚ΠΎΡ€Ρ‹Ρ… ΠΈΠ· Π½ΠΈΡ… этот Π°Π»Π³ΠΎΡ€ΠΈΡ‚ΠΌ Π±ΡƒΠ΄Π΅Ρ‚ чСрСсчур ΠΏΠΎΠ΄Ρ€ΠΎΠ±Π½Ρ‹ΠΌ ΠΈ ΠΏΠ΅Ρ€Π΅Π³Ρ€ΡƒΠΆΠ΅Π½Π½Ρ‹ΠΌ, Ρ‚Π°ΠΊ ΠΊΠ°ΠΊ ΠΈΡ… ΠΌΠΎΠΆΠ½ΠΎ Ρ€Π΅ΡˆΠΈΡ‚ΡŒ ΠΈ Π±Π΅Π· Π²ΡΠΏΠΎΠΌΠΎΠ³Π°Ρ‚Π΅Π»ΡŒΠ½ΠΎΠ³ΠΎ рисунка ΠΈΠ»ΠΈ Π΄Π°ΠΆΠ΅ Π±Π΅Π· записи Π²Ρ‚ΠΎΡ€ΠΎΠ³ΠΎ Π·Π°ΠΊΠΎΠ½Π° ΠΡŒΡŽΡ‚ΠΎΠ½Π°. Π­Ρ‚ΠΎ касаСтся, Π½Π°ΠΏΡ€ΠΈΠΌΠ΅Ρ€, Π·Π°Π΄Π°Π½ΠΈΠΉ, Π² ΠΊΠΎΡ‚ΠΎΡ€Ρ‹Ρ… Π½Π° Ρ‚Π΅Π»ΠΎ дСйствуСт Ρ‚ΠΎΠ»ΡŒΠΊΠΎ ΠΎΠ΄Π½Π° сила. Но ΠΏΡ€ΠΈΠ²Ρ‹Ρ‡ΠΊΠ° Ρ€Π΅ΡˆΠ°Ρ‚ΡŒ задания ΠΏΠΎ ΠΏΡ€ΠΈΠ²Π΅Π΄Π΅Π½Π½ΠΎΠΌΡƒ Π²Ρ‹ΡˆΠ΅ Π°Π»Π³ΠΎΡ€ΠΈΡ‚ΠΌΡƒ ΠΏΠΎΠΌΠΎΠΆΠ΅Ρ‚ ΡƒΡ‡Π΅Π½ΠΈΠΊΠ°ΠΌ ΡƒΡΠΏΠ΅ΡˆΠ½ΠΎ ΡΠΏΡ€Π°Π²ΠΈΡ‚ΡŒΡΡ с расчСтными Π·Π°Π΄Π°Ρ‡Π°ΠΌΠΈ ΠΏΠΎ Ρ€Π°Π·Π΄Π΅Π»Ρƒ Β«Π”ΠΈΠ½Π°ΠΌΠΈΠΊΠ°Β» ΠΏΠΎΠ²Ρ‹ΡˆΠ΅Π½Π½ΠΎΠ³ΠΎ ΠΈ высокого ΡƒΡ€ΠΎΠ²Π½Π΅ΠΉ слоТности – Ρ‚Π°ΠΊΠΈΠ΅ задания ΠΌΠΎΠ³ΡƒΡ‚ ΡΡ‚ΠΎΡΡ‚ΡŒ Π² Π•Π“Π­ ΠΏΠΎΠ΄ Π½ΠΎΠΌΠ΅Ρ€Π°ΠΌΠΈ 25 ΠΈ 29.

ΠžΡ‚Π²Π΅Ρ‚ΠΎΠΌ Π½Π° Π·Π°Π΄Π°Π½ΠΈΠ΅ 2 являСтся число, ΠΈΠΌΠ΅Π½Π½ΠΎ Π΅Π³ΠΎ Π½ΡƒΠΆΠ½ΠΎ Π²ΠΏΠΈΡΠ°Ρ‚ΡŒ Π² Π±Π»Π°Π½ΠΊ ΠΎΡ‚Π²Π΅Ρ‚ΠΎΠ² 1, Π½Π΅ указывая Π΅Π΄ΠΈΠ½ΠΈΡ†Ρ‹ измСрСния.

ΠŸΡ€ΠΈΠΌΠ΅Ρ€Ρ‹ Ρ€Π΅ΡˆΠ΅Π½ΠΈΡ

1. (Π•Π“Π­-2019)

ΠŸΡ€ΡƒΠΆΠΈΠ½Π° ΠΆΡ‘ΡΡ‚ΠΊΠΎΡΡ‚ΡŒΡŽ 2*104 Н/ΠΌ ΠΎΠ΄Π½ΠΈΠΌ ΠΊΠΎΠ½Ρ†ΠΎΠΌ Π·Π°ΠΊΡ€Π΅ΠΏΠ»Π΅Π½Π° Π² ΡˆΡ‚Π°Ρ‚ΠΈΠ²Π΅. На ΠΊΠ°ΠΊΡƒΡŽ Π²Π΅Π»ΠΈΡ‡ΠΈΠ½Ρƒ ΠΎΠ½Π° растянСтся ΠΏΠΎΠ΄ дСйствиСм силы 400 Н?

ΠžΡ‚Π²Π΅Ρ‚: ___________________________ см.

РСшСниС:

Π‘Π΄Π΅Π»Π°Π΅ΠΌ Ρ‡Π΅Ρ€Ρ‚Ρ‘ΠΆ

ΠŸΡ€ΡƒΠΆΠΈΠ½Π° ΠΏΠΎΠ΄ дСйствиСм силы F ΠΏΡ€ΠΈΠ²Π΅Π»ΠΈ Π² растянутоС состояниС. ΠšΡ€ΠΎΠΌΠ΅ Ρ€Π°ΡΡ‚ΡΠ³ΠΈΠ²Π°ΡŽΡ‰Π΅ΠΉ силы F ΠΈ силы упругости , стрСмящСйся Π²Π΅Ρ€Π½ΡƒΡ‚ΡŒ ΠΏΡ€ΡƒΠΆΠΈΠ½Ρƒ Π² нСрастянутоС состояниС, большС Π½ΠΈΠΊΠ°ΠΊΠΈΠ΅ силы Π½Π° Π½Π΅Π΅ Π½Π΅ Π΄Π΅ΠΉΡΡ‚Π²ΡƒΡŽΡ‚.

Π—Π°ΠΏΠΈΡˆΠ΅ΠΌ ΠΏΡ€ΠΎΠ΅ΠΊΡ†ΠΈΠΈ сил Π½Π° Π²Π΅Ρ€Ρ‚ΠΈΠΊΠ°Π»ΡŒΠ½ΡƒΡŽ ось Oy

F=Fупр

По Π·Π°ΠΊΠΎΠ½Ρƒ Π“ΡƒΠΊΠ°, сила упругости FΡƒΠΏΡ€ = k *Ξ”x, ΡΠ»Π΅Π΄ΠΎΠ²Π°Ρ‚Π΅Π»ΡŒΠ½ΠΎ,

kкоэффициСнт Тёсткости ΠΏΡ€ΡƒΠΆΠΈΠ½Ρ‹, Ξ”x – Π΅Ρ‘ ΡƒΠ΄Π»ΠΈΠ½Π΅Π½ΠΈΠ΅.

Π’Ρ‹Ρ€Π°Π·ΠΈΠΌ Π²Π΅Π»ΠΈΡ‡ΠΈΠ½Ρƒ растяТСния ΠΏΡ€ΡƒΠΆΠΈΠ½Ρ‹

ΠžΡ‚Π²Π΅Ρ‚: 2

  1. (Π•Π“Π­ – 2020. Π’Π°Ρ€ΠΈΠ°Π½Ρ‚ 1 досрочного Π•Π“Π­)

Π’Π΅Π»ΠΎ двиТСтся ΠΏΠΎ Π³ΠΎΡ€ΠΈΠ·ΠΎΠ½Ρ‚Π°Π»ΡŒΠ½ΠΎΠΉ плоскости. ΠΠΎΡ€ΠΌΠ°Π»ΡŒΠ½Π°Ρ ΡΠΎΡΡ‚Π°Π²Π»ΡΡŽΡ‰Π°Ρ силы воздСйствия Ρ‚Π΅Π»Π° Π½Π° ΠΏΠ»ΠΎΡΠΊΠΎΡΡ‚ΡŒ Ρ€Π°Π²Π½Π° 40 Н, сила трСния Ρ€Π°Π²Π½Π° 10 Н. ΠžΠΏΡ€Π΅Π΄Π΅Π»ΠΈΡ‚Π΅ коэффициСнт трСния скольТСния.

ΠžΡ‚Π²Π΅Ρ‚: _______ .

РСшСниС:

Π‘ΠΈΠ»Ρƒ трСния ΠΌΠΎΠΆΠ½ΠΎ Π½Π°ΠΉΡ‚ΠΈ ΠΏΠΎ Ρ„ΠΎΡ€ΠΌΡƒΠ»Π΅

FΡ‚Ρ€= Β΅N,

Π³Π΄Π΅ N – сила Ρ€Π΅Π°ΠΊΡ†ΠΈΠΈ ΠΎΠΏΠΎΡ€Ρ‹, ΠΈΠ»ΠΈ ΠΏΠΎ-Π΄Ρ€ΡƒΠ³ΠΎΠΌΡƒ Π½ΠΎΡ€ΠΌΠ°Π»ΡŒΠ½Π°Ρ ΡΠΎΡΡ‚Π°Π²Π»ΡΡŽΡ‰Π°Ρ силы воздСйствия Ρ‚Π΅Π»Π° Π½Π° ΠΏΠ»ΠΎΡΠΊΠΎΡΡ‚ΡŒ.

ΠžΡ‚Π²Π΅Ρ‚: 0,25.

  1. (Π•Π“Π­ – 2020. ДСмонстрационный Π²Π°Ρ€ΠΈΠ°Π½Ρ‚)

Π”Π²Π° ΠΎΠ΄ΠΈΠ½Π°ΠΊΠΎΠ²Ρ‹Ρ… ΠΌΠ°Π»Π΅Π½ΡŒΠΊΠΈΡ… ΡˆΠ°Ρ€ΠΈΠΊΠ° массой m ΠΊΠ°ΠΆΠ΄Ρ‹ΠΉ, расстояниС ΠΌΠ΅ΠΆΠ΄Ρƒ Ρ†Π΅Π½Ρ‚Ρ€Π°ΠΌΠΈ ΠΊΠΎΡ‚ΠΎΡ€Ρ‹Ρ… Ρ€Π°Π²Π½ΠΎ r, ΠΏΡ€ΠΈΡ‚ΡΠ³ΠΈΠ²Π°ΡŽΡ‚ΡΡ Π΄Ρ€ΡƒΠ³ ΠΊ Π΄Ρ€ΡƒΠ³Ρƒ с силами, Ρ€Π°Π²Π½Ρ‹ΠΌΠΈ ΠΏΠΎ ΠΌΠΎΠ΄ΡƒΠ»ΡŽ 0,2 пН. Каков ΠΌΠΎΠ΄ΡƒΠ»ΡŒ сил Π³Ρ€Π°Π²ΠΈΡ‚Π°Ρ†ΠΈΠΎΠ½Π½ΠΎΠ³ΠΎ притяТСния Π΄Π²ΡƒΡ… Π΄Ρ€ΡƒΠ³ΠΈΡ… ΡˆΠ°Ρ€ΠΈΠΊΠΎΠ², Ссли масса ΠΊΠ°ΠΆΠ΄ΠΎΠ³ΠΎ ΠΈΠ· Π½ΠΈΡ… Ρ€Π°Π²Π½Π° 2

m, Π° расстояниС ΠΌΠ΅ΠΆΠ΄Ρƒ ΠΈΡ… Ρ†Π΅Π½Ρ‚Ρ€Π°ΠΌΠΈ Ρ€Π°Π²Π½ΠΎ 2r?

ΠžΡ‚Π²Π΅Ρ‚: _______ пН.

РСшСниС:

По Π·Π°ΠΊΠΎΠ½Ρƒ ВсСмирного тяготСния ΡˆΠ°Ρ€Ρ‹ массами m1ΠΈ m2, находящиСся Π΄Ρ€ΡƒΠ³ ΠΎΡ‚ Π΄Ρ€ΡƒΠ³Π° Π½Π° расстоянии r, ΠΏΡ€ΠΈΡ‚ΡΠ³ΠΈΠ²Π°ΡŽΡ‚ΡΡ Π΄Ρ€ΡƒΠ³ ΠΊ Π΄Ρ€ΡƒΠ³Ρƒ с силой

.

Π’ ΠΏΠ΅Ρ€Π²ΠΎΠΌ случаС

Π’ΠΎ Π²Ρ‚ΠΎΡ€ΠΎΠΌ случаС

ΠžΡ‚Π²Π΅Ρ‚: 0,2

  1. (Π•Π“Π­ – 2019. ДСмонстрационный Π²Π°Ρ€ΠΈΠ°Π½Ρ‚)

По Π³ΠΎΡ€ΠΈΠ·ΠΎΠ½Ρ‚Π°Π»ΡŒΠ½ΠΎΠΌΡƒ ΠΏΠΎΠ»Ρƒ ΠΏΠΎ прямой Ρ€Π°Π²Π½ΠΎΠΌΠ΅Ρ€Π½ΠΎ тянут ящик, ΠΏΡ€ΠΈΠ»ΠΎΠΆΠΈΠ² ΠΊ Π½Π΅ΠΌΡƒ Π³ΠΎΡ€ΠΈΠ·ΠΎΠ½Ρ‚Π°Π»ΡŒΠ½ΡƒΡŽ силу 35 Н. ΠšΠΎΡΡ„Ρ„ΠΈΡ†ΠΈΠ΅Π½Ρ‚ трСния скольТСния ΠΌΠ΅ΠΆΠ΄Ρƒ ΠΏΠΎΠ»ΠΎΠΌ ΠΈ ящиком Ρ€Π°Π²Π΅Π½ 0,25. Π§Π΅ΠΌΡƒ Ρ€Π°Π²Π½Π° масса ящика?

ΠžΡ‚Π²Π΅Ρ‚ _______ ΠΊΠ³.

РСшСниС:

Π‘Π΄Π΅Π»Π°Π΅ΠΌ Ρ‡Π΅Ρ€Ρ‚Ρ‘ΠΆ, Π½Π° ΠΊΠΎΡ‚ΠΎΡ€ΠΎΠΌ ΠΎΠ±ΠΎΠ·Π½Π°Ρ‡ΠΈΠΌ всС силы, Π΄Π΅ΠΉΡΡ‚Π²ΡƒΡŽΡ‰ΠΈΠ΅ Π½Π° Ρ‚Π΅Π»ΠΎ.

По Π²Ρ‚ΠΎΡ€ΠΎΠΌΡƒ Π·Π°ΠΊΠΎΠ½Ρƒ ΠΡŒΡŽΡ‚ΠΎΠ½Π°, Ρ€Π°Π²Π½ΠΎΠ΄Π΅ΠΉΡΡ‚Π²ΡƒΡŽΡ‰Π°Ρ всСх сил, Π΄Π΅ΠΉΡΡ‚Π²ΡƒΡŽΡ‰ΠΈΡ… Π½Π° Ρ‚Π΅Π»ΠΎ, Π±ΡƒΠ΄Π΅Ρ‚ Ρ€Π°Π²Π½Π° Π½ΡƒΠ»ΡŽ, Ρ‚Π°ΠΊ ΠΊΠ°ΠΊ ΠΏΠΎ ΡƒΡΠ»ΠΎΠ²ΠΈΡŽ Π·Π°Π΄Π°Ρ‡ΠΈ Ρ‚Π΅Π»ΠΎ двиТСтся Ρ€Π°Π²Π½ΠΎΠΌΠ΅Ρ€Π½ΠΎ, Ρ‚ΠΎ Π΅ΡΡ‚ΡŒ ускорСниС Ρ‚Π΅Π»Π° a=0.

Π—Π°ΠΏΠΈΡˆΠ΅ΠΌ это Π² проСкциях Π½Π° оси Ox ΠΈ Oy

Ox: FΡ‚Ρ€ – F = 0,

Oy: N — m g=0.

ΠžΡ‚ΠΊΡƒΠ΄Π° N = mg, ΡΠ»Π΅Π΄ΠΎΠ²Π°Ρ‚Π΅Π»ΡŒΠ½ΠΎ,

FΡ‚Ρ€ = Β΅ N = Β΅ mg.

Масса Ρ‚Π΅Π»Π°

ΠžΡ‚Π²Π΅Ρ‚: 14

  1. (Π•Π“Π­ – 2018)

К ΠΏΡ€ΡƒΠΆΠΈΠ½Π΅ подвСсили Π³Ρ€ΡƒΠ· массой 150 Π³, вслСдствиС Ρ‡Π΅Π³ΠΎ ΠΏΡ€ΡƒΠΆΠΈΠ½Π° ΡƒΠ΄Π»ΠΈΠ½ΠΈΠ»Π°ΡΡŒ Π½Π° 1 см. Π§Π΅ΠΌΡƒ Π±ΡƒΠ΄Π΅Ρ‚ Ρ€Π°Π²Π½ΠΎ ΡƒΠ΄Π»ΠΈΠ½Π΅Π½ΠΈΠ΅ этой ΠΏΡ€ΡƒΠΆΠΈΠ½Ρ‹, Ссли ΠΊ Π½Π΅ΠΉ ΠΏΠΎΠ΄Π²Π΅ΡΠΈΡ‚ΡŒ Π³Ρ€ΡƒΠ· 450 Π³?

ΠžΡ‚Π²Π΅Ρ‚: __________ см.

РСшСниС:

ΠŸΠ΅Ρ€Π΅Π²Π΅Π΄Ρ‘ΠΌ Π΅Π΄ΠΈΠ½ΠΈΡ†Ρ‹ измСрСния физичСских Π²Π΅Π»ΠΈΡ‡ΠΈΠ½ Π² систСму БИ

m1 = 150 Π³ = 0,15 ΠΊΠ³, m2 = 450 Π³ = 0,45 ΠΊΠ³, Ξ”x=1 см = 0,01 ΠΌ.

Π‘Π΄Π΅Π»Π°Π΅ΠΌ Ρ‡Π΅Ρ€Ρ‚Ρ‘ΠΆ, Π½Π° ΠΊΠΎΡ‚ΠΎΡ€ΠΎΠΌ ΠΎΠ±ΠΎΠ·Π½Π°Ρ‡ΠΈΠΌ всС силы, Π΄Π΅ΠΉΡΡ‚Π²ΡƒΡŽΡ‰ΠΈΠ΅ Π½Π° Ρ‚Π΅Π»ΠΎ.

На Ρ‚Π΅Π»ΠΎ дСйствуСт сила тяТСсти (FΡ‚ = mg), направлСнная Π²Π΅Ρ€Ρ‚ΠΈΠΊΠ°Π»ΡŒΠ½ΠΎ Π²Π½ΠΈΠ·, ΠΈ сила упругости со стороны ΠΏΡ€ΡƒΠΆΠΈΠ½Ρ‹ (FΡƒΠΏΡ€ = k Ξ”x), направлСнная Π²Π΅Ρ€Ρ‚ΠΈΠΊΠ°Π»ΡŒΠ½ΠΎ Π²Π²Π΅Ρ€Ρ….

Π’ ΠΏΡ€ΠΎΠ΅ΠΊΡ†ΠΈΠΈ Π½Π° Π²Π΅Ρ€Ρ‚ΠΈΠΊΠ°Π»ΡŒΠ½ΡƒΡŽ ось Oy.

Fт =Fупр

mg = kΞ”x (1)

kкоэффициСнт Тёсткости ΠΏΡ€ΡƒΠΆΠΈΠ½Ρ‹, Ξ”x – Π΅Ρ‘ ΡƒΠ΄Π»ΠΈΠ½Π΅Π½ΠΈΠ΅.

Найдём, Ρ‡Π΅ΠΌΡƒ Ρ€Π°Π²Π΅Π½ коэффициСнт Тёсткости ΠΏΡ€ΡƒΠΆΠΈΠ½Ρ‹

Π’Ρ‹Ρ€Π°Π·ΠΈΠΌ ΠΈΠ· выраТСния (1) ΡƒΠ΄Π»ΠΈΠ½Π΅Π½ΠΈΠ΅ ΠΏΡ€ΡƒΠΆΠΈΠ½Ρ‹ Π²ΠΎ Π²Ρ‚ΠΎΡ€ΠΎΠΌ случаС

ΠžΡ‚Π²Π΅Ρ‚: 3

Π Π•ΠšΠžΠœΠ•ΠΠ”Π£Π•ΠœΠ«Π• Π’ΠžΠ’ΠΠ Π«

Как Π½Π°ΠΉΡ‚ΠΈ силу трСния Ссли извСстна. Как Π½Π°ΠΉΡ‚ΠΈ силу трСния. Π‘ΠΈΠ»Π° трСния ΠΈ коэффициСнт трСния

(ЗанятиС каникулярной ΡˆΠΊΠΎΠ»Ρ‹ для учащихся 8–9 ΠΊΠ».)

  • Активизация ΠΌΡ‹ΡΠ»ΠΈΡ‚Π΅Π»ΡŒΠ½ΠΎΠΉ Π΄Π΅ΡΡ‚Π΅Π»ΡŒΠ½ΠΎΡΡ‚ΠΈ учащихся.
  • Π€ΠΎΡ€ΠΌΠΈΡ€ΠΎΠ²Π°Π½ΠΈΠ΅ ΠΎΠ±ΠΎΠ±Ρ‰Π΅Π½Π½ΠΎΠ³ΠΎ умСния ΠΏΡ€ΠΎΠ²ΠΎΠ΄ΠΈΡ‚ΡŒ физичСскиС измСрСния.
  • Π€ΠΎΡ€ΠΌΠΈΡ€ΠΎΠ²Π°Π½ΠΈΠ΅ ΠΎΠ±ΠΎΠ±Ρ‰Π΅Π½Π½ΠΎΠ³ΠΎ умСния ΠΏΡ€ΠΎΠ²ΠΎΠ΄ΠΈΡ‚ΡŒ ΡΠΊΡΠΏΠ΅Ρ€ΠΈΠΌΠ΅Π½Ρ‚Π°Π»ΡŒΠ½ΡƒΡŽ ΠΏΡ€ΠΎΠ²Π΅Ρ€ΠΊΡƒ физичСских закономСрностСй.
  • Π€ΠΎΡ€ΠΌΠΈΡ€ΠΎΠ²Π°Π½ΠΈΠ΅ умСния ΡΠΈΡΡ‚Π΅ΠΌΠ°Ρ‚ΠΈΠ·ΠΈΡ€ΠΎΠ²Π°Ρ‚ΡŒ ΠΏΠΎΠ»ΡƒΡ‡Π΅Π½Π½Ρ‹Π΅ Ρ€Π΅Π·ΡƒΠ»ΡŒΡ‚Π°Ρ‚Ρ‹ Π² Π²ΠΈΠ΄Π΅ Ρ‚Π°Π±Π»ΠΈΡ†Ρ‹, ΡƒΠΌΠ΅Π½ΠΈΠ΅ Π΄Π΅Π»Π°Ρ‚ΡŒ Π²Ρ‹Π²ΠΎΠ΄ Π½Π° основС экспСримСнта.

ΠžΡ€Π³Π°Π½ΠΈΠ·Π°Ρ†ΠΈΡ провСдСния ΠΏΡ€Π°ΠΊΡ‚ΠΈΠΊΡƒΠΌΠ°: ВсС учащиСся ΠΏΡ€ΠΈΠ½ΠΈΠΌΠ°ΡŽΡ‰ΠΈΠ΅ участиС Π² Ρ€Π°Π±ΠΎΡ‚Π΅ ΠΏΡ€Π°ΠΊΡ‚ΠΈΠΊΡƒΠΌΠ° дСлятся Π½Π° Π³Ρ€ΡƒΠΏΠΏΡ‹. КаТдая Π³Ρ€ΡƒΠΏΠΏΠ° учащихся ΠΏΠΎΠ»ΡƒΡ‡Π°Π΅Ρ‚ Π·Π°Π΄Π°Π½ΠΈΠ΅ с ΠΊΡ€Π°Ρ‚ΠΊΠΈΠΌ описаниСм Ρ€Π°Π±ΠΎΡ‚Ρ‹.

По ΠΎΠΊΠΎΠ½Ρ‡Π°Π½ΠΈΠΈ выполнСния Ρ€Π°Π±ΠΎΡ‚Ρ‹ учащимся Π½Π΅ΠΎΠ±Ρ…ΠΎΠ΄ΠΈΠΌΠΎ ΡΠΎΡΡ‚Π°Π²ΠΈΡ‚ΡŒ ΠΎΡ‚Ρ‡Π΅Ρ‚. ΠžΡ‚Ρ‡Π΅Ρ‚ состоит ΠΈΠ· Ρ‚Π°Π±Π»ΠΈΡ†Ρ‹, вычислСния искомой Π²Π΅Π»ΠΈΡ‡ΠΈΠ½Ρ‹ ΠΈ Π΅Π΅ ΠΏΠΎΠ³Ρ€Π΅ΡˆΠ½ΠΎΡΡ‚ΠΈ, Π²Ρ‹Π²ΠΎΠ΄Π° ΠΏΠΎ Ρ€Π°Π±ΠΎΡ‚Π΅.

Π₯ΠΎΠ΄ Ρ€Π°Π±ΠΎΡ‚Ρ‹

I. Π’ΡΡ‚ΡƒΠΏΠΈΡ‚Π΅Π»ΡŒΠ½ΠΎΠ΅ слово учитСля:

Если ΠΏΠΎΠ»ΠΎΠΆΠΈΡ‚ΡŒ Π½Π° Π³ΠΎΡ€ΠΈΠ·ΠΎΠ½Ρ‚Π°Π»ΡŒΠ½ΡƒΡŽ ΠΏΠΎΠ²Π΅Ρ€Ρ…Π½ΠΎΡΡ‚ΡŒ брусок ΠΈ ΠΏΠΎΠ΄Π΅ΠΉΡΡ‚Π²ΠΎΠ²Π°Ρ‚ΡŒ Π½Π° Π½Π΅Π³ΠΎ с достаточной силой Π² Π³ΠΎΡ€ΠΈΠ·ΠΎΠ½Ρ‚Π°Π»ΡŒΠ½ΠΎΠΌ Π½Π°ΠΏΡ€Π°Π²Π»Π΅Π½ΠΈΠΈ, Ρ‚ΠΎ брусок станСт Π΄Π²ΠΈΠ³Π°Ρ‚ΡŒΡΡ. НСтрудно ΡƒΠ±Π΅Π΄ΠΈΡ‚ΡŒΡΡ, Ρ‡Ρ‚ΠΎ Π² этом случаС Π½Π° брусок Π΄Π΅ΠΉΡΡ‚Π²ΡƒΡŽΡ‚ Ρ‡Π΅Ρ‚Ρ‹Ρ€Π΅ силы: Π² Π²Π΅Ρ€Ρ‚ΠΈΠΊΠ°Π»ΡŒΠ½ΠΎΠΌ Π½Π°ΠΏΡ€Π°Π²Π»Π΅Π½ΠΈΠΈ – сила тяТСсти P ΠΈ сила Ρ€Π΅Π°ΠΊΡ†ΠΈΠΈ ΠΎΠΏΠΎΡ€Ρ‹ Q, Ρ€Π°Π²Π½Ρ‹Π΅ ΠΏΠΎ ΠΌΠΎΠ΄ΡƒΠ»ΡŽ ΠΏΡ€ΠΎΡ‚ΠΈΠ²ΠΎΠΏΠΎΠ»ΠΎΠΆΠ½Ρ‹Π΅ ΠΏΠΎ Π½Π°ΠΏΡ€Π°Π²Π»Π΅Π½ΠΈΡŽ; Π² Π³ΠΎΡ€ΠΈΠ·ΠΎΠ½Ρ‚Π°Π»ΡŒΠ½ΠΎΠΌ Π½Π°ΠΏΡ€Π°Π²Π»Π΅Π½ΠΈΠΈ – сила тяги F ΠΈ противополоТная ΠΏΠΎ Π½Π°ΠΏΡ€Π°Π²Π»Π΅Π½ΠΈΡŽ сила трСния F mp .

Π§Ρ‚ΠΎΠ±Ρ‹ брусок двигался Ρ€Π°Π²Π½ΠΎΠΌΠ΅Ρ€Π½ΠΎ ΠΈ прямолинСйно, Π½ΡƒΠΆΠ½ΠΎ, Ρ‡Ρ‚ΠΎΠ±Ρ‹ ΠΌΠΎΠ΄ΡƒΠ»ΡŒ силы тяги Π±Ρ‹Π» Ρ€Π°Π²Π΅Π½ ΠΌΠΎΠ΄ΡƒΠ»ΡŽ силы трСния.

На этом основан ΠΌΠ΅Ρ‚ΠΎΠ΄ измСрСния силы трСния. Π‘Π»Π΅Π΄ΡƒΠ΅Ρ‚ ΠΏΡ€ΠΈΠ»ΠΎΠΆΠΈΡ‚ΡŒ ΠΊ бруску силу тяги, которая Π±ΡƒΠ΄Π΅Ρ‚ ΠΏΠΎΠ΄Π΄Π΅Ρ€ΠΆΠΈΠ²Π°Ρ‚ΡŒ Ρ€Π°Π²Π½ΠΎΠΌΠ΅Ρ€Π½ΠΎΠ΅ прямолинСйноС Π΄Π²ΠΈΠΆΠ΅Π½ΠΈΠ΅ этого Ρ‚Π΅Π»Π°. По этой силС тяги ΠΎΠΏΡ€Π΅Π΄Π΅Π»ΡΡŽΡ‚ ΠΌΠΎΠ΄ΡƒΠ»ΡŒ силы трСния.

II. ΠŸΡ€Π°ΠΊΡ‚ΠΈΠΊΡƒΠΌ.

Π—Π°Π΄Π°Π½ΠΈΠ΅ Π³Ρ€ΡƒΠΏΠΏΠ΅ I.

ΠžΠΏΡ€Π΅Π΄Π΅Π»ΠΈΡ‚Π΅ коэффициСнт трСния скольТСния ΠΏΡ€ΠΈ Π΄Π²ΠΈΠΆΠ΅Π½ΠΈΠΈ бруска ΠΏΠΎ Π³ΠΎΡ€ΠΈΠ·ΠΎΠ½Ρ‚Π°Π»ΡŒΠ½ΠΎΠΉ повСрхности стола.

ΠžΠ±ΠΎΡ€ΡƒΠ΄ΠΎΠ²Π°Π½ΠΈΠ΅: Ρ‚Ρ€ΠΈΠ±ΠΎΠΌΠ΅Ρ‚Ρ€, дСрСвянная Π»ΠΈΠ½Π΅ΠΉΠΊΠ°, дСрСвянный бруска с трСмя отвСрстиями; Π΄ΠΈΠ½Π°ΠΌΠΎΠΌΠ΅Ρ‚Ρ€; Π½Π°Π±ΠΎΡ€ Π³Ρ€ΡƒΠ·ΠΎΠ² ΠΏΠΎ ΠΌΠ΅Ρ…Π°Π½ΠΈΠΊΠ΅.

ΠŸΠΎΡ€ΡΠ΄ΠΎΠΊ выполнСния Ρ€Π°Π±ΠΎΡ‚Ρ‹.

  1. ВычислитС Ρ†Π΅Π½Ρƒ дСлСния ΡˆΠΊΠ°Π»Ρ‹ Π΄ΠΈΠ½Π°ΠΌΠΎΠΌΠ΅Ρ‚Ρ€Π°.
  2. Π˜Π·ΠΌΠ΅Ρ€ΡŒΡ‚Π΅ вСс бруска ΠΏΡ€ΠΈ ΠΏΠΎΠΌΠΎΡ‰ΠΈ Π΄ΠΈΠ½Π°ΠΌΠΎΠΌΠ΅Ρ‚Ρ€Π°. Π Π΅Π·ΡƒΠ»ΡŒΡ‚Π°Ρ‚ измСрСния вСса Π·Π°ΠΏΠΈΡˆΠΈΡ‚Π΅ Π² Ρ‚Π°Π±Π»ΠΈΡ†Ρƒ.
  3. Π˜Π·ΠΌΠ΅Ρ€ΡŒΡ‚Π΅ силу трСния скольТСния бруска с Π³Ρ€ΡƒΠ·Π°ΠΌΠΈ ΠΏΠΎ столу. Для этого ΠΏΠ΅Ρ€Π΅ΠΌΠ΅Ρ‰Π°ΠΉΡ‚Π΅ брусок с Π³Ρ€ΡƒΠ·Π°ΠΌΠΈ Ρ€Π°Π²Π½ΠΎΠΌΠ΅Ρ€Π½ΠΎ ΠΏΠΎ столу ΠΏΡ€ΠΈ ΠΏΠΎΠΌΠΎΡ‰ΠΈ Π΄ΠΈΠ½Π°ΠΌΠΎΠΌΠ΅Ρ‚Ρ€Π°.
  4. Π Π΅Π·ΡƒΠ»ΡŒΡ‚Π°Ρ‚ измСрСния Π·Π°ΠΏΠΈΡˆΠΈΡ‚Π΅ Π² Ρ‚Π°Π±Π»ΠΈΡ†Ρƒ.
  5. НагруТая брусок ΠΎΠ΄Π½ΠΈΠΌ, двумя ΠΈ трСмя Π³Ρ€ΡƒΠ·Π°ΠΌΠΈ, ΠΈΠ·ΠΌΠ΅Ρ€ΡŒΡ‚Π΅ Π² ΠΊΠ°ΠΆΠ΄ΠΎΠΌ случаС силу трСния. Π”Π°Π½Π½Ρ‹Π΅ занСситС Π² Ρ‚Π°Π±Π»ΠΈΡ†Ρƒ.
  6. ВычислитС коэффициСнт трСния скольТСния
  7. ΠžΠΏΡ€Π΅Π΄Π΅Π»ΠΈΡ‚Π΅ ΠΈΠ½ΡΡ‚Ρ€ΡƒΠΌΠ΅Π½Ρ‚Π°Π»ΡŒΠ½ΡƒΡŽ ΠΏΠΎΠ³Ρ€Π΅ΡˆΠ½ΠΎΡΡ‚ΡŒ коэффициСнта трСния.
  8. Π‘Π΄Π΅Π»Π°ΠΉΡ‚Π΅ Π²Ρ‹Π²ΠΎΠ΄.

Π›Π΅Π³ΠΊΠΎ ΡƒΠ±Π΅Π΄ΠΈΡ‚ΡŒΡΡ, Ρ‡Ρ‚ΠΎ Π² случаС двиТСния Ρ‚Π΅Π»Π° ΠΏΠΎ Π³ΠΎΡ€ΠΈΠ·ΠΎΠ½Ρ‚Π°Π»ΡŒΠ½ΠΎΠΉ повСрхности сила Π½ΠΎΡ€ΠΌΠ°Π»ΡŒΠ½ΠΎΠ³ΠΎ давлСния Ρ€Π°Π²Π½Π° силС тяТСсти, Π΄Π΅ΠΉΡΡ‚Π²ΡƒΡŽΡ‰Π΅ΠΉ Π½Π° это Ρ‚Π΅Π»ΠΎ: N = P . Π­Ρ‚ΠΎ позволяСт Π²Ρ‹Ρ‡ΠΈΡΠ»ΠΈΡ‚ΡŒ коэффициСнт трСния:

Π¦Π΅Π½Π° дСлСния ΡˆΠΊΠ°Π»Ρ‹ Π΄ΠΈΠ½Π°ΠΌΠΎΠΌΠ΅Ρ‚Ρ€Π°, Ρ†.Π΄ = 0,1 Н.

1. ΠžΠΏΡ€Π΅Π΄Π΅Π»ΠΈΠ»ΠΈ вСс бруска ΠΈ Π³Ρ€ΡƒΠ·Π° с ΠΏΠΎΠΌΠΎΡ‰ΡŒΡŽ Π΄ΠΈΠ½Π°ΠΌΠΎΠΌΠ΅Ρ‚Ρ€Π°, записали Π² Ρ‚Π°Π±Π»ΠΈΡ†Ρƒ.

2. Двигая брусок Ρ€Π°Π²Π½ΠΎΠΌΠ΅Ρ€Π½ΠΎ ΠΏΠΎ дСрСвянной Π»ΠΈΠ½Π΅ΠΉΠΊΠ΅, ΠΎΠΏΡ€Π΅Π΄Π΅Π»ΠΈΠ»ΠΈ силу тяги, которая Ρ€Π°Π²Π½Π° силС трСния. Записали Π΅Π΅ Π·Π½Π°Ρ‡Π΅Π½ΠΈΠ΅ Π² Ρ‚Π°Π±Π»ΠΈΡ†Ρƒ.

3. ΠžΠΏΡ€Π΅Π΄Π΅Π»ΠΈΠ»ΠΈ коэффициСнт трСния для ΠΊΠ°ΠΆΠ΄ΠΎΠ³ΠΎ измСрСния силы трСния, занСсли ΠΈΡ… Π² Ρ‚Π°Π±Π»ΠΈΡ†Ρƒ.

4. ΠžΠΏΡ€Π΅Π΄Π΅Π»ΠΈΠ»ΠΈ ΠΏΠΎΠ³Ρ€Π΅ΡˆΠ½ΠΎΡΡ‚ΡŒ измСрСния для ΠΊΠ°ΠΆΠ΄ΠΎΠ³ΠΎ значСния коэффициСнта силы трСния.

1. ΠšΠΎΡΡ„Ρ„ΠΈΡ†ΠΈΠ΅Π½Ρ‚ трСния Ρ€Π°Π²Π΅Π½ 0,2.
2. Π˜Π½ΡΡ‚Ρ€ΡƒΠΌΠ΅Π½Ρ‚Π°Π»ΡŒΠ½Π°Ρ ΠΏΠΎΠ³Ρ€Π΅ΡˆΠ½ΠΎΡΡ‚ΡŒ измСрСния Ρ€Π°Π²Π½Π° 0,06.
3. ΠšΠΎΡΡ„Ρ„ΠΈΡ†ΠΈΠ΅Π½Ρ‚ трСния скольТСния ΠΏΡ€ΠΈ Π²Π·Π°ΠΈΠΌΠ½ΠΎΠΌ Π΄Π²ΠΈΠΆΠ΅Π½ΠΈΠΈ Ρ‚Π΅Π»Π° ΠΏΠΎ повСрхности стола являСтся Π²Π΅Π»ΠΈΡ‡ΠΈΠ½ΠΎΠΉ постоянной Π½Π΅ зависящСй ΠΎΡ‚ силы Π½ΠΎΡ€ΠΌΠ°Π»ΡŒΠ½ΠΎΠ³ΠΎ давлСния.

2. Π‘Ρ€Π°Π²Π½ΠΈΡ‚Π΅ коэффициСнт трСния покоя, скольТСния ΠΈ качСния. Π‘Π΄Π΅Π»Π°ΠΉΡ‚Π΅ Π²Ρ‹Π²ΠΎΠ΄.

ΠžΠ±ΠΎΡ€ΡƒΠ΄ΠΎΠ²Π°Π½ΠΈΠ΅: Π΄ΠΈΠ½Π°ΠΌΠΎΠΌΠ΅Ρ‚Ρ€, брусок дСрСвянный, Π³Ρ€ΡƒΠ·Ρ‹ с двумя ΠΊΡ€ΡŽΡ‡ΠΊΠ°ΠΌΠΈ – 2 ΡˆΡ‚., ΠΊΠ°Ρ€Π°Π½Π΄Π°ΡˆΠΈ ΠΊΡ€ΡƒΠ³Π»Ρ‹Π΅ – 2 ΡˆΡ‚.

ΠŸΠΎΡ€ΡΠ΄ΠΎΠΊ выполнСния Ρ€Π°Π±ΠΎΡ‚Ρ‹.

2. Π˜Π·ΠΌΠ΅Ρ€ΡŒΡ‚Π΅ вСс бруска с двумя Π³Ρ€ΡƒΠ·Π°ΠΌΠΈ ΠΏΡ€ΠΈ ΠΏΠΎΠΌΠΎΡ‰ΠΈ Π΄ΠΈΠ½Π°ΠΌΠΎΠΌΠ΅Ρ‚Ρ€Π°. Π Π΅Π·ΡƒΠ»ΡŒΡ‚Π°Ρ‚ измСрСния вСса Π·Π°ΠΏΠΈΡˆΠΈΡ‚Π΅ Π² Ρ‚Π΅Ρ‚Ρ€Π°Π΄ΡŒ.

3. Π˜Π·ΠΌΠ΅Ρ€ΡŒΡ‚Π΅ ΠΌΠ°ΠΊΡΠΈΠΌΠ°Π»ΡŒΠ½ΡƒΡŽ силу трСния покоя бруска ΠΏΠΎ столу. Для этого ΠΏΠΎΠ»ΠΎΠΆΠΈΡ‚Π΅ брусок Π½Π° стол, Π° Π½Π° брусок Π΄Π²Π° Π³Ρ€ΡƒΠ·Π°; ΠΊ бруску ΠΏΡ€ΠΈΡ†Π΅ΠΏΠΈΡ‚Π΅ Π΄ΠΈΠ½Π°ΠΌΠΎΠΌΠ΅Ρ‚Ρ€ ΠΈ ΠΏΡ€ΠΈΠ²Π΅Π΄ΠΈΡ‚Π΅ брусок с Π³Ρ€ΡƒΠ·Π°ΠΌΠΈ Π² Π΄Π²ΠΈΠΆΠ΅Π½ΠΈΠ΅. Π—Π°ΠΏΠΈΡˆΠΈΡ‚Π΅ показания Π΄ΠΈΠ½Π°ΠΌΠΎΠΌΠ΅Ρ‚Ρ€Π°, ΡΠΎΠΎΡ‚Π²Π΅Ρ‚ΡΡ‚Π²ΡƒΡŽΡ‰Π΅Π΅ Π½Π°Ρ‡Π°Π»Ρƒ двиТСния бруска.

4. Π˜Π·ΠΌΠ΅Ρ€ΡŒΡ‚Π΅ силу трСния скольТСния бруска с Π³Ρ€ΡƒΠ·Π°ΠΌΠΈ ΠΏΠΎ столу. Для этого ΠΏΠ΅Ρ€Π΅ΠΌΠ΅Ρ‰Π°ΠΉΡ‚Π΅ брусок с Π³Ρ€ΡƒΠ·Π°ΠΌΠΈ Ρ€Π°Π²Π½ΠΎΠΌΠ΅Ρ€Π½ΠΎ ΠΏΠΎ столу ΠΏΡ€ΠΈ ΠΏΠΎΠΌΠΎΡ‰ΠΈ Π΄ΠΈΠ½Π°ΠΌΠΎΠΌΠ΅Ρ‚Ρ€Π°. Π Π΅Π·ΡƒΠ»ΡŒΡ‚Π°Ρ‚ измСрСния силы Π·Π°ΠΏΠΈΡˆΠΈΡ‚Π΅ Π² Ρ‚Π΅Ρ‚Ρ€Π°Π΄ΡŒ.

5. Π˜Π·ΠΌΠ΅Ρ€ΡŒΡ‚Π΅ силу трСния качСния бруска ΠΏΠΎ столу. Для этого ΠΏΠΎΠ»ΠΎΠΆΠΈΡ‚Π΅ брусок с двумя Π³Ρ€ΡƒΠ·Π°ΠΌΠΈ Π½Π° Π΄Π²Π° ΠΊΡ€ΡƒΠ³Π»Ρ‹Ρ… ΠΊΠ°Ρ€Π°Π½Π΄Π°ΡˆΠ° ΠΈ ΠΏΠ΅Ρ€Π΅ΠΌΠ΅Ρ‰Π°ΠΉΡ‚Π΅ Ρ€Π°Π²Π½ΠΎΠΌΠ΅Ρ€Π½ΠΎ брусок ΠΏΠΎ столу ΠΏΡ€ΠΈ ΠΏΠΎΠΌΠΎΡ‰ΠΈ Π΄ΠΈΠ½Π°ΠΌΠΎΠΌΠ΅Ρ‚Ρ€Π°. Π Π΅Π·ΡƒΠ»ΡŒΡ‚Π°Ρ‚ измСрСния силы Π·Π°ΠΏΠΈΡˆΠΈΡ‚Π΅ Π² Ρ‚Π΅Ρ‚Ρ€Π°Π΄ΡŒ.

6. Π‘Π΄Π΅Π»Π°ΠΉΡ‚Π΅ Π²Ρ‹Π²ΠΎΠ΄ ΠΎ Ρ‚ΠΎΠΌ, какая сила большС:
Π°) вСс Ρ‚Π΅Π»Π° ΠΈΠ»ΠΈ максимальная сила трСния покоя?
Π±) максимальная сила трСния покоя ΠΈΠ»ΠΈ сила трСния скольТСния?
Π²) сила трСния скольТСния ΠΈΠ»ΠΈ сила трСния качСния?

7. Π‘Ρ€Π°Π²Π½ΠΈΡ‚Π΅ коэффициСнт трСния покоя, трСния скольТСния ΠΈ трСния качСния.

Π°) ВСс Ρ‚Π΅Π»Π° большС Ρ‡Π΅ΠΌ максимальная сила трСния покоя.

Π±) Максимальная сила трСния покоя большС Ρ‡Π΅ΠΌ сила трСния скольТСния.

Π²) Π‘ΠΈΠ»Π° трСния скольТСния большС Ρ‡Π΅ΠΌ сила трСния качСния.

Π³) ΠŸΡ€ΠΈ Π½Π΅ΠΈΠ·ΠΌΠ΅Π½Π½ΠΎΠΌ вСсС Ρ‚Π΅Π»Π°, наимСньшСС Π·Π½Π°Ρ‡Π΅Π½ΠΈΠ΅ коэффициСнт трСния ΠΈΠΌΠ΅Π΅Ρ‚ ΠΏΡ€ΠΈ ΠΊΠ°Ρ‡Π΅Π½ΠΈΠΈ Ρ‚Π΅Π»Π°, Π° наибольшСС Π² случаС покоя.

3. ΠžΠΏΡ€Π΅Π΄Π΅Π»ΠΈΡ‚Π΅ коэффициСнт трСния скольТСния ΠΏΡ€ΠΈ Π΄Π²ΠΈΠΆΠ΅Π½ΠΈΠΈ бруска вдоль повСрхности Ρ€Π΅Π·ΠΈΠ½Ρ‹, Π½Π΅ΡˆΠ»ΠΈΡ„ΠΎΠ²Π°Π½Π½ΠΎΠΉ дСрСвянной Ρ€Π΅ΠΉΠΊΠΈ, Π½Π°ΠΆΠ΄Π°Ρ‡Π½ΠΎΠΉ Π±ΡƒΠΌΠ°Π³ΠΈ.

ΠžΠ±ΠΎΡ€ΡƒΠ΄ΠΎΠ²Π°Π½ΠΈΠ΅: Π΄ΠΈΠ½Π°ΠΌΠΎΠΌΠ΅Ρ‚Ρ€, брусок дСрСвянный, Π³Ρ€ΡƒΠ·Ρ‹ с двумя ΠΊΡ€ΡŽΡ‡ΠΊΠ°ΠΌΠΈ – 2 ΡˆΡ‚., ΠΎΡ‚Ρ€Π΅Π· Π»ΠΈΠ½ΠΎΠ»Π΅ΡƒΠΌΠ°, дСрСвянная Π½Π΅ΡˆΠ»ΠΈΡ„ΠΎΠ²Π°Π½Π½Π°Ρ Ρ€Π΅ΠΉΠΊΠ°, наТдачная Π±ΡƒΠΌΠ°Π³Π°.

ΠŸΠΎΡ€ΡΠ΄ΠΎΠΊ выполнСния Ρ€Π°Π±ΠΎΡ‚Ρ‹.

1. ВычислитС Ρ†Π΅Π½Ρƒ дСлСния ΡˆΠΊΠ°Π»Ρ‹ Π΄ΠΈΠ½Π°ΠΌΠΎΠΌΠ΅Ρ‚Ρ€Π°.
2. Π˜Π·ΠΌΠ΅Ρ€ΡŒΡ‚Π΅ вСс бруска ΠΏΡ€ΠΈ ΠΏΠΎΠΌΠΎΡ‰ΠΈ Π΄ΠΈΠ½Π°ΠΌΠΎΠΌΠ΅Ρ‚Ρ€Π°. Π Π΅Π·ΡƒΠ»ΡŒΡ‚Π°Ρ‚ измСрСния вСса Π·Π°ΠΏΠΈΡˆΠΈΡ‚Π΅ Π² Ρ‚Π°Π±Π»ΠΈΡ†Ρƒ.
3. Π˜Π·ΠΌΠ΅Ρ€ΡŒΡ‚Π΅ силу трСния скольТСния бруска с Π³Ρ€ΡƒΠ·Π°ΠΌΠΈ ΠΏΠΎ повСрхности Ρ€Π΅Π·ΠΈΠ½Ρ‹, дСрСвянной Π½Π΅ΡˆΠ»ΠΈΡ„ΠΎΠ²Π°Π½Π½ΠΎΠΉ Π»ΠΈΠ½Π΅ΠΉΠΊΠΈ ΠΈ ΠΏΠΎ повСрхности Π½Π°ΠΆΠ΄Π°Ρ‡Π½ΠΎΠΉ Π±ΡƒΠΌΠ°Π³ΠΈ. Для этого ΠΏΠ΅Ρ€Π΅ΠΌΠ΅Ρ‰Π°ΠΉΡ‚Π΅ брусок с Π³Ρ€ΡƒΠ·Π°ΠΌΠΈ Ρ€Π°Π²Π½ΠΎΠΌΠ΅Ρ€Π½ΠΎ ΠΏΠΎ столу ΠΏΡ€ΠΈ ΠΏΠΎΠΌΠΎΡ‰ΠΈ Π΄ΠΈΠ½Π°ΠΌΠΎΠΌΠ΅Ρ‚Ρ€Π°. Π Π΅Π·ΡƒΠ»ΡŒΡ‚Π°Ρ‚ измСрСния Π·Π°ΠΏΠΈΡˆΠΈΡ‚Π΅ Π² Ρ‚Π°Π±Π»ΠΈΡ†Ρƒ.
4. ВычислитС коэффициСнт трСния скольТСния.
5. Π‘Π΄Π΅Π»Π°ΠΉΡ‚Π΅ Π²Ρ‹Π²ΠΎΠ΄.

Π¦Π΅Π½Π° дСлСния ΡˆΠΊΠ°Π»Ρ‹ Π΄ΠΈΠ½Π°ΠΌΠΎΠΌΠ΅Ρ‚Ρ€Π°, Ρ†.Π΄ = 0,1 Н.

1. Π‘ΠΈΠ»Π° трСния:

Π°) зависит ΠΎΡ‚ Ρ€ΠΎΠ΄Π° трущихся повСрхностСй.
Π±) зависит ΠΎΡ‚ ΡˆΠ΅Ρ€ΠΎΡ…ΠΎΠ²Π°Ρ‚ΠΎΡΡ‚ΠΈ трущихся повСрхностСй.
Π²) Ρ‡Π΅ΠΌ большС ΡˆΠ΅Ρ€ΠΎΡ…ΠΎΠ²Π°Ρ‚ΠΎΡΡ‚ΠΈ повСрхности, Ρ‚Π΅ΠΌ коэффициСнт трСния большС.

2. Бпособы увСличСния ΠΈΠ»ΠΈ ΡƒΠΌΠ΅Π½ΡŒΡˆΠ΅Π½ΠΈΡ силы трСния скольТСния:

Π£Π²Π΅Π»ΠΈΡ‡ΠΈΡ‚ΡŒ: ΡƒΠ²Π΅Π»ΠΈΡ‡ΠΈΡ‚ΡŒ ΡˆΠ΅Ρ€ΠΎΡ…ΠΎΠ²Π°Ρ‚ΠΎΡΡ‚ΡŒ трущихся повСрхностСй, Π½Π°ΡΡ‹ΠΏΠ°Ρ‚ΡŒ ΠΌΠ΅ΠΆΠ΄Ρƒ трущихся повСрхностСй частицы (струТку, ΠΎΠΏΠΈΠ»ΠΊΠΈ, пСсок).

Π£ΠΌΠ΅Π½ΡŒΡˆΠΈΡ‚ΡŒ: ΡˆΠ»ΠΈΡ„ΠΎΠ²ΠΊΠ°, ΠΏΠΎΠ»ΠΈΡ€ΠΎΠ²ΠΊΠ° трущихся повСрхностСй, нанСсСниС смазки.

Π—Π°Π΄Π°Π½ΠΈΠ΅ Π³Ρ€ΡƒΠΏΠΏΠ΅ II.

Π˜Π·ΠΌΠ΅Ρ€Π΅Π½ΠΈΠ΅ коэффициСнт трСния скольТСния, ΠΈΡΠΏΠΎΠ»ΡŒΠ·ΡƒΡ Π½Π°ΠΊΠ»ΠΎΠ½Π½ΡƒΡŽ ΠΏΠ»ΠΎΡΠΊΠΎΡΡ‚ΡŒ

ΠžΠ±ΠΎΡ€ΡƒΠ΄ΠΎΠ²Π°Π½ΠΈΠ΅ : Π»ΠΈΠ½Π΅ΠΉΠΊΠ° дСрСвянная ΠΎΡ‚ Ρ‚Ρ€ΠΈΠ±ΠΎΠΌΠ΅Ρ‚Ρ€Π°, брусок дСрСвянный, Π»ΠΈΠ½Π΅ΠΉΠΊΠ° ΠΈΠ·ΠΌΠ΅Ρ€ΠΈΡ‚Π΅Π»ΡŒΠ½Π°Ρ, ΡˆΡ‚Π°Ρ‚ΠΈΠ².

ΠŸΠΎΡ€ΡΠ΄ΠΎΠΊ выполнСния Ρ€Π°Π±ΠΎΡ‚Ρ‹ .

1. Π˜ΡΠΏΠΎΠ»ΡŒΠ·ΡƒΡ ΡˆΡ‚Π°Ρ‚ΠΈΠ², Π·Π°ΠΊΡ€Π΅ΠΏΠΈΡ‚Π΅ Π»ΠΈΠ½Π΅ΠΉΠΊΡƒ ΠΏΠΎΠ΄ ΡƒΠ³Π»ΠΎΠΌ ΠΊ столу.
2. ΠŸΠΎΠ»ΠΎΠΆΠΈΡ‚Π΅ брусок Π½Π° Π·Π°ΠΊΡ€Π΅ΠΏΠ»Π΅Π½Π½ΡƒΡŽ ΠΏΠΎΠ΄ ΡƒΠ³Π»ΠΎΠΌ Π΄Π΅Ρ€Π΅Π²ΡΠ½Π½ΡƒΡŽ Π»ΠΈΠ½Π΅ΠΉΠΊΡƒ.
3. МСняя ΡƒΠ³ΠΎΠ» Π½Π°ΠΊΠ»ΠΎΠ½Π° Π»ΠΈΠ½Π΅ΠΉΠΊΠΈ, Π½Π°ΠΉΠ΄ΠΈΡ‚Π΅ Ρ‚Π°ΠΊΠΎΠΉ ΠΌΠ°ΠΊΡΠΈΠΌΠ°Π»ΡŒΠ½Ρ‹ΠΉ ΡƒΠ³ΠΎΠ», ΠΏΡ€ΠΈ ΠΊΠΎΡ‚ΠΎΡ€ΠΎΠΌ брусок Π΅Ρ‰Π΅ покоится.
4. Π˜Π·ΠΌΠ΅Ρ€ΡŒΡ‚Π΅ Π΄Π»ΠΈΠ½Ρƒ основания Π»ΠΈΠ½Π΅ΠΉΠΊΠΈ ΠΈ высоту подъСма Π»ΠΈΠ½Π΅ΠΉΠΊΠΈ.
5. РассчитайтС Π·Π½Π°Ρ‡Π΅Π½ΠΈΠ΅ коэффициСнта трСния скольТСния Π΄Π΅Ρ€Π΅Π²Π° ΠΎ Π΄Π΅Ρ€Π΅Π²ΠΎ ΠΏΠΎ Ρ„ΠΎΡ€ΠΌΡƒΠ»Π΅:

6. РассчитайтС ΠΏΠΎΠ³Ρ€Π΅ΡˆΠ½ΠΎΡΡ‚ΡŒ измСрСния.
7. Π’Ρ‹Π²ΠΎΠ΄.

Π­ΠΊΡΠΏΠ΅Ρ€ΠΈΠΌΠ΅Π½Ρ‚Π°Π»ΡŒΠ½Ρ‹Π΅ Π΄Π°Π½Π½Ρ‹Π΅.

Π˜Π·ΠΌΠ΅Ρ€ΠΈΠ»ΠΈ высоту подъСма ΠΈ Π΄Π»ΠΈΠ½Ρƒ основания Π»ΠΈΠ½Π΅ΠΉΠΊΠΈ.

1. ΠšΠΎΡΡ„Ρ„ΠΈΡ†ΠΈΠ΅Π½Ρ‚ трСния Ρ€Π°Π²Π΅Π½ 0,3.
2. ΠŸΠΎΠ³Ρ€Π΅ΡˆΠ½ΠΎΡΡ‚ΡŒ измСрСния Ρ€Π°Π²Π½Π° 0,0016.

2. Π˜Π·ΠΌΠ΅Ρ€Π΅Π½ΠΈΠ΅ коэффициСнта трСния скольТСния, Ρ‡Π΅Ρ€Π΅Π· ΠΎΠΏΡ€ΠΎΠΊΠΈΠ΄Ρ‹Π²Π°Π½ΠΈΠ΅ бруска

ΠžΠ±ΠΎΡ€ΡƒΠ΄ΠΎΠ²Π°Π½ΠΈΠ΅: брусок дСрСвянный, Π»ΠΈΠ½Π΅ΠΉΠΊΠ° дСрСвянная ΠΎΡ‚ Ρ‚Ρ€ΠΈΠ±ΠΎΠΌΠ΅Ρ‚Ρ€Π°, Π½ΠΈΡ‚ΡŒ, Π»ΠΈΠ½Π΅ΠΉΠΊΠ° учСничСская.

ΠŸΠΎΡ€ΡΠ΄ΠΎΠΊ выполнСния Ρ€Π°Π±ΠΎΡ‚Ρ‹.

ВСорСтичСскоС обоснованиС: Брусок с привязанной ΠΊ Π΄Π»ΠΈΠ½Π½ΠΎΠΉ Π³Ρ€Π°Π½ΠΈ Π½ΠΈΡ‚ΡŒΡŽ ΠΏΠΎΡΡ‚Π°Π²ΡŒΡ‚Π΅ Ρ‚ΠΎΡ€Ρ†ΠΎΠΌ Π½Π° Π³ΠΎΡ€ΠΈΠ·ΠΎΠ½Ρ‚Π°Π»ΡŒΠ½ΡƒΡŽ ΠΏΠΎΠ²Π΅Ρ€Ρ…Π½ΠΎΡΡ‚ΡŒ стола ΠΈ тянитС Π·Π° Π½ΠΈΡ‚ΡŒ. Если Π½ΠΈΡ‚ΡŒ Π·Π°ΠΊΡ€Π΅ΠΏΠ»Π΅Π½Π° нСвысоко Π½Π°Π΄ ΠΏΠΎΠ²Π΅Ρ€Ρ…Π½ΠΎΡΡ‚ΡŒΡŽ стола, Ρ‚ΠΎ брусок Π±ΡƒΠ΄Π΅Ρ‚ ΡΠΊΠΎΠ»ΡŒΠ·ΠΈΡ‚ΡŒ. ΠŸΡ€ΠΈ ΠΎΠΏΡ€Π΅Π΄Π΅Π»Π΅Π½Π½ΠΎΠΉ высотС h Ρ‚ΠΎΡ‡ΠΊΠΈ А крСплСния Π½ΠΈΡ‚ΠΈ сила натяТСния Π½ΠΈΡ‚ΠΈ F ΠΎΠΏΡ€ΠΎΠΊΠΈΠ΄Ρ‹Π²Π°Π΅Ρ‚ брусок.

Условия равновСсия для этого случая ΠΎΡ‚Π½ΠΎΡΠΈΡ‚Π΅Π»ΡŒΠ½ΠΎ Ρ‚ΠΎΡ‡ΠΊΠΈ – ΡƒΠ³Π»Π° опрокидывания:

Fh – mga/2 = 0;

Богласно II Π·Π°ΠΊΠΎΠ½Ρƒ ΠΡŒΡŽΡ‚ΠΎΠ½Π°: F – FΡ‚Ρ€ = 0;

ΠžΠ±Ρ€Π°Π±ΠΎΡ‚ΠΊΠ° Ρ€Π΅Π·ΡƒΠ»ΡŒΡ‚Π°Ρ‚ΠΎΠ².

4. Π‘Π΄Π΅Π»Π°ΠΉΡ‚Π΅ Π²Ρ‹Π²ΠΎΠ΄.

Π­ΠΊΡΠΏΠ΅Ρ€ΠΈΠΌΠ΅Π½Ρ‚Π°Π»ΡŒΠ½Ρ‹ΠΉ расчСт.

a = 45 Β± 1 ΠΌΠΌ, h = 80 Β± 1 ΠΌΠΌ.

1. ΠšΠΎΡΡ„Ρ„ΠΈΡ†ΠΈΠ΅Π½Ρ‚ трСния Ρ€Π°Π²Π΅Π½ 0,28.
2. Π˜Π½ΡΡ‚Ρ€ΡƒΠΌΠ΅Π½Ρ‚Π°Π»ΡŒΠ½Π°Ρ ΠΏΠΎΠ³Ρ€Π΅ΡˆΠ½ΠΎΡΡ‚ΡŒ измСрСния Ρ€Π°Π²Π½Π° 0,0098.

3. Π˜Π·ΠΌΠ΅Ρ€Π΅Π½ΠΈΠ΅ коэффициСнта трСния скольТСния с ΠΏΠΎΠΌΠΎΡ‰ΡŒΡŽ ΠΊΠ°Ρ€Π°Π½Π΄Π°ΡˆΠ°.

ΠžΠ±ΠΎΡ€ΡƒΠ΄ΠΎΠ²Π°Π½ΠΈΠ΅: ΠΊΠ°Ρ€Π°Π½Π΄Π°Ρˆ, Π»ΠΈΠ½Π΅ΠΉΠΊΠ° дСрСвянная ΠΎΡ‚ Ρ‚Ρ€ΠΈΠ±ΠΎΠΌΠ΅Ρ‚Ρ€Π°, Π»ΠΈΠ½Π΅ΠΉΠΊΠ° учСничСская.

ΠŸΠΎΡ€ΡΠ΄ΠΎΠΊ выполнСния Ρ€Π°Π±ΠΎΡ‚Ρ‹.

ВСорСтичСскоС обоснованиС: ΠŸΠΎΡΡ‚Π°Π²ΡŒΡ‚Π΅ ΠΊΠ°Ρ€Π°Π½Π΄Π°Ρˆ Π½Π° стол Π²Π΅Ρ€Ρ‚ΠΈΠΊΠ°Π»ΡŒΠ½ΠΎ, Π½Π°ΠΆΠΌΠΈΡ‚Π΅ Π½Π° Π½Π΅Π³ΠΎ, Π½Π°ΠΊΠ»ΠΎΠ½ΠΈΡ‚Π΅ ΠΈ Π½Π°Π±Π»ΡŽΠ΄Π°ΠΉΡ‚Π΅ Ρ…Π°Ρ€Π°ΠΊΡ‚Π΅Ρ€ Π΅Π³ΠΎ падСния. ΠŸΡ€ΠΈ Π½Π΅Π±ΠΎΠ»ΡŒΡˆΠΈΡ… ΡƒΠ³Π»Π°Ρ… Π½Π°ΠΊΠ»ΠΎΠ½Π° ΠΊ Π²Π΅Ρ€Ρ‚ΠΈΠΊΠ°Π»ΠΈ ΠΊΠ°Ρ€Π°Π½Π΄Π°Ρˆ Π½Π΅ ΠΏΡ€ΠΎΡΠΊΠ°Π»ΡŒΠ·Ρ‹Π²Π°Π΅Ρ‚ ΠΎΡ‚Π½ΠΎΡΠΈΡ‚Π΅Π»ΡŒΠ½ΠΎ повСрхности стола ΠΏΡ€ΠΈ любой Π²Π΅Π»ΠΈΡ‡ΠΈΠ½Π΅ силы, ΠΏΡ€ΠΈΠΆΠΈΠΌΠ°ΡŽΡ‰Π΅ΠΉ Π΅Π³ΠΎ ΠΊ столу. ΠŸΡ€ΠΎΡΠΊΠ°Π»ΡŒΠ·Ρ‹Π²Π°Π½ΠΈΠ΅ начинаСтся с Π½Π΅ΠΊΠΎΡ‚ΠΎΡ€ΠΎΠ³ΠΎ критичСского ΡƒΠ³Π»Π°, зависящСго ΠΎΡ‚ силы трСния.

ЗаписываСм Π²Ρ‚ΠΎΡ€ΠΎΠΉ Π·Π°ΠΊΠΎΠ½ ΠΡŒΡŽΡ‚ΠΎΠ½Π° Π² проСкциях Π½Π° ΠΊΠΎΠΎΡ€Π΄ΠΈΠ½Π°Ρ‚Π½Ρ‹Π΅ оси ΠΏΡ€ΠΈ ΡƒΠ³Π»Π΅ Π½Π°ΠΊΠ»ΠΎΠ½Π°, Ρ€Π°Π²Π½ΠΎΠΌ критичСскому. (Π‘ΠΈΠ»ΠΎΠΉ тяТСсти mg, Π΄Π΅ΠΉΡΡ‚Π²ΡƒΡŽΡ‰Π΅ΠΉ Π½Π° ΠΊΠ°Ρ€Π°Π½Π΄Π°Ρˆ, ΠΏΠΎ ΡΡ€Π°Π²Π½Π΅Π½ΠΈΡŽ с большой силой F ΠΏΡ€Π΅Π½Π΅Π±Ρ€Π΅Π³Π°Π΅ΠΌ).

ΠžΠ±Ρ€Π°Π±ΠΎΡ‚ΠΊΠ° Ρ€Π΅Π·ΡƒΠ»ΡŒΡ‚Π°Ρ‚ΠΎΠ²:

1. РассчитайтС ΠΏΠΎ Ρ„ΠΎΡ€ΠΌΡƒΠ»Π΅ Π·Π½Π°Ρ‡Π΅Π½ΠΈΠ΅ коэффициСнта трСния скольТСния Π΄Π΅Ρ€Π΅Π²Π° ΠΎ Π΄Π΅Ρ€Π΅Π²ΠΎ.
2. ΠžΠΏΡ€Π΅Π΄Π΅Π»ΠΈΡ‚Π΅ ΠΏΠΎΠ³Ρ€Π΅ΡˆΠ½ΠΎΡΡ‚ΡŒ ΠΈΠ·ΠΌΠ΅Ρ€Π΅Π½ΠΈΠΉ.
3. Π—Π°ΠΏΠΈΡˆΠΈΡ‚Π΅ ΠΏΠΎΠ»ΡƒΡ‡Π΅Π½Π½Ρ‹ΠΉ ΠΎΡ‚Π²Π΅Ρ‚ с ΡƒΡ‡Π΅Ρ‚ΠΎΠΌ Π΄ΠΎΠΏΡƒΡ‰Π΅Π½Π½Ρ‹Ρ… ΠΏΠΎΠ³Ρ€Π΅ΡˆΠ½ΠΎΡΡ‚Π΅ΠΉ ΠΈΠ·ΠΌΠ΅Ρ€Π΅Π½ΠΈΠΉ.
4. Π‘Π΄Π΅Π»Π°ΠΉΡ‚Π΅ Π²Ρ‹Π²ΠΎΠ΄.

Π­ΠΊΡΠΏΠ΅Ρ€ΠΈΠΌΠ΅Π½Ρ‚Π°Π»ΡŒΠ½Ρ‹ΠΉ расчСт.

1. ΠžΠ±Ρ€Π°Π±ΠΎΡ‚ΠΊΠ° Ρ€Π΅Π·ΡƒΠ»ΡŒΡ‚Π°Ρ‚ΠΎΠ²

Ξ± = 30 0 ,

Β΅= tgΞ± = sina /cosa

1. ΠšΠΎΡΡ„Ρ„ΠΈΡ†ΠΈΠ΅Π½Ρ‚ трСния Ρ€Π°Π²Π΅Π½ 0,58.

III. ПодвСдСниС ΠΈΡ‚ΠΎΠ³ΠΎΠ² ΠΏΡ€Π°ΠΊΡ‚ΠΈΠΊΡƒΠΌΠ°:

Π‘ΠΈΠ»Π° трСния скольТСния зависит:

Π°) ΠžΡ‚ Ρ€ΠΎΠ΄Π° трущихся повСрхностСй.
Π±) ΠžΡ‚ ΡˆΠ΅Ρ€ΠΎΡ…ΠΎΠ²Π°Ρ‚ΠΎΡΡ‚ΠΈ трущихся повСрхностСй.
Π²) ΠŸΡ€ΡΠΌΠΎ ΠΏΡ€ΠΎΠΏΠΎΡ€Ρ†ΠΈΠΎΠ½Π°Π»ΡŒΠ½ΠΎ ΠΎΡ‚ силы давлСния.
Π³) ΠšΠΎΡΡ„Ρ„ΠΈΡ†ΠΈΠ΅Π½Ρ‚ трСния скольТСния ΠΏΡ€ΠΈ Π²Π·Π°ΠΈΠΌΠ½ΠΎΠΌ Π΄Π²ΠΈΠΆΠ΅Π½ΠΈΠΈ Ρ‚Π΅Π»Π° ΠΏΠΎ повСрхности являСтся Π²Π΅Π»ΠΈΡ‡ΠΈΠ½ΠΎΠΉ постоянной Π½Π΅ зависящСй ΠΎΡ‚ силы Π½ΠΎΡ€ΠΌΠ°Π»ΡŒΠ½ΠΎΠ³ΠΎ давлСния.
Π΄) Π§Π΅ΠΌ большС ΡˆΠ΅Ρ€ΠΎΡ…ΠΎΠ²Π°Ρ‚ΠΎΡΡ‚ΠΈ повСрхности, Ρ‚Π΅ΠΌ коэффициСнт трСния большС.

ΠžΠΏΡ€Π΅Π΄Π΅Π»Π΅Π½ΠΈΠ΅

Π‘ΠΈΠ»ΠΎΠΉ трСния Π½Π°Π·Ρ‹Π²Π°ΡŽΡ‚ силу, которая Π²ΠΎΠ·Π½ΠΈΠΊΠ°Π΅Ρ‚ ΠΏΡ€ΠΈ ΠΎΡ‚Π½ΠΎΡΠΈΡ‚Π΅Π»ΡŒΠ½ΠΎΠΌ ΠΏΠ΅Ρ€Π΅ΠΌΠ΅Ρ‰Π΅Π½ΠΈΠΈ (ΠΈΠ»ΠΈ ΠΏΠΎΠΏΡ‹Ρ‚ΠΊΠ΅ пСрСмСщСния) Ρ‚Π΅Π» ΠΈ являСтся Ρ€Π΅Π·ΡƒΠ»ΡŒΡ‚Π°Ρ‚ΠΎΠΌ сопротивлСния двиТСнию ΠΎΠΊΡ€ΡƒΠΆΠ°ΡŽΡ‰Π΅ΠΉ срСды ΠΈΠ»ΠΈ Π΄Ρ€ΡƒΠ³ΠΈΡ… Ρ‚Π΅Π».

Π‘ΠΈΠ»Ρ‹ трСния Π²ΠΎΠ·Π½ΠΈΠΊΠ°ΡŽΡ‚ Ρ‚ΠΎΠ³Π΄Π°, ΠΊΠΎΠ³Π΄Π° ΡΠΎΠΏΡ€ΠΈΠΊΠ°ΡΠ°ΡŽΡ‰ΠΈΠ΅ΡΡ Ρ‚Π΅Π»Π° (ΠΈΠ»ΠΈ ΠΈΡ… части) ΠΏΠ΅Ρ€Π΅ΠΌΠ΅Ρ‰Π°ΡŽΡ‚ΡΡ ΠΎΡ‚Π½ΠΎΡΠΈΡ‚Π΅Π»ΡŒΠ½ΠΎ Π΄Ρ€ΡƒΠ³ Π΄Ρ€ΡƒΠ³Π°. ΠŸΡ€ΠΈ этом Ρ‚Ρ€Π΅Π½ΠΈΠ΅, ΠΊΠΎΡ‚ΠΎΡ€ΠΎΠ΅ появляСтся ΠΏΡ€ΠΈ ΠΎΡ‚Π½ΠΎΡΠΈΡ‚Π΅Π»ΡŒΠ½ΠΎΠΌ ΠΏΠ΅Ρ€Π΅ΠΌΠ΅Ρ‰Π΅Π½ΠΈΠΈ ΡΠΎΠΏΡ€ΠΈΠΊΠ°ΡΠ°ΡŽΡ‰ΠΈΡ…ΡΡ Ρ‚Π΅Π», Π½Π°Π·Ρ‹Π²Π°ΡŽΡ‚ внСшним. Π’Ρ€Π΅Π½ΠΈΠ΅, Π²ΠΎΠ·Π½ΠΈΠΊΠ°ΡŽΡ‰Π΅Π΅ ΠΌΠ΅ΠΆΠ΄Ρƒ частями ΠΎΠ΄Π½ΠΎΠ³ΠΎ сплошного Ρ‚Π΅Π»Π° (Π³Π°Π·, ΠΆΠΈΠ΄ΠΊΠΎΡΡ‚ΡŒ) Π½Π°Π·Π²Π°Π½ΠΎ Π²Π½ΡƒΡ‚Ρ€Π΅Π½Π½ΠΈΠΌ.

Π‘ΠΈΠ»Π° трСния – это Π²Π΅ΠΊΡ‚ΠΎΡ€, ΠΊΠΎΡ‚ΠΎΡ€Ρ‹ΠΉ ΠΈΠΌΠ΅Π΅Ρ‚ Π½Π°ΠΏΡ€Π°Π²Π»Π΅Π½ΠΈΠ΅ вдоль ΠΊΠ°ΡΠ°Ρ‚Π΅Π»ΡŒΠ½ΠΎΠΉ ΠΊ трущимся повСрхностям (слоям). ΠŸΡ€ΠΈ этом эта сила Π½Π°ΠΏΡ€Π°Π²Π»Π΅Π½Π° Π² сторону противодСйствия ΠΎΡ‚Π½ΠΎΡΠΈΡ‚Π΅Π»ΡŒΠ½ΠΎΠΌΡƒ ΡΠΌΠ΅Ρ‰Π΅Π½ΠΈΡŽ этих повСрхностСй (слоСв). Π’Π°ΠΊ, Ссли Π΄Π²Π° слоя Тидкости ΠΏΠ΅Ρ€Π΅ΠΌΠ΅Ρ‰Π°ΡŽΡ‚ΡΡ Π΄Ρ€ΡƒΠ³ ΠΏΠΎ Π΄Ρ€ΡƒΠ³Ρƒ, ΠΏΡ€ΠΈ этом двиТутся с Ρ€Π°Π·Π»ΠΈΡ‡Π½Ρ‹ΠΌΠΈ скоростями, Ρ‚ΠΎ сила, которая ΠΏΡ€ΠΈΠ»ΠΎΠΆΠ΅Π½Π° ΠΊ слою, ΠΏΠ΅Ρ€Π΅ΠΌΠ΅Ρ‰Π°ΡŽΡ‰Π΅ΠΌΡƒΡΡ с большСй ΡΠΊΠΎΡ€ΠΎΡΡ‚ΡŒΡŽ, ΠΈΠΌΠ΅Π΅Ρ‚ Π½Π°ΠΏΡ€Π°Π²Π»Π΅Π½ΠΈΠ΅ Π² сторону, которая ΠΏΡ€ΠΎΡ‚ΠΈΠ²ΠΎΠΏΠΎΠ»ΠΎΠΆΠ½Π° двиТСнию. Π‘ΠΈΠ»Π° ΠΆΠ΅, которая воздСйствуСт Π½Π° слой, ΠΊΠΎΡ‚ΠΎΡ€Ρ‹ΠΉ двиТСтся с мСньшСй ΡΠΊΠΎΡ€ΠΎΡΡ‚ΡŒΡŽ, Π½Π°ΠΏΡ€Π°Π²Π»Π΅Π½Π° ΠΏΠΎ двиТСнию.

Π’ΠΈΠ΄Ρ‹ трСния

Π’Ρ€Π΅Π½ΠΈΠ΅, ΠΊΠΎΡ‚ΠΎΡ€ΠΎΠ΅ Π²ΠΎΠ·Π½ΠΈΠΊΠ°Π΅Ρ‚ ΠΌΠ΅ΠΆΠ΄Ρƒ повСрхностями Ρ‚Π²Π΅Ρ€Π΄Ρ‹Ρ… Ρ‚Π΅Π», Π½Π°Π·Ρ‹Π²Π°ΡŽΡ‚ сухим. Оно Π²ΠΎΠ·Π½ΠΈΠΊΠ°Π΅Ρ‚ Π½Π΅ Ρ‚ΠΎΠ»ΡŒΠΊΠΎ ΠΏΡ€ΠΈ скольТСнии повСрхностСй, Π½ΠΎ ΠΈ ΠΏΡ€ΠΈ ΠΏΠΎΠΏΡ‹Ρ‚ΠΊΠ΅ Π²Ρ‹Π·Π²Π°Ρ‚ΡŒ ΠΏΠ΅Ρ€Π΅ΠΌΠ΅Ρ‰Π΅Π½ΠΈΠ΅ повСрхностСй. ΠŸΡ€ΠΈ этом Π²ΠΎΠ·Π½ΠΈΠΊΠ°Π΅Ρ‚ сила трСния покоя. Π’Π½Π΅ΡˆΠ½Π΅Π΅ Ρ‚Ρ€Π΅Π½ΠΈΠ΅, ΠΊΠΎΡ‚ΠΎΡ€ΠΎΠ΅ появляСтся ΠΌΠ΅ΠΆΠ΄Ρƒ двиТущимися Ρ‚Π΅Π»Π°ΠΌΠΈ, Π½Π°Π·Ρ‹Π²Π°ΡŽΡ‚ кинСматичСским.

Π—Π°ΠΊΠΎΠ½Ρ‹ сухого трСния говорят ΠΎ Ρ‚ΠΎΠΌ, Ρ‡Ρ‚ΠΎ максимальная сила трСния покоя ΠΈ сила трСния скольТСния Π½Π΅ зависят ΠΎΡ‚ ΠΏΠ»ΠΎΡ‰Π°Π΄ΠΈ повСрхностСй соприкосновСния ΡΠΎΠΏΡ€ΠΈΠΊΠ°ΡΠ°ΡŽΡ‰ΠΈΡ…ΡΡ Ρ‚Π΅Π», ΠΏΠΎΠ΄Π²Π΅Ρ€ΠΆΠ΅Π½Π½Ρ‹Ρ… Ρ‚Ρ€Π΅Π½ΠΈΡŽ. Π­Ρ‚ΠΈ силы ΠΏΡ€ΠΎΠΏΠΎΡ€Ρ†ΠΈΠΎΠ½Π°Π»ΡŒΠ½Ρ‹ ΠΌΠΎΠ΄ΡƒΠ»ΡŽ силы Π½ΠΎΡ€ΠΌΠ°Π»ΡŒΠ½ΠΎΠ³ΠΎ давлСния (N), которая ΠΏΡ€ΠΈΠΆΠΈΠΌΠ°Π΅Ρ‚ трущиСся повСрхности:

Π³Π΄Π΅ – Π±Π΅Π·Ρ€Π°Π·ΠΌΠ΅Ρ€Π½Ρ‹ΠΉ коэффициСнт трСния (покоя ΠΈΠ»ΠΈ скольТСния). Π”Π°Π½Π½Ρ‹ΠΉ коэффициСнт зависит ΠΎΡ‚ ΠΏΡ€ΠΈΡ€ΠΎΠ΄Ρ‹ ΠΈ состояния повСрхностСй трущихся Ρ‚Π΅Π», Π½Π°ΠΏΡ€ΠΈΠΌΠ΅Ρ€ ΠΎΡ‚ наличия ΡˆΠ΅Ρ€ΠΎΡ…ΠΎΠ²Π°Ρ‚ΠΎΡΡ‚Π΅ΠΉ. Если Ρ‚Ρ€Π΅Π½ΠΈΠ΅ Π²ΠΎΠ·Π½ΠΈΠΊΠ°Π΅Ρ‚ ΠΊΠ°ΠΊ Ρ€Π΅Π·ΡƒΠ»ΡŒΡ‚Π°Ρ‚ скольТСния, Ρ‚ΠΎ коэффициСнт трСния являСтся Ρ„ΡƒΠ½ΠΊΡ†ΠΈΠ΅ΠΉ скорости. Π”ΠΎΠ²ΠΎΠ»ΡŒΠ½ΠΎ часто вмСсто коэффициСнта трСния ΠΏΡ€ΠΈΠΌΠ΅Π½ΡΡŽΡ‚ ΡƒΠ³ΠΎΠ» трСния, ΠΊΠΎΡ‚ΠΎΡ€Ρ‹ΠΉ Ρ€Π°Π²Π΅Π½:

Π£Π³ΠΎΠ» Ρ€Π°Π²Π΅Π½ ΠΌΠΈΠ½ΠΈΠΌΠ°Π»ΡŒΠ½ΠΎΠΌΡƒ ΡƒΠ³Π»Ρƒ Π½Π°ΠΊΠ»ΠΎΠ½Π° плоскости ΠΊ Π³ΠΎΡ€ΠΈΠ·ΠΎΠ½Ρ‚Ρƒ, ΠΏΡ€ΠΈ ΠΊΠΎΡ‚ΠΎΡ€ΠΎΠΌ Ρ‚Π΅Π»ΠΎ, Π»Π΅ΠΆΠ°Ρ‰Π΅Π΅ Π½Π° этой плоскости, Π½Π°Ρ‡ΠΈΠ½Π°Π΅Ρ‚ ΡΠΊΠΎΠ»ΡŒΠ·ΠΈΡ‚ΡŒ, ΠΏΠΎΠ΄ воздСйствиС силы тяТСсти.

Π‘ΠΎΠ»Π΅Π΅ Ρ‚ΠΎΡ‡Π½Ρ‹ΠΌ ΡΡ‡ΠΈΡ‚Π°ΡŽΡ‚ Π·Π°ΠΊΠΎΠ½ трСния, ΠΊΠΎΡ‚ΠΎΡ€Ρ‹ΠΉ ΠΏΡ€ΠΈΠ½ΠΈΠΌΠ°Π΅Ρ‚ Π²ΠΎ Π²Π½ΠΈΠΌΠ°Π½ΠΈΠ΅ силы притяТСния ΠΌΠ΅ΠΆΠ΄Ρƒ ΠΌΠΎΠ»Π΅ΠΊΡƒΠ»Π°ΠΌΠΈ Ρ‚Π΅Π», ΠΊΠΎΡ‚ΠΎΡ€Ρ‹Π΅ ΠΏΠΎΠ΄Π²Π΅Ρ€Π³Π°ΡŽΡ‚ΡΡ Ρ‚Ρ€Π΅Π½ΠΈΡŽ:

Π³Π΄Π΅ S – общая ΠΏΠ»ΠΎΡ‰Π°Π΄ΡŒ ΠΊΠΎΠ½Ρ‚Π°ΠΊΡ‚Π° Ρ‚Π΅Π», p 0 – Π΄ΠΎΠ±Π°Π²ΠΎΡ‡Π½ΠΎΠ΅ Π΄Π°Π²Π»Π΅Π½ΠΈΠ΅, ΠΊΠΎΡ‚ΠΎΡ€ΠΎΠ΅ вызываСтся силами молСкулярного притяТСния, – истинный коэффициСнт трСния.

Π’Ρ€Π΅Π½ΠΈΠ΅ ΠΌΠ΅ΠΆΠ΄Ρƒ Ρ‚Π²Π΅Ρ€Π΄Ρ‹ΠΌ Ρ‚Π΅Π»ΠΎΠΌ ΠΈ ΠΆΠΈΠ΄ΠΊΠΎΡΡ‚ΡŒΡŽ (ΠΈΠ»ΠΈ Π³Π°Π·ΠΎΠΌ) Π½Π°Π·Ρ‹Π²Π°ΡŽΡ‚ вязким (ΠΆΠΈΠ΄ΠΊΠΈΠΌ). Π‘ΠΈΠ»Π° вязкого трСния становится Ρ€Π°Π²Π½ΠΎΠΉ Π½ΡƒΠ»ΡŽ, Ссли ΡΠΊΠΎΡ€ΠΎΡΡ‚ΡŒ ΠΎΡ‚Π½ΠΎΡΠΈΡ‚Π΅Π»ΡŒΠ½ΠΎΠ³ΠΎ двиТСния Ρ‚Π΅Π» обращаСтся Π² Π½ΡƒΠ»ΡŒ.

ΠŸΡ€ΠΈ Π΄Π²ΠΈΠΆΠ΅Π½ΠΈΠΈ Ρ‚Π΅Π»Π° Π² Тидкости ΠΈΠ»ΠΈ Π³Π°Π·Π΅ ΠΏΠΎΡΠ²Π»ΡΡŽΡ‚ΡΡ силы сопротивлСния срСды, ΠΊΠΎΡ‚ΠΎΡ€Ρ‹Π΅ ΠΌΠΎΠ³ΡƒΡ‚ ΡΡ‚Π°Ρ‚ΡŒ сущСствСнно большС, Ρ‡Π΅ΠΌ силы трСния. Π’Π΅Π»ΠΈΡ‡ΠΈΠ½Π° силы трСния скольТСния зависит ΠΎΡ‚ Ρ„ΠΎΡ€ΠΌΡ‹, Ρ€Π°Π·ΠΌΠ΅Ρ€ΠΎΠ² ΠΈ состояния повСрхности Ρ‚Π΅Π»Π°, скорости двиТСния Ρ‚Π΅Π»Π° ΠΎΡ‚Π½ΠΎΡΠΈΡ‚Π΅Π»ΡŒΠ½ΠΎ срСды, вязкости срСды. ΠŸΡ€ΠΈ Π½Π΅ ΠΎΡ‡Π΅Π½ΡŒ Π±ΠΎΠ»ΡŒΡˆΠΈΡ… скоростях сила трСния вычисляСтся ΠΏΡ€ΠΈ ΠΏΠΎΠΌΠΎΡ‰ΠΈ Ρ„ΠΎΡ€ΠΌΡƒΠ»Ρ‹:

Π³Π΄Π΅ Π·Π½Π°ΠΊ минус ΠΎΠ·Π½Π°Ρ‡Π°Π΅Ρ‚, Ρ‡Ρ‚ΠΎ сила трСния ΠΈΠΌΠ΅Π΅Ρ‚ Π½Π°ΠΏΡ€Π°Π²Π»Π΅Π½ΠΈΠ΅ Π² сторону ΠΏΡ€ΠΎΡ‚ΠΈΠ²ΠΎΠΏΠΎΠ»ΠΎΠΆΠ½ΡƒΡŽ Π½Π°ΠΏΡ€Π°Π²Π»Π΅Π½ΠΈΡŽ Π²Π΅ΠΊΡ‚ΠΎΡ€Π° скорости. ΠŸΡ€ΠΈ ΡƒΠ²Π΅Π»ΠΈΡ‡Π΅Π½ΠΈΠΈ скоростСй двиТСния Ρ‚Π΅Π» Π² вязкой срСдС Π»ΠΈΠ½Π΅ΠΉΠ½Ρ‹ΠΉ Π·Π°ΠΊΠΎΠ½ (4) ΠΏΠ΅Ρ€Π΅Ρ…ΠΎΠ΄ΠΈΡ‚ Π² ΠΊΠ²Π°Π΄Ρ€Π°Ρ‚ΠΈΡ‡Π½Ρ‹ΠΉ:

ΠšΠΎΡΡ„Ρ„ΠΈΡ†ΠΈΠ΅Π½Ρ‚Ρ‹ ΠΈ сущСствСнно зависимы ΠΎΡ‚ Ρ„ΠΎΡ€ΠΌΡ‹, Ρ€Π°Π·ΠΌΠ΅Ρ€ΠΎΠ², состояния повСрхностСй Ρ‚Π΅Π», вязкости срСды.

Помимо этого Π²Ρ‹Π΄Π΅Π»ΡΡŽΡ‚ Ρ‚Ρ€Π΅Π½ΠΈΠ΅ качСния.Π’ ΠΏΠ΅Ρ€Π²ΠΎΠΌ ΠΏΡ€ΠΈΠ±Π»ΠΈΠΆΠ΅Π½ΠΈΠΈ Ρ‚Ρ€Π΅Π½ΠΈΠ΅ качСния Ρ€Π°ΡΡΡ‡ΠΈΡ‚Ρ‹Π²Π°ΡŽΡ‚, примСняя Ρ„ΠΎΡ€ΠΌΡƒΠ»Ρƒ:

Π³Π΄Π΅ k – коэффициСнт трСния качСния, ΠΊΠΎΡ‚ΠΎΡ€Ρ‹ΠΉ ΠΈΠΌΠ΅Π΅Ρ‚ Ρ€Π°Π·ΠΌΠ΅Ρ€Π½ΠΎΡΡ‚ΡŒ Π΄Π»ΠΈΠ½Ρ‹ ΠΈ зависит ΠΎΡ‚ ΠΌΠ°Ρ‚Π΅Ρ€ΠΈΠ°Π»Π° Ρ‚Π΅Π», ΠΏΠΎΠ΄Π²Π΅Ρ€ΠΆΠ΅Π½Π½Ρ‹Ρ… ΠΊΠΎΠ½Ρ‚Π°ΠΊΡ‚Ρƒ ΠΈ качСств повСрхностСй ΠΈ Ρ‚.Π΄. N – сила Π½ΠΎΡ€ΠΌΠ°Π»ΡŒΠ½ΠΎΠ³ΠΎ давлСния, r – радиус катящСгося Ρ‚Π΅Π»Π°.

Π•Π΄ΠΈΠ½ΠΈΡ†Ρ‹ измСрСния силы трСния

Основной Π΅Π΄ΠΈΠ½ΠΈΡ†Π΅ΠΉ измСрСния силы трСния (ΠΊΠ°ΠΊ ΠΈ любой Π΄Ρ€ΡƒΠ³ΠΎΠΉ силы) Π² систСмС БИ являСтся: [P]=H

Π’ Π‘Π“Π‘: [P]=Π΄ΠΈΠ½.

ΠŸΡ€ΠΈΠΌΠ΅Ρ€Ρ‹ Ρ€Π΅ΡˆΠ΅Π½ΠΈΡ Π·Π°Π΄Π°Ρ‡

ΠŸΡ€ΠΈΠΌΠ΅Ρ€

Π—Π°Π΄Π°Π½ΠΈΠ΅. На Π³ΠΎΡ€ΠΈΠ·ΠΎΠ½Ρ‚Π°Π»ΡŒΠ½ΠΎΠΌ дискС Π»Π΅ΠΆΠΈΡ‚ малСнькоС Ρ‚Π΅Π»ΠΎ. Диск вращаСтся Π²ΠΎΠΊΡ€ΡƒΠ³ оси, которая ΠΏΡ€ΠΎΡ…ΠΎΠ΄ΠΈΡ‚ Ρ‡Π΅Ρ€Π΅Π· Π΅Π³ΠΎ Ρ†Π΅Π½Ρ‚Ρ€, пСрпСндикулярно плоскости с ΡƒΠ³Π»ΠΎΠ²ΠΎΠΉ ΡΠΊΠΎΡ€ΠΎΡΡ‚ΡŒΡŽ . На ΠΊΠ°ΠΊΠΎΠΌ расстоянии ΠΎΡ‚ Ρ†Π΅Π½Ρ‚Ρ€Π° диска ΠΌΠΎΠΆΠ΅Ρ‚ Π½Π°Ρ…ΠΎΠ΄ΠΈΡ‚ΡŒΡΡ Π² состоянии равновСсия Ρ‚Π΅Π»ΠΎ, Ссли коэффициСнт трСния ΠΌΠ΅ΠΆΠ΄Ρƒ диском ΠΈ Ρ‚Π΅Π»ΠΎΠΌ Ρ€Π°Π²Π΅Π½ ?

РСшСниС. Π˜Π·ΠΎΠ±Ρ€Π°Π·ΠΈΠΌ Π½Π° рис.1 силы, ΠΊΠΎΡ‚ΠΎΡ€Ρ‹Π΅ Π±ΡƒΠ΄ΡƒΡ‚ Π΄Π΅ΠΉΡΡ‚Π²ΠΎΠ²Π°Ρ‚ΡŒ Π½Π° Ρ‚Π΅Π»ΠΎ, ΠΏΠΎΠ»ΠΎΠΆΠ΅Π½Π½ΠΎΠ΅ Π½Π° Π²Ρ€Π°Ρ‰Π°ΡŽΡ‰ΠΈΠΉΡΡ диск.

Π’ соотвСтствии со Π²Ρ‚ΠΎΡ€Ρ‹ΠΌ Π·Π°ΠΊΠΎΠ½ΠΎΠΌ ΠΡŒΡŽΡ‚ΠΎΠ½Π° ΠΈΠΌΠ΅Π΅ΠΌ:

Π’ ΠΏΡ€ΠΎΠ΅ΠΊΡ†ΠΈΠΈ Π½Π° ось YΠΈΠ· уравнСния (1.1) ΠΏΠΎΠ»ΡƒΡ‡ΠΈΠΌ:

Π’ ΠΏΡ€ΠΎΠ΅ΠΊΡ†ΠΈΠΈ Π½Π° ось X ΠΈΠΌΠ΅Π΅ΠΌ:

Π³Π΄Π΅ ускорСниС двиТСния малСнького Ρ‚Π΅Π»Π° Ρ€Π°Π²Π½ΠΎ ΠΏΠΎ ΠΌΠΎΠ΄ΡƒΠ»ΡŒ Π½ΠΎΡ€ΠΌΠ°Π»ΡŒΠ½ΠΎΠΉ ΡΠΎΡΡ‚Π°Π²Π»ΡΡŽΡ‰Π΅ΠΉ ΠΏΠΎΠ»Π½ΠΎΠ³ΠΎ ускорСния. БилутрСния покоя Π½Π°ΠΉΠ΄Π΅ΠΌ ΠΊΠ°ΠΊ:

ΠΏΡ€ΠΈΠΌΠ΅ΠΌ Π²ΠΎ Π²Π½ΠΈΠΌΠ°Π½ΠΈΠ΅ Π²Ρ‹Ρ€Π°ΠΆΠ΅Π½ΠΈΠ΅ (1.2), Ρ‚ΠΎΠ³Π΄Π° ΠΈΠΌΠ΅Π΅ΠΌ:

приравняСм ΠΏΡ€Π°Π²Ρ‹Π΅ части Π²Ρ‹Ρ€Π°ΠΆΠ΅Π½ΠΈΠΉ (1.3) ΠΈ (1.5):

Π³Π΄Π΅ малСнькоС Ρ‚Π΅Π»ΠΎ (Ρ‚Π°ΠΊ ΠΊΠ°ΠΊ ΠΎΠ½ΠΎ находится Π² состоянии покоя Π½Π° дискС) двиТСтся со ΡΠΊΠΎΡ€ΠΎΡΡ‚ΡŒΡŽ, Ρ€Π°Π²Π½ΠΎΠΉ.

ΠšΠΎΡΡ„Ρ„ΠΈΡ†ΠΈΠ΅Π½Ρ‚ трСния β€” это основная характСристика трСния ΠΊΠ°ΠΊ явлСния. Он опрСдСляСтся Π²ΠΈΠ΄ΠΎΠΌ ΠΈ состояниСм повСрхностСй трущихся Ρ‚Π΅Π».

ΠžΠŸΠ Π•Π”Π•Π›Π•ΠΠ˜Π•

ΠšΠΎΡΡ„Ρ„ΠΈΡ†ΠΈΠ΅Π½Ρ‚ΠΎΠΌ трСния Π½Π°Π·Ρ‹Π²Π°ΡŽΡ‚ коэффициСнт ΠΏΡ€ΠΎΠΏΠΎΡ€Ρ†ΠΈΠΎΠ½Π°Π»ΡŒΠ½ΠΎΡΡ‚ΠΈ, ΡΠ²ΡΠ·Ρ‹Π²Π°ΡŽΡ‰ΠΈΠΉ силу трСния () ΠΈ силу Π½ΠΎΡ€ΠΌΠ°Π»ΡŒΠ½ΠΎΠ³ΠΎ давлСния (N) Ρ‚Π΅Π»Π° Π½Π° ΠΎΠΏΠΎΡ€Ρƒ. Π§Π°Ρ‰Π΅ всСго коэффициСнт трСния ΠΎΠ±ΠΎΠ·Π½Π°Ρ‡Π°ΡŽΡ‚ Π±ΡƒΠΊΠ²ΠΎΠΉ . И Ρ‚Π°ΠΊ, коэффициСнт трСния Π²Ρ…ΠΎΠ΄ΠΈΡ‚ Π² Π·Π°ΠΊΠΎΠ½ ΠšΡƒΠ»ΠΎΠ½Π° β€” Амонтона:

Π”Π°Π½Π½Ρ‹ΠΉ коэффициСнт трСния Π½Π΅ зависит ΠΎΡ‚ ΠΏΠ»ΠΎΡ‰Π°Π΄Π΅ΠΉ, ΡΠΎΠΏΡ€ΠΈΠΊΠ°ΡΠ°ΡŽΡ‰ΠΈΡ…ΡΡ повСрхностСй.

Π’ Π΄Π°Π½Π½ΠΎΠΌ случаС Ρ€Π΅Ρ‡ΡŒ ΠΈΠ΄Π΅Ρ‚ ΠΎ коэффициСнтС трСния скольТСния, ΠΊΠΎΡ‚ΠΎΡ€Ρ‹ΠΉ зависит ΠΎΡ‚ совокупных свойств трущихся повСрхностСй ΠΈ являСтся Π±Π΅Π·Ρ€Π°Π·ΠΌΠ΅Ρ€Π½ΠΎΠΉ Π²Π΅Π»ΠΈΡ‡ΠΈΠ½ΠΎΠΉ. ΠšΠΎΡΡ„Ρ„ΠΈΡ†ΠΈΠ΅Π½Ρ‚ трСния зависит ΠΎΡ‚: качСства ΠΎΠ±Ρ€Π°Π±ΠΎΡ‚ΠΊΠΈ повСрхностСй, трущихся Ρ‚Π΅Π», присутствия Π½Π° Π½ΠΈΡ… грязи, скорости двиТСния Ρ‚Π΅Π» Π΄Ρ€ΡƒΠ³ ΠΎΡ‚Π½ΠΎΡΠΈΡ‚Π΅Π»ΡŒΠ½ΠΎ Π΄Ρ€ΡƒΠ³Π° ΠΈ Ρ‚.Π΄. ΠšΠΎΡΡ„Ρ„ΠΈΡ†ΠΈΠ΅Π½Ρ‚ трСния ΠΎΠΏΡ€Π΅Π΄Π΅Π»ΡΡŽΡ‚ эмпиричСски (ΠΎΠΏΡ‹Ρ‚Π½Ρ‹ΠΌ ΠΏΡƒΡ‚Π΅ΠΌ).

ΠšΠΎΡΡ„Ρ„ΠΈΡ†ΠΈΠ΅Π½Ρ‚ трСния, ΠΊΠΎΡ‚ΠΎΡ€Ρ‹ΠΉ соотвСтствуСт максимальной силС трСния покоя Π² Π±ΠΎΠ»ΡŒΡˆΠΈΠ½ΡΡ‚Π²Π΅ случаСв большС, Ρ‡Π΅ΠΌ коэффициСнт трСния двиТСния.

Для большСго числа ΠΏΠ°Ρ€ ΠΌΠ°Ρ‚Π΅Ρ€ΠΈΠ°Π»ΠΎΠ² Π²Π΅Π»ΠΈΡ‡ΠΈΠ½Π° коэффициСнта трСния Π½Π΅ большС Π΅Π΄ΠΈΠ½ΠΈΡ†Ρ‹ ΠΈ Π»Π΅ΠΆΠΈΡ‚ Π² ΠΏΡ€Π΅Π΄Π΅Π»Π°Ρ…

Π£Π³ΠΎΠ» трСния

Иногда вмСсто коэффициСнта трСния ΠΏΡ€ΠΈΠΌΠ΅Π½ΡΡŽΡ‚ ΡƒΠ³ΠΎΠ» трСния (), ΠΊΠΎΡ‚ΠΎΡ€Ρ‹ΠΉ связан с коэффициСнтом ΡΠΎΠΎΡ‚Π½ΠΎΡˆΠ΅Π½ΠΈΠ΅ΠΌ:

Π’Π°ΠΊ, ΡƒΠ³ΠΎΠ» трСния соотвСтствуСт ΠΌΠΈΠ½ΠΈΠΌΠ°Π»ΡŒΠ½ΠΎΠΌΡƒ ΡƒΠ³Π»Ρƒ Π½Π°ΠΊΠ»ΠΎΠ½Π° плоскости ΠΏΠΎ ΠΎΡ‚Π½ΠΎΡˆΠ΅Π½ΠΈΡŽ ΠΊ Π³ΠΎΡ€ΠΈΠ·ΠΎΠ½Ρ‚Ρƒ, ΠΏΡ€ΠΈ ΠΊΠΎΡ‚ΠΎΡ€ΠΎΠΌ Ρ‚Π΅Π»ΠΎ, Π»Π΅ΠΆΠ°Ρ‰Π΅Π΅ Π½Π° этой плоскости, Π½Π°Ρ‡Π½Π΅Ρ‚ ΡΠΊΠΎΠ»ΡŒΠ·ΠΈΡ‚ΡŒ Π²Π½ΠΈΠ· ΠΏΠΎΠ΄ воздСйствиСм силы тяТСсти. ΠŸΡ€ΠΈ этом выполняСтся равСнство:

Π˜ΡΡ‚ΠΈΠ½Π½Ρ‹ΠΉ коэффициСнт трСния

Π—Π°ΠΊΠΎΠ½ трСния, ΠΊΠΎΡ‚ΠΎΡ€Ρ‹ΠΉ ΡƒΡ‡ΠΈΡ‚Ρ‹Π²Π°Π΅Ρ‚ влияниС сил притяТСния ΠΌΠ΅ΠΆΠ΄Ρƒ ΠΌΠΎΠ»Π΅ΠΊΡƒΠ»Π°ΠΌΠΈ, трущихся повСрхностСй Π·Π°ΠΏΠΈΡΡ‹Π²Π°ΡŽ ΡΠ»Π΅Π΄ΡƒΡŽΡ‰ΠΈΠΌ ΠΎΠ±Ρ€Π°Π·ΠΎΠΌ:

Π³Π΄Π΅ β€” Π½Π°Π·Ρ‹Π²Π°ΡŽΡ‚ истинным коэффициСнтом трСния, β€” Π΄ΠΎΠ±Π°Π²ΠΎΡ‡Π½ΠΎΠ΅ Π΄Π°Π²Π»Π΅Π½ΠΈΠ΅, ΠΊΠΎΡ‚ΠΎΡ€ΠΎΠ΅ вызываСтся силами мСТмолСкулярного притяТСния, S β€” общая ΠΏΠ»ΠΎΡ‰Π°Π΄ΡŒ нСпосрСдствСнного ΠΊΠΎΠ½Ρ‚Π°ΠΊΡ‚Π° трущихся Ρ‚Π΅Π».

ΠšΠΎΡΡ„Ρ„ΠΈΡ†ΠΈΠ΅Π½Ρ‚ трСния качСния

ΠšΠΎΡΡ„Ρ„ΠΈΡ†ΠΈΠ΅Π½Ρ‚ трСния качСния (k) ΠΌΠΎΠΆΠ½ΠΎ ΠΎΠΏΡ€Π΅Π΄Π΅Π»ΠΈΡ‚ΡŒ ΠΊΠ°ΠΊ ΠΎΡ‚Π½ΠΎΡˆΠ΅Π½ΠΈΠ΅ ΠΌΠΎΠΌΠ΅Π½Ρ‚Π° силы трСния качСния () ΠΊ силС с ΠΊΠΎΡ‚ΠΎΡ€ΠΎΠΉ Ρ‚Π΅Π»ΠΎ приТимаСтся ΠΊ ΠΎΠΏΠΎΡ€Π΅ (N):

ΠžΡ‚ΠΌΠ΅Ρ‚ΠΈΠΌ, Ρ‡Ρ‚ΠΎ коэффициСнт трСния качСния ΠΎΠ±ΠΎΠ·Π½Π°Ρ‡Π°ΡŽΡ‚ Ρ‡Π°Ρ‰Π΅ Π±ΡƒΠΊΠ²ΠΎΠΉ . Π­Ρ‚ΠΎΡ‚ коэффициСнт, Π² ΠΎΡ‚Π»ΠΈΡ‡ΠΈΠ΅ ΠΎΡ‚ Π²Ρ‹ΡˆΠ΅ пСрСчислСнных коэффициСнтов трСния, ΠΈΠΌΠ΅Π΅Ρ‚ Ρ€Π°Π·ΠΌΠ΅Ρ€Π½ΠΎΡΡ‚ΡŒ Π΄Π»ΠΈΠ½Ρ‹. Π’ΠΎ Π΅ΡΡ‚ΡŒ Π² систСмС БИ ΠΎΠ½ измСряСтся Π² ΠΌΠ΅Ρ‚Ρ€Π°Ρ….

ΠšΠΎΡΡ„Ρ„ΠΈΡ†ΠΈΠ΅Π½Ρ‚ трСния качСния ΠΌΠ½ΠΎΠ³ΠΎ мСньшС, Ρ‡Π΅ΠΌ коэффициСнт трСния скольТСния.

ΠŸΡ€ΠΈΠΌΠ΅Ρ€Ρ‹ Ρ€Π΅ΡˆΠ΅Π½ΠΈΡ Π·Π°Π΄Π°Ρ‡

ΠŸΠ Π˜ΠœΠ•Π  1

Π—Π°Π΄Π°Π½ΠΈΠ΅Π’Π΅Ρ€Π΅Π²ΠΊΠ° Π»Π΅ΠΆΠΈΡ‚ частично Π½Π° столС, Ρ‡Π°ΡΡ‚ΡŒ Π΅Π΅ ΡΠ²Π΅ΡˆΠΈΠ²Π°Π΅Ρ‚ΡΡ со стола. Если Ρ‚Ρ€Π΅Ρ‚ΡŒ Π΄Π»ΠΈΠ½Ρ‹ Π²Π΅Ρ€Π΅Π²ΠΊΠΈ свСсится со стола, Ρ‚ΠΎ ΠΎΠ½Π° Π½Π°Ρ‡ΠΈΠ½Π°Π΅Ρ‚ ΡΠΊΠΎΠ»ΡŒΠ·ΠΈΡ‚ΡŒ. Каков коэффициСнт трСния Π²Π΅Ρ€Π΅Π²ΠΊΠΈ ΠΎ стол?
Π Π΅ΡˆΠ΅Π½ΠΈΠ΅Π’Π΅Ρ€Π΅Π²ΠΊΠ° ΡΠΊΠΎΠ»ΡŒΠ·ΠΈΡ‚ со стола ΠΏΠΎΠ΄ дСйствиСм силы тяТСсти. ΠžΠ±ΠΎΠ·Π½Π°Ρ‡ΠΈΠΌ силу тяТСсти, которая дСйствуСт Π½Π° Π΅Π΄ΠΈΠ½ΠΈΡ†Ρƒ Π΄Π»ΠΈΠ½Ρ‹ Π²Π΅Ρ€Π΅Π²ΠΊΠΈ ΠΊΠ°ΠΊ . Π’ Ρ‚Π°ΠΊΠΎΠΌ случаС Π² ΠΌΠΎΠΌΠ΅Π½Ρ‚ Π½Π°Ρ‡Π°Π»Π° скольТСния сила тяТСсти, которая дСйствуСт Π½Π° ΡΠ²Π΅ΡˆΠΈΠ²Π°ΡŽΡ‰ΡƒΡŽΡΡ Ρ‡Π°ΡΡ‚ΡŒ Π²Π΅Ρ€Π΅Π²ΠΊΠΈ, Ρ€Π°Π²Π½Π°:

Π”ΠΎ Π½Π°Ρ‡Π°Π»Π° скольТСния эта сила ΡƒΡ€Π°Π²Π½ΠΎΠ²Π΅ΡˆΠΈΠ²Π°Π΅Ρ‚ΡΡ силой трСния, которая дСйствуСт Π½Π° Ρ‡Π°ΡΡ‚ΡŒ Π²Π΅Ρ€Π΅Π²ΠΊΠΈ, которая Π»Π΅ΠΆΠΈΡ‚ Π½Π° столС:

Π’Π°ΠΊ ΠΊΠ°ΠΊ силы ΡƒΡ€Π°Π²Π½ΠΎΠ²Π΅ΡˆΠΈΠ²Π°ΡŽΡ‚ΡΡ, Ρ‚ΠΎ ΠΌΠΎΠΆΠ½ΠΎ Π·Π°ΠΏΠΈΡΠ°Ρ‚ΡŒ ():

ΠžΡ‚Π²Π΅Ρ‚

ΠŸΠ Π˜ΠœΠ•Π  2

Π—Π°Π΄Π°Π½ΠΈΠ΅ΠšΠ°ΠΊΠΎΠ² коэффициСнт трСния Ρ‚Π΅Π»Π° ΠΎ ΠΏΠ»ΠΎΡΠΊΠΎΡΡ‚ΡŒ (), Ссли Π·Π°Π²ΠΈΡΠΈΠΌΠΎΡΡ‚ΡŒ ΠΏΡƒΡ‚ΠΈ, ΠΊΠΎΡ‚ΠΎΡ€ΠΎΠ΅ ΠΎΠ½ΠΎ ΠΏΡ€ΠΎΡ…ΠΎΠ΄ΠΈΡ‚ Π·Π°Π΄Π°Π½ΠΎ ΡƒΡ€Π°Π²Π½Π΅Π½ΠΈΠ΅ΠΌ: Π³Π΄Π΅ ΠŸΠ»ΠΎΡΠΊΠΎΡΡ‚ΡŒ составляСт ΡƒΠ³ΠΎΠ» с Π³ΠΎΡ€ΠΈΠ·ΠΎΠ½Ρ‚ΠΎΠΌ.
Π Π΅ΡˆΠ΅Π½ΠΈΠ΅Π—Π°ΠΏΠΈΡˆΠ΅ΠΌ Π²Ρ‚ΠΎΡ€ΠΎΠΉ Π·Π°ΠΊΠΎΠ½ ΠΡŒΡŽΡ‚ΠΎΠ½Π° для сил, ΠΏΡ€ΠΈΠ»ΠΎΠΆΠ΅Π½Π½Ρ‹Ρ… ΠΊ двиТущСмуся Ρ‚Π΅Π»Ρƒ:

Π‘ΠΈΠ»Π° трСния – Π²Π΅Π»ΠΈΡ‡ΠΈΠ½Π°, с ΠΊΠΎΡ‚ΠΎΡ€ΠΎΠΉ Π²Π·Π°ΠΈΠΌΠΎΠ΄Π΅ΠΉΡΡ‚Π²ΡƒΡŽΡ‚ Π΄Π²Π΅ повСрхности ΠΏΡ€ΠΈ Π΄Π²ΠΈΠΆΠ΅Π½ΠΈΠΈ. Она зависит ΠΎΡ‚ характСристики Ρ‚Π΅Π», направлСния двиТСния. Благодаря Ρ‚Ρ€Π΅Π½ΠΈΡŽ ΡΠΊΠΎΡ€ΠΎΡΡ‚ΡŒ Ρ‚Π΅Π»Π° ΡƒΠΌΠ΅Π½ΡŒΡˆΠ°Π΅Ρ‚ΡΡ, ΠΈ вскорС ΠΎΠ½ΠΎ останавливаСтся.

Π‘ΠΈΠ»Π° трСния – направлСнная Π²Π΅Π»ΠΈΡ‡ΠΈΠ½Π°, нСзависящая ΠΎΡ‚ ΠΏΠ»ΠΎΡ‰Π°Π΄ΠΈ ΠΎΠΏΠΎΡ€Ρ‹ ΠΈ ΠΏΡ€Π΅Π΄ΠΌΠ΅Ρ‚Π°, Ρ‚Π°ΠΊ ΠΊΠ°ΠΊ ΠΏΡ€ΠΈ Π΄Π²ΠΈΠΆΠ΅Π½ΠΈΠΈ ΠΈ ΡƒΠ²Π΅Π»ΠΈΡ‡Π΅Π½ΠΈΠΈ ΠΏΠ»ΠΎΡ‰Π°Π΄ΠΈ ΠΏΠΎΠ²Ρ‹ΡˆΠ°Π΅Ρ‚ΡΡ сила Ρ€Π΅Π°ΠΊΡ†ΠΈΠΈ ΠΎΠΏΠΎΡ€Ρ‹. Π­Ρ‚Π° Π²Π΅Π»ΠΈΡ‡ΠΈΠ½Π° участвуСт Π² расчСтС силы трСния. Π’ ΠΈΡ‚ΠΎΠ³Π΅ FΡ‚Ρ€=N*m. Π—Π΄Π΅ΡΡŒ N – рСакция ΠΎΠΏΠΎΡ€Ρ‹, Π° m – коэффициСнт, ΠΊΠΎΡ‚ΠΎΡ€Ρ‹ΠΉ являСтся постоянной Π²Π΅Π»ΠΈΡ‡ΠΈΠ½ΠΎΠΉ, Ссли Π½Π΅Ρ‚ нСобходимости Π² ΠΎΡ‡Π΅Π½ΡŒ Ρ‚ΠΎΡ‡Π½Ρ‹Ρ… расчСтах. ΠŸΡ€ΠΈ ΠΏΠΎΠΌΠΎΡ‰ΠΈ этой Ρ„ΠΎΡ€ΠΌΡƒΠ»Ρ‹ ΠΌΠΎΠΆΠ½ΠΎ Π²Ρ‹Ρ‡ΠΈΡΠ»ΠΈΡ‚ΡŒ силу трСния скольТСния, ΠΊΠΎΡ‚ΠΎΡ€ΡƒΡŽ ΠΎΠ±ΡΠ·Π°Ρ‚Π΅Π»ΡŒΠ½ΠΎ стоит ΡƒΡ‡ΠΈΡ‚Ρ‹Π²Π°Ρ‚ΡŒ ΠΏΡ€ΠΈ Ρ€Π΅ΡˆΠ΅Π½ΠΈΠΈ Π·Π°Π΄Π°Ρ‡, связанных с Π΄Π²ΠΈΠΆΠ΅Π½ΠΈΠ΅ΠΌ. Если Ρ‚Π΅Π»ΠΎ вращаСтся Π½Π° повСрхности, Ρ‚ΠΎ Π² Ρ„ΠΎΡ€ΠΌΡƒΠ»Ρƒ Π½Π΅ΠΎΠ±Ρ…ΠΎΠ΄ΠΈΠΌΠΎ Π²ΠΊΠ»ΡŽΡ‡ΠΈΡ‚ΡŒ силу качСния. Π’ΠΎΠ³Π΄Π° Ρ‚Ρ€Π΅Π½ΠΈΠ΅ ΠΌΠΎΠΆΠ½ΠΎ Π½Π°ΠΉΡ‚ΠΈ ΠΏΠΎ Ρ„ΠΎΡ€ΠΌΡƒΠ»Π΅ FΡ‚Ρ€ΠΊΠ°Ρ‡ = f*N/r. Богласно Ρ„ΠΎΡ€ΠΌΡƒΠ»Π΅, ΠΏΡ€ΠΈ Π²Ρ€Π°Ρ‰Π΅Π½ΠΈΠΈ Ρ‚Π΅Π»Π° ΠΈΠΌΠ΅Π΅Ρ‚ Π·Π½Π°Ρ‡Π΅Π½ΠΈΠ΅ Π΅Π³ΠΎ радиус. Π’Π΅Π»ΠΈΡ‡ΠΈΠ½Π° f – коэффициСнт, ΠΊΠΎΡ‚ΠΎΡ€Ρ‹ΠΉ ΠΌΠΎΠΆΠ½ΠΎ Π½Π°ΠΉΡ‚ΠΈ, зная, ΠΈΠ· ΠΊΠ°ΠΊΠΎΠ³ΠΎ ΠΌΠ°Ρ‚Π΅Ρ€ΠΈΠ°Π»Π° ΠΈΠ·Π³ΠΎΡ‚ΠΎΠ²Π»Π΅Π½ΠΎ Ρ‚Π΅Π»ΠΎ ΠΈ ΠΏΠΎΠ²Π΅Ρ€Ρ…Π½ΠΎΡΡ‚ΡŒ. Π­Ρ‚ΠΎ коэффициСнт, ΠΊΠΎΡ‚ΠΎΡ€Ρ‹ΠΉ находится ΠΏΠΎ Ρ‚Π°Π±Π»ΠΈΡ†Π΅.

Π‘ΡƒΡ‰Π΅ΡΡ‚Π²ΡƒΡŽΡ‚ Ρ‚Ρ€ΠΈ силы трСния:

  • покоя;
  • скольТСния;
  • качСния.
Π’Ρ€Π΅Π½ΠΈΠ΅ покоя Π½Π΅ позволяСт Π΄Π²ΠΈΠ³Π°Ρ‚ΡŒΡΡ ΠΏΡ€Π΅Π΄ΠΌΠ΅Ρ‚Ρƒ, ΠΊ двиТСнию ΠΊΠΎΡ‚ΠΎΡ€ΠΎΠ³ΠΎ Π½Π΅ прикладываСтся усилиС. БоотвСтствСнно Π³Π²ΠΎΠ·Π΄ΠΈ, Π·Π°Π±ΠΈΡ‚Ρ‹Π΅ Π² Π΄Π΅Ρ€Π΅Π²ΡΠ½Π½ΡƒΡŽ ΠΏΠΎΠ²Π΅Ρ€Ρ…Π½ΠΎΡΡ‚ΡŒ, Π½Π΅ Π²Ρ‹ΠΏΠ°Π΄Π°ΡŽΡ‚. Π‘Π°ΠΌΠΎΠ΅ интСрСсноС, Ρ‡Ρ‚ΠΎ Ρ‡Π΅Π»ΠΎΠ²Π΅ΠΊ Ρ…ΠΎΠ΄ΠΈΡ‚ благодаря Ρ‚Ρ€Π΅Π½ΠΈΡŽ покоя, ΠΊΠΎΡ‚ΠΎΡ€ΠΎΠ΅ Π½Π°ΠΏΡ€Π°Π²Π»Π΅Π½ΠΎ Π² сторону двиТСния, это являСтся ΠΈΡΠΊΠ»ΡŽΡ‡Π΅Π½ΠΈΠ΅ΠΌ ΠΈΠ· ΠΏΡ€Π°Π²ΠΈΠ». Π’ ΠΈΠ΄Π΅Π°Π»Π΅ ΠΏΡ€ΠΈ взаимодСйствии Π΄Π²ΡƒΡ… Π°Π±ΡΠΎΠ»ΡŽΡ‚Π½ΠΎ Π³Π»Π°Π΄ΠΊΠΈΡ… повСрхностСй Π½Π΅ Π΄ΠΎΠ»ΠΆΠ½ΠΎ Π²ΠΎΠ·Π½ΠΈΠΊΠ°Ρ‚ΡŒ силы трСния. На самом Π΄Π΅Π»Π΅ Π½Π΅Π²ΠΎΠ·ΠΌΠΎΠΆΠ½ΠΎ, Ρ‡Ρ‚ΠΎΠ±Ρ‹ ΠΏΡ€Π΅Π΄ΠΌΠ΅Ρ‚ находился Π² состоянии покоя ΠΈΠ»ΠΈ двиТСния Π±Π΅Π· сопротивлСния повСрхностСй. Π’ΠΎ врСмя двиТСния Π² Тидкости Π²ΠΎΠ·Π½ΠΈΠΊΠ°Π΅Ρ‚ вязкоС сопротивлСниС. Π’ ΠΎΡ‚Π»ΠΈΡ‡ΠΈΠ΅ ΠΎΡ‚ Π²ΠΎΠ·Π΄ΡƒΡˆΠ½ΠΎΠΉ срСды, Ρ‚Π΅Π»ΠΎ Π² Тидкости Π½Π΅ ΠΌΠΎΠΆΠ΅Ρ‚ Π½Π°Ρ…ΠΎΠ΄ΠΈΡ‚ΡŒΡΡ Π² состоянии покоя. Оно ΠΏΠΎΠ΄ воздСйствиСм Π²ΠΎΠ΄Ρ‹ Π½Π°Ρ‡ΠΈΠ½Π°Π΅Ρ‚ Π΄Π²ΠΈΠΆΠ΅Π½ΠΈΠ΅, соотвСтствСнно Π² Тидкости Π½Π΅ сущСствуСт трСния покоя. Π’ΠΎ врСмя пСрСмСщСния Π² Π²ΠΎΠ΄Π΅ сопротивлСниС двиТСнию Π²ΠΎΠ·Π½ΠΈΠΊΠ°Π΅Ρ‚ благодаря Ρ€Π°Π·Π½ΠΎΠΉ скорости ΠΏΠΎΡ‚ΠΎΠΊΠΎΠ², ΠΎΠΊΡ€ΡƒΠΆΠ°ΡŽΡ‰ΠΈΡ… Ρ‚Π΅Π»ΠΎ. Π§Ρ‚ΠΎΠ±Ρ‹ ΡΠ½ΠΈΠ·ΠΈΡ‚ΡŒ сопротивлСниС ΠΏΡ€ΠΈ ΠΏΠ΅Ρ€Π΅ΠΌΠ΅Ρ‰Π΅Π½ΠΈΠΈ Π² Тидкостях, Ρ‚Π΅Π»Ρƒ ΠΏΡ€ΠΈΠ΄Π°ΡŽΡ‚ ΠΎΠ±Ρ‚Π΅ΠΊΠ°Π΅ΠΌΡƒΡŽ Ρ„ΠΎΡ€ΠΌΡƒ. Π’ ΠΏΡ€ΠΈΡ€ΠΎΠ΄Π΅ для прСодолСния сопротивлСния Π² Π²ΠΎΠ΄Π΅ Π½Π° Ρ‚Π΅Π»Π΅ Ρ€Ρ‹Π± имССтся смазка, ΡΠ½ΠΈΠΆΠ°ΡŽΡ‰Π°Ρ Ρ‚Ρ€Π΅Π½ΠΈΠ΅ ΠΏΡ€ΠΈ Π΄Π²ΠΈΠΆΠ΅Π½ΠΈΠΈ. ΠŸΠΎΠΌΠ½ΠΈΡ‚Π΅, ΠΏΡ€ΠΈ Π΄Π²ΠΈΠΆΠ΅Π½ΠΈΠΈ ΠΎΠ΄Π½ΠΎΠ³ΠΎ Ρ‚Π΅Π»Π° Π² Тидкостях Π²ΠΎΠ·Π½ΠΈΠΊΠ°Π΅Ρ‚ Ρ€Π°Π·Π½ΠΎΠ΅ Π·Π½Π°Ρ‡Π΅Π½ΠΈΠ΅ сопротивлСния.


Π§Ρ‚ΠΎΠ±Ρ‹ ΡΠ½ΠΈΠ·ΠΈΡ‚ΡŒ сопротивлСниС ΠΏΠ΅Ρ€Π΅ΠΌΠ΅Ρ‰Π΅Π½ΠΈΡŽ ΠΏΡ€Π΅Π΄ΠΌΠ΅Ρ‚ΠΎΠ² Π² Π²ΠΎΠ·Π΄ΡƒΡ…Π΅, Ρ‚Π΅Π»Π°ΠΌ ΠΏΡ€ΠΈΠ΄Π°ΡŽΡ‚ ΠΎΠ±Ρ‚Π΅ΠΊΠ°Π΅ΠΌΡƒΡŽ Ρ„ΠΎΡ€ΠΌΡƒ. ИмСнно поэтому самолСты ΠΈΠ·Π³ΠΎΡ‚Π°Π²Π»ΠΈΠ²Π°ΡŽΡ‚ ΠΈΠ· Π³Π»Π°Π΄ΠΊΠΎΠΉ стали с ΠΎΠΊΡ€ΡƒΠ³Π»Ρ‹ΠΌ корпусом, Π·Π°ΡƒΠΆΠ΅Π½Π½Ρ‹ΠΌ спСрСди. На Ρ‚Ρ€Π΅Π½ΠΈΠ΅ Π² Тидкости влияСт Π΅Π΅ Ρ‚Π΅ΠΌΠΏΠ΅Ρ€Π°Ρ‚ΡƒΡ€Π°. Для Ρ‚ΠΎΠ³ΠΎ Ρ‡Ρ‚ΠΎΠ±Ρ‹ Π°Π²Ρ‚ΠΎΠΌΠΎΠ±ΠΈΠ»ΡŒ Π²ΠΎ врСмя ΠΌΠΎΡ€ΠΎΠ·Π° Π½ΠΎΡ€ΠΌΠ°Π»ΡŒΠ½ΠΎ Π΅Π·Π΄ΠΈΠ», Π΅Π³ΠΎ Π½Π΅ΠΎΠ±Ρ…ΠΎΠ΄ΠΈΠΌΠΎ ΠΏΡ€Π΅Π΄Π²Π°Ρ€ΠΈΡ‚Π΅Π»ΡŒΠ½ΠΎ Ρ€Π°Π·ΠΎΠ³Ρ€Π΅Ρ‚ΡŒ. Π’ Ρ€Π΅Π·ΡƒΠ»ΡŒΡ‚Π°Ρ‚Π΅ этого Π²ΡΠ·ΠΊΠΎΡΡ‚ΡŒ масла ΡƒΠΌΠ΅Π½ΡŒΡˆΠ°Π΅Ρ‚ΡΡ, Ρ‡Ρ‚ΠΎ сниТаСт сопротивлСниС ΠΈ ΡƒΠΌΠ΅Π½ΡŒΡˆΠ°Π΅Ρ‚ износ Π΄Π΅Ρ‚Π°Π»Π΅ΠΉ. Π’ΠΎ врСмя пСрСмСщСния Π² Тидкости сопротивлСниС ΠΌΠΎΠΆΠ΅Ρ‚ ΡƒΠ²Π΅Π»ΠΈΡ‡ΠΈΠ²Π°Ρ‚ΡŒΡΡ ΠΈΠ·-Π·Π° возникновСния Ρ‚ΡƒΡ€Π±ΡƒΠ»Π΅Π½Ρ‚Π½Ρ‹Ρ… ΠΏΠΎΡ‚ΠΎΠΊΠΎΠ². Π’ Ρ‚Π°ΠΊΠΎΠΌ случаС Π½Π°ΠΏΡ€Π°Π²Π»Π΅Π½ΠΈΠ΅ двиТСния становится Ρ…Π°ΠΎΡ‚ΠΈΡ‡Π½Ρ‹ΠΌ. Π’ΠΎΠ³Π΄Π° Ρ„ΠΎΡ€ΠΌΡƒΠ»Π° ΠΏΡ€ΠΈΠΎΠ±Ρ€Π΅Ρ‚Π°Π΅Ρ‚ Π²ΠΈΠ΄: F=v2*k. Π—Π΄Π΅ΡΡŒ v – ΡΠΊΠΎΡ€ΠΎΡΡ‚ΡŒ, Π° k – коэффициСнт, зависящий ΠΎΡ‚ свойств Ρ‚Π΅Π»Π° ΠΈ Тидкости.


Зная физичСскиС свойства Ρ‚Π΅Π» ΠΈ ΡΠΎΠΏΡƒΡ‚ΡΡ‚Π²ΡƒΡŽΡ‰ΠΈΠ΅ силы, Π²ΠΎΠ·Π΄Π΅ΠΉΡΡ‚Π²ΡƒΡŽΡ‰ΠΈΠ΅ Π½Π° ΠΏΡ€Π΅Π΄ΠΌΠ΅Ρ‚, Π²Π°ΠΌ Π»Π΅Π³ΠΊΠΎ удастся Ρ€Π°ΡΡΡ‡ΠΈΡ‚Π°Ρ‚ΡŒ силу трСния.

Π’Ρ€Π΅Π½ΠΈΠ΅ являСтся Ρ‚Π΅ΠΌ физичСским процСссом, Π±Π΅Π· ΠΊΠΎΡ‚ΠΎΡ€ΠΎΠ³ΠΎ Π½Π΅ ΠΌΠΎΠ³Π»ΠΎ Π±Ρ‹ ΡΡƒΡ‰Π΅ΡΡ‚Π²ΠΎΠ²Π°Ρ‚ΡŒ само Π΄Π²ΠΈΠΆΠ΅Π½ΠΈΠ΅ Π² нашСм ΠΌΠΈΡ€Π΅. Π’ Ρ„ΠΈΠ·ΠΈΠΊΠ΅ для вычислСния Π°Π±ΡΠΎΠ»ΡŽΡ‚Π½ΠΎΠ³ΠΎ значСния силы трСния Π½Π΅ΠΎΠ±Ρ…ΠΎΠ΄ΠΈΠΌΠΎ Π·Π½Π°Ρ‚ΡŒ ΡΠΏΠ΅Ρ†ΠΈΠ°Π»ΡŒΠ½Ρ‹ΠΉ коэффициСнт для рассматриваСмых трущихся повСрхностСй. На этот вопрос ΠΎΡ‚Π²Π΅Ρ‚ΠΈΡ‚ данная ΡΡ‚Π°Ρ‚ΡŒΡ.

Π’Ρ€Π΅Π½ΠΈΠ΅ Π² Ρ„ΠΈΠ·ΠΈΠΊΠ΅

ΠŸΡ€Π΅ΠΆΠ΄Π΅ Ρ‡Π΅ΠΌ ΠΎΡ‚Π²Π΅Ρ‡Π°Ρ‚ΡŒ Π½Π° вопрос, ΠΊΠ°ΠΊ коэффициСнт трСния Π½Π°Ρ…ΠΎΠ΄ΠΈΡ‚ΡŒ, Π½Π΅ΠΎΠ±Ρ…ΠΎΠ΄ΠΈΠΌΠΎ Ρ€Π°ΡΡΠΌΠΎΡ‚Ρ€Π΅Ρ‚ΡŒ, Ρ‡Ρ‚ΠΎ Ρ‚Π°ΠΊΠΎΠ΅ Ρ‚Ρ€Π΅Π½ΠΈΠ΅ ΠΈ ΠΊΠ°ΠΊΠΎΠΉ силой ΠΎΠ½ΠΎ характСризуСтся.

Π’ Ρ„ΠΈΠ·ΠΈΠΊΠ΅ Π²Ρ‹Π΄Π΅Π»ΡΡŽΡ‚ Ρ‚Ρ€ΠΈ Π²ΠΈΠ΄Π° этого процСсса, Ρ‡Ρ‚ΠΎ ΠΏΡ€ΠΎΡ‚Π΅ΠΊΠ°Π΅Ρ‚ ΠΌΠ΅ΠΆΠ΄Ρƒ Ρ‚Π²Π΅Ρ€Π΄Ρ‹ΠΌΠΈ ΠΎΠ±ΡŠΠ΅ΠΊΡ‚Π°ΠΌΠΈ. Π­Ρ‚ΠΎ скольТСния ΠΈ качСния. Π’Ρ€Π΅Π½ΠΈΠ΅ покоя Π²ΠΎΠ·Π½ΠΈΠΊΠ°Π΅Ρ‚ всСгда, ΠΊΠΎΠ³Π΄Π° внСшняя сила пытаСтся ΡΠ΄Π²ΠΈΠ½ΡƒΡ‚ΡŒ с мСста ΠΎΠ±ΡŠΠ΅ΠΊΡ‚. БкольТСния Ρ‚Ρ€Π΅Π½ΠΈΠ΅, судя ΠΏΠΎ названию, Π²ΠΎΠ·Π½ΠΈΠΊΠ°Π΅Ρ‚ ΠΏΡ€ΠΈ скольТСнии ΠΎΠ΄Π½ΠΎΠΉ повСрхности ΠΏΠΎ Π΄Ρ€ΡƒΠ³ΠΎΠΉ. НаконСц, качСния трСния появляСтся, ΠΊΠΎΠ³Π΄Π° ΠΊΡ€ΡƒΠ³Π»Ρ‹ΠΉ ΠΎΠ±ΡŠΠ΅ΠΊΡ‚ (колСсо, ΡˆΠ°Ρ€ΠΈΠΊ) катится ΠΏΠΎ Π½Π΅ΠΊΠΎΡ‚ΠΎΡ€ΠΎΠΉ повСрхности.

ΠžΠ±ΡŠΠ΅Π΄ΠΈΠ½ΡΠ΅Ρ‚ всС Π²ΠΈΠ΄Ρ‹ Ρ‚ΠΎΡ‚ Ρ„Π°ΠΊΡ‚, Ρ‡Ρ‚ΠΎ ΠΎΠ½ΠΈ ΠΏΡ€Π΅ΠΏΡΡ‚ΡΡ‚Π²ΡƒΡŽΡ‚ Π»ΡŽΠ±ΠΎΠΌΡƒ двиТСнию ΠΈ Ρ‚ΠΎΡ‡ΠΊΠ° прилоТСния ΠΈΡ… сил находится Π² области ΠΊΠΎΠ½Ρ‚Π°ΠΊΡ‚Π° повСрхностСй Π΄Π²ΡƒΡ… ΠΎΠ±ΡŠΠ΅ΠΊΡ‚ΠΎΠ². Π’Π°ΠΊΠΆΠ΅ всС эти Π²ΠΈΠ΄Ρ‹ пСрСводят ΠΌΠ΅Ρ…Π°Π½ΠΈΡ‡Π΅ΡΠΊΡƒΡŽ ΡΠ½Π΅Ρ€Π³ΠΈΡŽ Π² Ρ‚Π΅ΠΏΠ»ΠΎ.

ΠŸΡ€ΠΈΡ‡ΠΈΠ½Π°ΠΌΠΈ сил трСния скольТСния ΠΈ покоя ΡΠ²Π»ΡΡŽΡ‚ΡΡ ΡˆΠ΅Ρ€ΠΎΡ…ΠΎΠ²Π°Ρ‚ΠΎΡΡ‚ΠΈ микроскопичСского ΠΌΠ°ΡΡˆΡ‚Π°Π±Π° Π½Π° повСрхностях, ΠΊΠΎΡ‚ΠΎΡ€Ρ‹Π΅ трутся. ΠšΡ€ΠΎΠΌΠ΅ Ρ‚ΠΎΠ³ΠΎ, эти Π²ΠΈΠ΄Ρ‹ обусловлСны диполь-Π΄ΠΈΠΏΠΎΠ»ΡŒΠ½Ρ‹ΠΌ ΠΈ Π΄Ρ€ΡƒΠ³ΠΈΠΌΠΈ Π²ΠΈΠ΄Π°ΠΌΠΈ взаимодСйствий ΠΌΠ΅ΠΆΠ΄Ρƒ Π°Ρ‚ΠΎΠΌΠ°ΠΌΠΈ ΠΈ ΠΌΠΎΠ»Π΅ΠΊΡƒΠ»Π°ΠΌΠΈ, ΠΊΠΎΡ‚ΠΎΡ€Ρ‹Π΅ ΠΎΠ±Ρ€Π°Π·ΡƒΡŽΡ‚ трущиСся Ρ‚Π΅Π»Π°.

ΠŸΡ€ΠΈΡ‡ΠΈΠ½Π° качСния трСния связана с гистСрСзисом ΡƒΠΏΡ€ΡƒΠ³ΠΎΠΉ Π΄Π΅Ρ„ΠΎΡ€ΠΌΠ°Ρ†ΠΈΠΈ, которая появляСтся Π² Ρ‚ΠΎΡ‡ΠΊΠ΅ ΠΊΠΎΠ½Ρ‚Π°ΠΊΡ‚Π° катящСгося ΠΎΠ±ΡŠΠ΅ΠΊΡ‚Π° ΠΈ повСрхности.

Π‘ΠΈΠ»Π° трСния ΠΈ коэффициСнт трСния

ВсС Ρ‚Ρ€ΠΈ Π²ΠΈΠ΄Π° сил Ρ‚Π²Π΅Ρ€Π΄ΠΎΠ³ΠΎ трСния ΠΎΠΏΠΈΡΡ‹Π²Π°ΡŽΡ‚ΡΡ выраТСниями, ΠΈΠΌΠ΅ΡŽΡ‰ΠΈΠΌΠΈ ΠΎΠ΄Π½Ρƒ ΠΈ Ρ‚Ρƒ ΠΆΠ΅ Ρ„ΠΎΡ€ΠΌΡƒ. ΠŸΡ€ΠΈΠ²Π΅Π΄Π΅ΠΌ Π΅Π΅:

Π—Π΄Π΅ΡΡŒ N — сила, Π΄Π΅ΠΉΡΡ‚Π²ΡƒΡŽΡ‰Π°Ρ пСрпСндикулярно повСрхности Π½Π° Ρ‚Π΅Π»ΠΎ. Она называСтся Ρ€Π΅Π°ΠΊΡ†ΠΈΠ΅ΠΉ ΠΎΠΏΠΎΡ€Ρ‹. Π’Π΅Π»ΠΈΡ‡ΠΈΠ½Π° Β΅ t — называСтся коэффициСнтом ΡΠΎΠΎΡ‚Π²Π΅Ρ‚ΡΡ‚Π²ΡƒΡŽΡ‰Π΅Π³ΠΎ Π²ΠΈΠ΄Π° трСния.

ΠšΠΎΡΡ„Ρ„ΠΈΡ†ΠΈΠ΅Π½Ρ‚Ρ‹ для трСния скольТСния ΠΈ покоя ΡΠ²Π»ΡΡŽΡ‚ΡΡ Π²Π΅Π»ΠΈΡ‡ΠΈΠ½Π°ΠΌΠΈ Π±Π΅Π·Ρ€Π°Π·ΠΌΠ΅Ρ€Π½Ρ‹ΠΌΠΈ. Π­Ρ‚ΠΎ ΠΌΠΎΠΆΠ½ΠΎ ΠΏΠΎΠ½ΡΡ‚ΡŒ, Ссли ΠΏΠΎΡΠΌΠΎΡ‚Ρ€Π΅Ρ‚ΡŒ Π½Π° равСнство силы трСния ΠΈ трСния коэффициСнта. ЛСвая Ρ‡Π°ΡΡ‚ΡŒ равСнства выраТаСтся Π² Π½ΡŒΡŽΡ‚ΠΎΠ½Π°Ρ…, правая Ρ‡Π°ΡΡ‚ΡŒ Ρ‚Π°ΠΊΠΆΠ΅ выраТаСтся Π² Π½ΡŒΡŽΡ‚ΠΎΠ½Π°Ρ…, ΠΏΠΎΡΠΊΠΎΠ»ΡŒΠΊΡƒ Π²Π΅Π»ΠΈΡ‡ΠΈΠ½Π° N — это сила.

Π§Ρ‚ΠΎ касаСтся качСния трСния, Ρ‚ΠΎ коэффициСнт для Π½Π΅Π³ΠΎ Ρ‚ΠΎΠΆΠ΅ Π±ΡƒΠ΄Π΅Ρ‚ Π²Π΅Π»ΠΈΡ‡ΠΈΠ½ΠΎΠΉ Π±Π΅Π·Ρ€Π°Π·ΠΌΠ΅Ρ€Π½ΠΎΠΉ, ΠΎΠ΄Π½Π°ΠΊΠΎ ΠΎΠ½ опрСдСляСтся Π² Π²ΠΈΠ΄Π΅ ΠΎΡ‚Π½ΠΎΡˆΠ΅Π½ΠΈΡ Π»ΠΈΠ½Π΅ΠΉΠ½ΠΎΠΉ характСристики ΡƒΠΏΡ€ΡƒΠ³ΠΎΠΉ Π΄Π΅Ρ„ΠΎΡ€ΠΌΠ°Ρ†ΠΈΠΈ ΠΊ радиусу катящСгося ΠΎΠ±ΡŠΠ΅ΠΊΡ‚Π°.

Π‘Π»Π΅Π΄ΡƒΠ΅Ρ‚ ΡΠΊΠ°Π·Π°Ρ‚ΡŒ, Ρ‡Ρ‚ΠΎ Ρ‚ΠΈΠΏΠΈΡ‡Π½Ρ‹ΠΌΠΈ значСниями коэффициСнтов трСния скольТСния ΠΈ покоя ΡΠ²Π»ΡΡŽΡ‚ΡΡ дСсятыС Π΄ΠΎΠ»ΠΈ Π΅Π΄ΠΈΠ½ΠΈΡ†Ρ‹. Для этот коэффициСнт соотвСтствуСт сотым ΠΈ тысячным долям Π΅Π΄ΠΈΠ½ΠΈΡ†Ρ‹.

Как Π½Π°Ρ…ΠΎΠ΄ΠΈΡ‚ΡŒ коэффициСнт трСния?

ΠšΠΎΡΡ„Ρ„ΠΈΡ†ΠΈΠ΅Π½Ρ‚ Β΅ t зависит ΠΎΡ‚ ряда Ρ„Π°ΠΊΡ‚ΠΎΡ€ΠΎΠ², ΠΊΠΎΡ‚ΠΎΡ€Ρ‹Π΅ слоТно ΡƒΡ‡Π΅ΡΡ‚ΡŒ матСматичСски. ΠŸΠ΅Ρ€Π΅Ρ‡ΠΈΡΠ»ΠΈΠΌ Π½Π΅ΠΊΠΎΡ‚ΠΎΡ€Ρ‹Π΅ ΠΈΠ· Π½ΠΈΡ…:

  • ΠΌΠ°Ρ‚Π΅Ρ€ΠΈΠ°Π» трущихся повСрхностСй;
  • качСство ΠΎΠ±Ρ€Π°Π±ΠΎΡ‚ΠΊΠΈ повСрхности;
  • Π½Π°Π»ΠΈΡ‡ΠΈΠ΅ Π½Π° Π½Π΅ΠΉ грязи, Π²ΠΎΠ΄Ρ‹ ΠΈ Ρ‚Π°ΠΊ Π΄Π°Π»Π΅Π΅;
  • Ρ‚Π΅ΠΌΠΏΠ΅Ρ€Π°Ρ‚ΡƒΡ€Ρ‹ повСрхностСй.

ΠŸΠΎΡΡ‚ΠΎΠΌΡƒ Ρ„ΠΎΡ€ΠΌΡƒΠ»Ρ‹ для Β΅ t Π½Π΅ сущСствуСт, ΠΈ Π΅Π³ΠΎ приходится ΠΈΠ·ΠΌΠ΅Ρ€ΡΡ‚ΡŒ ΡΠΊΡΠΏΠ΅Ρ€ΠΈΠΌΠ΅Π½Ρ‚Π°Π»ΡŒΠ½ΠΎ. Π§Ρ‚ΠΎΠ±Ρ‹ ΠΏΠΎΠ½ΡΡ‚ΡŒ, ΠΊΠ°ΠΊ коэффициСнт трСния Π½Π°Ρ…ΠΎΠ΄ΠΈΡ‚ΡŒ, слСдуСт Π΅Π³ΠΎ Π²Ρ‹Ρ€Π°Π·ΠΈΡ‚ΡŒ ΠΈΠ· Ρ„ΠΎΡ€ΠΌΡƒΠ»Ρ‹ для F t . ИмССм:

ΠŸΠΎΠ»ΡƒΡ‡Π°Π΅Ρ‚ΡΡ, Ρ‡Ρ‚ΠΎ для знания Β΅ t Π½Π΅ΠΎΠ±Ρ…ΠΎΠ΄ΠΈΠΌΠΎ Π½Π°ΠΉΡ‚ΠΈ трСния силу ΠΈ Ρ€Π΅Π°ΠΊΡ†ΠΈΡŽ ΠΎΠΏΠΎΡ€Ρ‹.

Π‘ΠΎΠΎΡ‚Π²Π΅Ρ‚ΡΡ‚Π²ΡƒΡŽΡ‰ΠΈΠΉ экспСримСнт Π²Ρ‹ΠΏΠΎΠ»Π½ΡΡŽΡ‚ ΡΠ»Π΅Π΄ΡƒΡŽΡ‰ΠΈΠΌ ΠΎΠ±Ρ€Π°Π·ΠΎΠΌ:

  1. Π‘Π΅Ρ€ΡƒΡ‚ Ρ‚Π΅Π»ΠΎ ΠΈ ΠΏΠ»ΠΎΡΠΊΠΎΡΡ‚ΡŒ, Π½Π°ΠΏΡ€ΠΈΠΌΠ΅Ρ€, ΠΈΠ·Π³ΠΎΡ‚ΠΎΠ²Π»Π΅Π½Π½Ρ‹Π΅ ΠΈΠ· Π΄Π΅Ρ€Π΅Π²Π°.
  2. Π¦Π΅ΠΏΠ»ΡΡŽΡ‚ Π΄ΠΈΠ½Π°ΠΌΠΎΠΌΠ΅Ρ‚Ρ€ ΠΊ Ρ‚Π΅Π»Ρƒ ΠΈ Ρ€Π°Π²Π½ΠΎΠΌΠ΅Ρ€Π½ΠΎ ΠΏΠ΅Ρ€Π΅ΠΌΠ΅Ρ‰Π°ΡŽΡ‚ Π΅Π³ΠΎ ΠΏΠΎ повСрхности.

ΠŸΡ€ΠΈ этом Π΄ΠΈΠ½Π°ΠΌΠΎΠΌΠ΅Ρ‚Ρ€ ΠΏΠΎΠΊΠ°Π·Ρ‹Π²Π°Π΅Ρ‚ Π½Π΅ΠΊΠΎΡ‚ΠΎΡ€ΡƒΡŽ силу, которая Ρ€Π°Π²Π½Π° F t . Ρ€Π°Π²Π½Π° вСсу Ρ‚Π΅Π»Π° Π½Π° Π³ΠΎΡ€ΠΈΠ·ΠΎΠ½Ρ‚Π°Π»ΡŒΠ½ΠΎΠΉ повСрхности.

ΠžΠΏΠΈΡΠ°Π½Π½Ρ‹ΠΉ способ позволяСт ΠΏΠΎΠ½ΡΡ‚ΡŒ, Ρ‡Π΅ΠΌΡƒ Ρ€Π°Π²Π΅Π½ коэффициСнт трСния покоя ΠΈ скольТСния. Аналогичным ΠΎΠ±Ρ€Π°Π·ΠΎΠΌ ΠΌΠΎΠΆΠ½ΠΎ ΡΠΊΡΠΏΠ΅Ρ€ΠΈΠΌΠ΅Π½Ρ‚Π°Π»ΡŒΠ½ΠΎ ΠΎΠΏΡ€Π΅Π΄Π΅Π»ΠΈΡ‚ΡŒ Β΅ t качСния.

Π”Ρ€ΡƒΠ³ΠΎΠΉ ΡΠΊΡΠΏΠ΅Ρ€ΠΈΠΌΠ΅Π½Ρ‚Π°Π»ΡŒΠ½Ρ‹ΠΉ ΠΌΠ΅Ρ‚ΠΎΠ΄ опрСдСлСния Β΅ t приводится Π² Ρ„ΠΎΡ€ΠΌΠ΅ Π·Π°Π΄Π°Ρ‡ΠΈ Π² ΡΠ»Π΅Π΄ΡƒΡŽΡ‰Π΅ΠΌ ΠΏΡƒΠ½ΠΊΡ‚Π΅.

Π—Π°Π΄Π°Ρ‡Π° Π½Π° вычислСниС Β΅t

ДСрСвянный брус находится Π½Π° стСклянной повСрхности. Наклоняя ΠΏΠ»Π°Π²Π½ΠΎ ΠΏΠΎΠ²Π΅Ρ€Ρ…Π½ΠΎΡΡ‚ΡŒ, установили, Ρ‡Ρ‚ΠΎ скольТСниС бруса начинаСтся ΠΏΡ€ΠΈ ΡƒΠ³Π»Π΅ Π½Π°ΠΊΠ»ΠΎΠ½Π° 15 o . Π§Π΅ΠΌΡƒ Ρ€Π°Π²Π΅Π½ коэффициСнт трСния покоя для ΠΏΠ°Ρ€Ρ‹ Π΄Π΅Ρ€Π΅Π²ΠΎ-стСкло?

Когда брус находился Π½Π° Π½Π°ΠΊΠ»ΠΎΠ½Π½ΠΎΠΉ плоскости ΠΏΡ€ΠΈ 15 o , Ρ‚ΠΎ покоя сила трСния для Π½Π΅Π³ΠΎ ΠΈΠΌΠ΅Π»Π° максимальноС Π·Π½Π°Ρ‡Π΅Π½ΠΈΠ΅. Она Ρ€Π°Π²Π½Π°:

Π‘ΠΈΠ»Π° N опрСдСляСтся ΠΏΠΎ Ρ„ΠΎΡ€ΠΌΡƒΠ»Π΅:

ΠŸΡ€ΠΈΠΌΠ΅Π½ΡΡ Ρ„ΠΎΡ€ΠΌΡƒΠ»Ρƒ для Β΅ t , ΠΏΠΎΠ»ΡƒΡ‡Π°Π΅ΠΌ:

Β΅ t = F t /N = m*g*sin(Ξ±)/(m*g*cos(Ξ±)) = tg(Ξ±).

ΠŸΠΎΠ΄ΡΡ‚Π°Π²Π»ΡΡ ΡƒΠ³ΠΎΠ» Ξ±, ΠΏΡ€ΠΈΡ…ΠΎΠ΄ΠΈΠΌ ΠΊ ΠΎΡ‚Π²Π΅Ρ‚Ρƒ: Β΅ t = 0,27.

Π—Π°Π΄Π°Ρ‡ΠΈ ⚠️ Π½Π° силу трСния: Ρ€Π΅ΡˆΠ΅Π½ΠΈΠ΅ Ρ‚ΠΈΠΏΠΎΠ²Ρ‹Ρ… ΠΏΡ€ΠΈΠΌΠ΅Ρ€ΠΎΠ²

Π”Π²ΠΈΠΆΠ΅Π½ΠΈΠ΅ Ρ‚Π΅Π»Π° ΠΏΠΎ повСрхности Π΄Ρ€ΡƒΠ³ΠΎΠ³ΠΎ Ρ‚Π΅Π»Π° всСгда связано с ΠΏΡ€Π΅ΠΎΠ΄ΠΎΠ»Π΅Π½ΠΈΠ΅ΠΌ силы трСния. Насколько ΠΎΠ½Π° замСдляСт ΠΏΠ΅Ρ€Π΅Π΄Π²ΠΈΠΆΠ΅Π½ΠΈΠ΅? Π’ ΠΊΠ°ΠΊΡƒΡŽ сторону Π½Π°ΠΏΡ€Π°Π²Π»Π΅Π½Π°? Зависит Π»ΠΈ ΠΎΡ‚ присутствия ΠΌΠ΅ΠΆΠ΄Ρƒ ΡΠΎΠΏΡ€ΠΈΠΊΠ°ΡΠ°ΡŽΡ‰ΠΈΠΌΠΈΡΡ повСрхностями Тидкости? Π­Ρ‚ΠΎ вопросы, Π½Π° ΠΊΠΎΡ‚ΠΎΡ€Ρ‹Π΅ ΠΎΡ‚Π²Π΅Ρ‡Π°Π΅Ρ‚ ΡΠΏΠ΅Ρ†ΠΈΠ°Π»ΡŒΠ½Ρ‹ΠΉ Ρ€Π°Π·Π΄Π΅Π» Ρ„ΠΈΠ·ΠΈΠΊΠΈ.

Π˜ΡΡ‚ΠΎΡ‡Π½ΠΈΠΊ: klevo.net

Π‘ΠΈΠ»Π° трСния β€” Ρ‡Ρ‚ΠΎ это Π·Π° ΠΏΠΎΠΊΠ°Π·Π°Ρ‚Π΅Π»ΡŒ?

БоприкосновСниС Π΄Π²ΡƒΡ… повСрхностСй Π½Π΅ΠΈΠ·ΠΌΠ΅Π½Π½ΠΎ Π²Π΅Π΄Π΅Ρ‚ ΠΊ появлСнию силы трСния. Π•Π΅ Π²Π΅Π»ΠΈΡ‡ΠΈΠ½Π° зависит ΠΎΡ‚ состояния Ρ‚Π΅Π» ΠΈ особСнностСй ΠΈΡ… двиТСния:

  • ΠΌΠ΅ΠΆΠ΄Ρƒ Π½Π΅ΠΏΠΎΠ΄Π²ΠΈΠΆΠ½Ρ‹ΠΌΠΈ Ρ‚Π΅Π»Π°ΠΌΠΈ присутствуСт Ρ‚Ρ€Π΅Π½ΠΈΠ΅ покоя;
  • ΠΏΠ΅Ρ€Π΅ΠΊΠ°Ρ‡ΠΈΠ²Π°ΡŽΡ‰ΠΈΠΌΠΈΡΡ β€” Ρ‚Ρ€Π΅Π½ΠΈΠ΅ качСния;
  • ΡΠΊΠΎΠ»ΡŒΠ·ΡΡ‰ΠΈΠΌΠΈ β€” Ρ‚Ρ€Π΅Π½ΠΈΠ΅ скольТСния;
  • Π² ΠΆΠΈΠ΄ΠΊΠΎΠΉ срСдС Ρ‚Π°ΠΊΠΎΠΉ процСсс носит Π½Π°Π·Π²Π°Π½ΠΈΠ΅ силы сопротивлСния срСды.

Π‘ΠΈΠ»Π°, появлСниС ΠΊΠΎΡ‚ΠΎΡ€ΠΎΠΉ зависит ΠΎΡ‚ соприкосновСния Π΄Π²ΡƒΡ… повСрхностСй, называСтся силой трСния.

Π”Ρ€ΡƒΠ³ΠΈΠΌΠΈ словами, ΠΊΠ°ΠΆΠ΄ΠΎΠ΅ повСрхностноС Π΄Π²ΠΈΠΆΠ΅Π½ΠΈΠ΅ Ρ‚Π΅ΠΌ слабСС, Ρ‡Π΅ΠΌ Π²Ρ‹ΡˆΠ΅ Ρ‚Ρ€Π΅Π½ΠΈΠ΅ ΡΠΎΠΏΡ€ΠΈΠΊΠ°ΡΠ°ΡŽΡ‰ΠΈΡ…ΡΡ сторон. ΠžΠ±ΡŠΡΡΠ½ΡΠ΅Ρ‚ΡΡ это Ρ‚Π΅ΠΌ, Ρ‡Ρ‚ΠΎ сила трСния всСгда Π½Π°ΠΏΡ€Π°Π²Π»Π΅Π½Π° ΠΏΡ€ΠΎΡ‚ΠΈΠ² этого двиТСния ΠΈ распространяСтся Π² плоскости, Π½Π°ΠΏΡ€Π°Π²Π»Π΅Π½Π½ΠΎΠΉ ΠΏΠΎ ΠΊΠ°ΡΠ°Ρ‚Π΅Π»ΡŒΠ½ΠΎΠΉ. Β 

Для понимания Π΄Π°Π½Π½ΠΎΠ³ΠΎ процСсса Π²Π°ΠΆΠ½ΠΎ ΠΎΠΏΠΈΡ€Π°Ρ‚ΡŒΡΡ Π½Π° прямо ΠΏΡ€ΠΎΠΏΠΎΡ€Ρ†ΠΈΠΎΠ½Π°Π»ΡŒΠ½ΡƒΡŽ Π·Π°Π²ΠΈΡΠΈΠΌΠΎΡΡ‚ΡŒ силы Π½ΠΎΡ€ΠΌΠ°Π»ΡŒΠ½ΠΎΠ³ΠΎ давлСния ΠΈ свойств ΡΠΎΠΏΡ€ΠΈΠΊΠ°ΡΠ°ΡŽΡ‰ΠΈΡ…ΡΡ повСрхностСй. Она, Π² свою ΠΎΡ‡Π΅Ρ€Π΅Π΄ΡŒ, ΠΎΠ±ΡŠΡΡΠ½ΡΠ΅Ρ‚ΡΡ сущСствованиСм элСктромагнитного поля ΠΎΠΏΡ€Π΅Π΄Π΅Π»Π΅Π½Π½ΠΎΠΉ Π²Π΅Π»ΠΈΡ‡ΠΈΠ½Ρ‹.

ЕстСствСнно, Ρ‡Ρ‚ΠΎ Ρ‚Ρ€Π΅Π½ΠΈΠ΅, Π²ΠΎΠ·Π½ΠΈΠΊΠ°ΡŽΡ‰Π΅Π΅ Π²Π½ΡƒΡ‚Ρ€ΠΈ ΠΌΠ΅Ρ…Π°Π½ΠΈΠ·ΠΌΠΎΠ², носит Π½Π°Π·Π²Π°Π½ΠΈΠ΅ Π²Π½ΡƒΡ‚Ρ€Π΅Π½Π½Π΅Π³ΠΎ, снаруТи β€” внСшнСго. Π’Π°ΠΊ, Ссли Ρ€Π°Π±ΠΎΡ‚Π°ΡŽΡ‰ΠΈΠΉ ΠΏΡ€ΠΈΠ±ΠΎΡ€ Π½Π΅ двиТСтся Π² пространствС, Π² Π½Π΅ΠΌ Π²ΠΎΠ·Π½ΠΈΠΊΠ°ΡŽΡ‚ Π²Π½ΡƒΡ‚Ρ€Π΅Π½Π½ΠΈΠ΅ cΠΈΠ»Ρ‹ трСния. Если ΠΎΠ½ пСрСмСщаСтся ΠΎΡ‚Π½ΠΎΡΠΈΡ‚Π΅Π»ΡŒΠ½ΠΎ Π΄Ρ€ΡƒΠ³ΠΈΡ… Ρ‚Π΅Π», ΠΎΠ½ Π΄ΠΎΠ»ΠΆΠ΅Π½ ΠΏΡ€Π΅ΠΎΠ΄ΠΎΠ»Π΅Π²Π°Ρ‚ΡŒ внСшнюю силу трСния.

Π˜ΡΡ‚ΠΎΡ‡Π½ΠΈΠΊ:Β 900igr.net

ДСйствиС силы трСния ΠΌΠΎΠΆΠ½ΠΎ Π½Π°Π±Π»ΡŽΠ΄Π°Ρ‚ΡŒ Π½Π° ΠΏΡ€ΠΈΠΌΠ΅Ρ€Π΅:

Π’Π΅Π»ΠΎ Π½Π° Π³ΠΎΡ€ΠΈΠ·ΠΎΠ½Ρ‚Π°Π»ΡŒΠ½ΠΎΠΉ повСрхности, ΠΏΡ€ΠΈ отсутствии воздСйствия Π½Π° Π½Π΅Π³ΠΎ посторонних сил, Π»Π΅ΠΆΠΈΡ‚ Π½Π΅ΠΏΠΎΠ΄Π²ΠΈΠΆΠ½ΠΎ. Начиная ΠΏΡ€ΠΈΠΌΠ΅Π½ΡΡ‚ΡŒ Π½Π΅ΠΊΡƒΡŽ силу двиТСния Fдв происходит ΠΏΠΎΠΏΡ‹Ρ‚ΠΊΠ° ΡΠ΄Π²ΠΈΠ½ΡƒΡ‚ΡŒ Π΅Π³ΠΎ с мСста.

Π˜ΡΡ‚ΠΎΡ‡Π½ΠΈΠΊ:Β infourok.ru

Π‘Π½Π°Ρ‡Π°Π»Π° это Π½Π΅ удаСтся ΠΈΠ·-Π·Π° Ρ‚ΠΎΠ³ΠΎ, Ρ‡Ρ‚ΠΎ FΡ‚Ρ€ ΠΏΡ€Π΅Π²Ρ‹ΡˆΠ°Π΅Ρ‚ Π²Π΅Π»ΠΈΡ‡ΠΈΠ½Ρƒ внСшнСй силы. УвСличивая модСль послСднСй, Π΄ΠΎΠ±ΠΈΠ²Π°ΡŽΡ‚ΡΡ ΡƒΡ€Π°Π²Π½ΠΎΠ²Π΅ΡˆΠΈΠ²Π°Π½ΠΈΡ, Π° Π·Π°Ρ‚Π΅ΠΌ β€” ΠΏΡ€Π΅Π²Ρ‹ΡˆΠ΅Π½ΠΈΠ΅ силы двиТСния. Π’ Π΄Π°Π½Π½ΠΎΠΌ случаС, сила трСния β€” это сила покоя.

Π”Π°ΠΆΠ΅ максимальноС Ρ‚Ρ€Π΅Π½ΠΈΠ΅ Π½Π΅ опрСдСляСтся ΠΏΠ»ΠΎΡ‰Π°Π΄ΡŒΡŽ ΡΠΎΠΏΡ€ΠΈΠΊΠ°ΡΠ°ΡŽΡ‰ΠΈΡ…ΡΡ повСрхностСй Ρ‚Π΅Π», Π½ΠΎ зависит ΠΎΡ‚ силы \(N\) (Π½ΠΎΡ€ΠΌΠ°Π»ΡŒΠ½ΠΎΠ΅ Π΄Π°Π²Π»Π΅Π½ΠΈΠ΅) ΠΈ коэффициСнта трСния покоя \(\mu0\).

\(FΡ‚Ρ€\;ΠΏΠΎΠΊ=\mu0N\)

ΠŸΡ€ΠΎΠ΄ΠΎΠ»ΠΆΠ°Ρ ΡƒΠ²Π΅Π»ΠΈΡ‡ΠΈΠ²Π°Ρ‚ΡŒ Π΄Π°Π²Π»Π΅Π½ΠΈΠ΅, Π΄ΠΎΠ±ΠΈΠ²Π°ΡŽΡ‚ΡΡ Ρ‚ΠΎΠ³ΠΎ, Ρ‡Ρ‚ΠΎ Ρ‚Π΅Π»ΠΎ Π½Π°Ρ‡ΠΈΠ½Π°Π΅Ρ‚ ΡΠΊΠΎΠ»ΡŒΠ·ΠΈΡ‚ΡŒ. Π’Π΅ΠΏΠ΅Ρ€ΡŒ ΠΏΡ€ΠΈ Π΅Π³ΠΎ Π΄Π²ΠΈΠΆΠ΅Π½ΠΈΠΈ дСйствуСт сила трСния скольТСния, Π½Π° ΠΏΡ€Π΅ΠΎΠ΄ΠΎΠ»Π΅Π½ΠΈΠ΅ ΠΊΠΎΡ‚ΠΎΡ€ΠΎΠΉ Ρ‚Π°ΠΊΠΆΠ΅ Π΄ΠΎΠ»ΠΆΠ½ΠΎ Ρ…Π²Π°Ρ‚Π°Ρ‚ΡŒ значСния внСшнСй силы.

Если рассматриваСмый ΠΏΡ€Π΅Π΄ΠΌΠ΅Ρ‚ ΠΊΡ€ΡƒΠ³Π»ΠΎΠΉ Ρ„ΠΎΡ€ΠΌΡ‹, Π΅Π³ΠΎ Π΄Π²ΠΈΠΆΠ΅Π½ΠΈΠ΅ сопровоТдаСтся силой трСния качСния. ΠšΠΎΡΡ„Ρ„ΠΈΡ†ΠΈΠ΅Π½Ρ‚ трСния ΠΏΡ€ΠΈ этом Π³ΠΎΡ€Π°Π·Π΄ΠΎ мСньшС, хотя особСнности процСсса ΠΈΠ΄Π΅Π½Ρ‚ΠΈΡ‡Π½Ρ‹.Β 

Π’Π΅Π»ΠΎ, находящССся ΠΏΠΎ повСрхности ΠΏΠΎΠ΄ Π½Π°ΠΊΠ»ΠΎΠ½ΠΎΠΌ, испытываСт Π½Π° сСбС воздСйствиС Π΄ΠΎΠΏΠΎΠ»Π½ΠΈΡ‚Π΅Π»ΡŒΠ½ΠΎΠΉ силы β€” силы ΠΎΠΏΠΎΡ€Ρ‹.

ΠŸΠΎΠ½ΡΡ‚ΠΈΠ΅ ΠΈ ΠΎΠΏΡ€Π΅Π΄Π΅Π»Π΅Π½ΠΈΠ΅, Π² ΠΊΠ°ΠΊΠΈΡ… Π΅Π΄ΠΈΠ½ΠΈΡ†Π°Ρ… измСряСтся

ΠšΠ»Π°ΡΡΠΈΡ‡Π΅ΡΠΊΠΎΠΉ Ρ„ΠΎΡ€ΠΌΡƒΠ»ΠΎΠΉ для опрСдСлСния FΡ‚Ρ€ ΠΏΡ€Π΅Π΄ΠΌΠ΅Ρ‚Π°, Π»Π΅ΠΆΠ°Ρ‰Π΅Π³ΠΎ Π½Π° Π³ΠΎΡ€ΠΈΠ·ΠΎΠ½Ρ‚Π°Π»ΡŒΠ½ΠΎΠΉ ΠΎΠΏΠΎΡ€Π΅, являСтся:

\(F=\;k\ast N\)

Π³Π΄Π΅ \(k\) β€” коэффициСнт трСния. Π­Ρ‚ΠΎ постоянная Π²Π΅Π»ΠΈΡ‡ΠΈΠ½Π°, которая отраТаСтся Π² ΡΠΏΠ΅Ρ†ΠΈΠ°Π»ΡŒΠ½Ρ‹Ρ… тСхничСских Ρ‚Π°Π±Π»ΠΈΡ†Π°Ρ… ΠΈ зависит ΠΎΡ‚ ΠΏΡ€ΠΈΡ€ΠΎΠ΄Ρ‹ вСщСства.

\(N\) β€” рСакция ΠΎΠΏΠΎΡ€Ρ‹.

KоэффициСнт k ΠΌΠΎΠΆΠ΅Ρ‚ Π²ΡΡ‚Ρ€Π΅Ρ‡Π°Ρ‚ΡŒΡΡ Π² Π²ΠΈΠ΄Π΅ Π±ΡƒΠΊΠ²Ρ‹ \mu.

Помимо Π½Π΅Π³ΠΎ, Π²Π°ΠΆΠ½ΠΎ ΠΏΡ€Π°Π²ΠΈΠ»ΡŒΠ½ΠΎ ΠΎΠΏΡ€Π΅Π΄Π΅Π»ΠΈΡ‚ΡŒ Ρ€Π΅Π°ΠΊΡ†ΠΈΡŽ ΠΎΠΏΠΎΡ€Ρ‹. Она высчитываСтся ΠΏΠΎ Ρ„ΠΎΡ€ΠΌΡƒΠ»Π΅: \(N=m\ast g,\) Π³Π΄Π΅ \(m\) β€” извСстная масса Ρ‚Π΅Π»Π°, g β€” ΠΏΠΎΠΊΠ°Π·Π°Ρ‚Π΅Π»ΡŒ свободного падСния, Ρ€Π°Π²Π½Ρ‹ΠΉ 9,8ΠΌ/с2.

ΠŸΡ€Π΅Π΄ΠΌΠ΅Ρ‚, ΡΠΎΠ²Π΅Ρ€ΡˆΠ°ΡŽΡ‰ΠΈΠΉ Π΄Π²ΠΈΠΆΠ΅Π½ΠΈΠ΅ ΠΏΠΎ Π½Π°ΠΊΠ»ΠΎΠ½Π½ΠΎΠΉ повСрхности, испытываСт Π½Π° сСбС воздСйствиС Π½Π΅ΡΠΊΠΎΠ»ΡŒΠΊΠΈΡ… сил. ΠŸΠΎΡΡ‚ΠΎΠΌΡƒ Ρ„ΠΎΡ€ΠΌΡƒΠ»Π° для Π΅Π³ΠΎ FΡ‚Ρ€ ΠΏΡ€ΠΈΠ½ΠΈΠΌΠ°Π΅Ρ‚ Π²ΠΈΠ΄:

\(FΡ‚Ρ€=k\ast m\ast g\ast\cos\alpha\)

Π’ Ρ„ΠΎΡ€ΠΌΡƒΠ»Π΅ ΠΈΡΠΏΠΎΠ»ΡŒΠ·ΡƒΠ΅Ρ‚ΡΡ гравитационная постоянная g. Π•Π΅ Π²Π΅Π»ΠΈΡ‡ΠΈΠ½Π° Ρ€Π°Π²Π½Π° 9,8 ΠΌ/с2.Β 

Π˜ΡΡ‚ΠΎΡ‡Π½ΠΈΠΊ:Β zen.yandex.ru

Для измСрСния силы трСния Π² БИ сущСствуСт Π΅Π΄ΠΈΠ½ΠΈΡ†Π° Н (ΠΡŒΡŽΡ‚ΠΎΠ½). Π’ систСмС CΠ“Π‘ ΠΎΠ½Π° измСряСтся Π² Π΄ΠΈΠ½Π°Ρ… (Π΄ΠΈΠ½).

Π’Ρ‹Ρ€Π°Π·ΠΈΡ‚ΡŒ смысл Π΅Π΄ΠΈΠ½ΠΈΡ†Ρ‹ ΠΡŒΡŽΡ‚ΠΎΠ½ ΠΌΠΎΠΆΠ½ΠΎ Ρ„ΠΎΡ€ΠΌΡƒΠ»ΠΎΠΉ:

\(H=кг\ast м/с2\)

Π—Π°Π΄Π°Ρ‡ΠΈ Π½Π° силу трСния, Ρ€Π΅ΡˆΠ΅Π½ΠΈΠ΅ Ρ‚ΠΈΠΏΠΎΠ²Ρ‹Ρ… ΠΏΡ€ΠΈΠΌΠ΅Ρ€ΠΎΠ²

Задания ΠΏΠΎ Ρ‚Π΅ΠΌΠ΅ Β«Π‘ΠΈΠ»Π° трСния» ΠΌΠΎΠ³ΡƒΡ‚ ΠΈΠΌΠ΅Ρ‚ΡŒΒ  Ρ€Π°Π·Π½Ρ‹Π΅ направлСния:

  1. На ΠΎΠΏΡ€Π΅Π΄Π΅Π»Π΅Π½ΠΈΠ΅ силы трСния.
  2. На ΠΎΠΏΡ€Π΅Π΄Π΅Π»Π΅Π½ΠΈΠ΅ коэффициСнта трСния.
  3. На ΠΎΠΏΡ€Π΅Π΄Π΅Π»Π΅Π½ΠΈΠ΅ силы трСния покоя.
  4. На ΠΎΠΏΡ€Π΅Π΄Π΅Π»Π΅Π½ΠΈΠ΅ силы трСния скольТСния.
  5. На ΠΎΠΏΡ€Π΅Π΄Π΅Π»Π΅Π½ΠΈΠ΅ коэффициСнта трСния скольТСния.

ΠŸΡ€ΠΈΠΌΠ΅Ρ€ β„–1

Масса Ρ‚Π΅Π»Π°, находящСгося Π½Π° столС, составляСт \(5 ΠΊΠ³. Β΅=0,2\). К Ρ‚Π΅Π»Ρƒ ΠΏΡ€ΠΈΠ»Π°Π³Π°ΡŽΡ‚ внСшнюю силу, Ρ€Π°Π²Π½ΡƒΡŽ \(2,5Н\). Какая сила трСния ΠΏΡ€ΠΈ этом Π²ΠΎΠ·Π½ΠΈΠΊΠ°Π΅Ρ‚ (ΠΏΠΎ ΠΌΠΎΠ΄ΡƒΠ»ΡŽ)?

РСшСниС: ΠΏΠΎ Ρ„ΠΎΡ€ΠΌΡƒΠ»Π΅ для максимальной силы трСния \(Fмакс\;Ρ‚Ρ€=\mu mg=0,2\ast5\ast10=10Н\)

Π’Π½Π΅ΡˆΠ½ΡΡ сила ΠΏΠΎ ΡƒΡΠ»ΠΎΠ²ΠΈΡŽ Π·Π°Π΄Π°Ρ‡ΠΈ мСньшС, максимальной, поэтому Ρ‚Π΅Π»ΠΎ находится Π² ΠΏΠΎΠΊΠΎΠ΅. FΡ‚Ρ€ ΡƒΡ€Π°Π²Π½ΠΎΠ²Π΅ΡˆΠΈΠ²Π°Π΅Ρ‚ внСшнюю силу. Π‘Π»Π΅Π΄ΠΎΠ²Π°Ρ‚Π΅Π»ΡŒΠ½ΠΎ, ΠΎΠ½Π° равняСтся \(2,5Н.\)

ΠŸΡ€ΠΈΠΌΠ΅Ρ€ β„–2

Брусок ΠΈΠ· ΠΌΠ΅Ρ‚Π°Π»Π»Π° вСсит 4 ΠΊΠ³ ΠΈ Π»Π΅ΠΆΠΈΡ‚ Π½Π° Π³ΠΎΡ€ΠΈΠ·ΠΎΠ½Ρ‚Π°Π»ΡŒΠ½ΠΎΠΉ повСрхности. Π˜Π·Π²Π΅ΡΡ‚Π½ΠΎ, Ρ‡Ρ‚ΠΎ ΠΏΠΎΠ΄Π²ΠΈΠ½ΡƒΡ‚ΡŒ Π΅Π³ΠΎ ΠΌΠΎΠΆΠ½ΠΎ, ΠΏΡ€ΠΈΠ»ΠΎΠΆΠΈΠ² силу 20 Н, ΠΈΠΌΠ΅ΡŽΡ‰ΡƒΡŽ Π³ΠΎΡ€ΠΈΠ·ΠΎΠ½Ρ‚Π°Π»ΡŒΠ½ΠΎΠ΅ Π½Π°ΠΏΡ€Π°Π²Π»Π΅Π½ΠΈΠ΅. Если Π½Π° эту ΠΆΠ΅ ΠΏΠΎΠ²Π΅Ρ€Ρ…Π½ΠΎΡΡ‚ΡŒ ΠΏΠΎΠ»ΠΎΠΆΠΈΡ‚ΡŒ ΠΏΡ€Π΅Π΄ΠΌΠ΅Ρ‚ ΠΈΠ· пластика с массой 2 ΠΊΠ³, нСобходимая сила Π·Π½Π°Ρ‡ΠΈΡ‚Π΅Π»ΡŒΠ½ΠΎ измСнится. Какой Π²Π΅Π»ΠΈΡ‡ΠΈΠ½Π΅ ΠΎΠ½Π° Π±ΡƒΠ΄Π΅Ρ‚ Ρ€Π°Π²Π½Π°, Ссли коэффициСнт трСния пластикового ΠΏΡ€Π΅Π΄ΠΌΠ΅Ρ‚Π° Π² 2 Ρ€Π°Π·Π° мСньшС мСталличСского.

РСшСниС:

На брусок ΠΈΠ· ΠΌΠ΅Ρ‚Π°Π»Π»Π° дСйствуСт сила согласно Ρ„ΠΎΡ€ΠΌΡƒΠ»Π΅ \(F1=m1\ast g\ast\;Β΅1\), Π½Π° пластиковый β€” \(F2=m2\ast g\ast\;Β΅2=Β΅1/2m2\ast g\).

Π’ Π½Π°Ρ‡Π°Π»Π΅ дСйствия \(F=FΡ‚Ρ€\).

Π€ΠΎΡ€ΠΌΡƒΠ»Π°, ΠΏΠΎΠ·Π²ΠΎΠ»ΡΡŽΡ‰Π°Ρ Ρ€Π΅ΡˆΠΈΡ‚ΡŒ Π·Π°Π΄Π°Ρ‡Ρƒ, ΠΈΠΌΠ΅Π΅Ρ‚ ΡΠ»Π΅Π΄ΡƒΡŽΡ‰ΠΈΠΉ Π²ΠΈΠ΄: \(F2=F1/2\ast m2/m1=1/2\ast20\ast2/4=5Н\).

ΠŸΡ€ΠΈΠΌΠ΅Ρ€ β„–3

Π‘Π°Π½ΠΊΠΈ вСсят 5 ΠΊΠ³. ΠŸΡ€ΠΈ скольТСнии ΠΏΠΎ Π³ΠΎΡ€ΠΈΠ·ΠΎΠ½Ρ‚Π°Π»ΡŒΠ½ΠΎΠΉ повСрхности Π½Π° полозья дСйствуСт сила трСния 6 Н. ΠžΠΏΡ€Π΅Π΄Π΅Π»ΠΈΡ‚ΡŒ коэффициСнт трСния, Ссли ускорСниС свободного падСния Π² Π΄Π°Π½Π½ΠΎΠΉ ситуации Ρ€Π°Π²Π½ΠΎ 10 ΠΌ/с2.

РСшСниС: ΠΏΡ€ΠΈ скольТСнии полозьСв санок ΠΏΠΎ повСрхности сила трСния скольТСния обуславливаСтся силой Ρ€Π΅Π°ΠΊΡ†ΠΈΠΈ ΠΎΠΏΠΎΡ€Ρ‹, Π° Ρ‚Π°ΠΊΠΆΠ΅ коэффициСнтом Β΅. Π€ΠΎΡ€ΠΌΡƒΠ»Π° ΠΈΠΌΠ΅Π΅Ρ‚ ΡΠ»Π΅Π΄ΡƒΡŽΡ‰ΠΈΠΉ Π²ΠΈΠ΄: \(F=\;Β΅N\). Π‘ Π΄Ρ€ΡƒΠ³ΠΎΠΉ стороны, Π²Ρ‚ΠΎΡ€ΠΎΠΉ Π·Π°ΠΊΠΎΠ½ ΠΡŒΡŽΡ‚ΠΎΠ½Π° Π΄ΠΈΠΊΡ‚ΡƒΠ΅Ρ‚, Ρ‡Ρ‚ΠΎ \(N=mg\). ΠžΡ‚ΡΡŽΠ΄Π° Π²Ρ‹Ρ‚Π΅ΠΊΠ°Π΅Ρ‚, Ρ‡Ρ‚ΠΎ \(Β΅=F/mg=6H/5ΠΊΠ³\ast10ΠΌ/с2=0,12\).

ΠŸΡ€ΠΈΠΌΠ΅Ρ€ β„–4

Π’Π΅Π»ΠΎ ΠΈΠΌΠ΅Π΅Ρ‚ массу 5 ΠΊΠ³. Оно ΡΠΎΠ²Π΅Ρ€ΡˆΠ°Π΅Ρ‚ Π΄Π²ΠΈΠΆΠ΅Π½ΠΈΠ΅ Π² Π³ΠΎΡ€ΠΈΠ·ΠΎΠ½Ρ‚Π°Π»ΡŒΠ½ΠΎΠΉ плоскости. ΠŸΡ€ΠΈ этом сила трСния составляСт 10 Н. ΠžΠΏΡ€Π΅Π΄Π΅Π»ΠΈΡ‚ΡŒ Π²Π΅Π»ΠΈΡ‡ΠΈΠ½Ρƒ силы трСния скольТСния ΠΏΡ€ΠΈ условии, Ρ‡Ρ‚ΠΎ масса ΡƒΠΌΠ΅Π½ΡŒΡˆΠΈΡ‚ΡΡ Π½Π° 2 ΠΊΠ³, Π° коэффициСнт останСтся Π±Π΅Π· ΠΈΠ·ΠΌΠ΅Π½Π΅Π½ΠΈΠΉ.

РСшСниС: сила трСния ΠΈΠΌΠ΅Π΅Ρ‚ Ρ„ΠΎΡ€ΠΌΡƒΠ»Ρƒ \(F=\;Β΅\ast N\). Если Ρ‚Π΅Π»ΠΎ двиТСтся Π³ΠΎΡ€ΠΈΠ·ΠΎΠ½Ρ‚Π°Π»ΡŒΠ½ΠΎ ΠΏΠΎ ΠΎΠΏΠΎΡ€Π΅, согласно Π²Ρ‚ΠΎΡ€ΠΎΠΌΡƒ Π·Π°ΠΊΠΎΠ½Ρƒ ΠΡŒΡŽΡ‚ΠΎΠ½Π°, Π΅Π³ΠΎ \(N\) равняСтся ΠΏΡ€ΠΎΠΈΠ·Π²Π΅Π΄Π΅Π½ΠΈΡŽ \(m\ast g\).

Π˜ΡΡ…ΠΎΠ΄Ρ ΠΈΠ· этого, \(FΡ‚Ρ€\)Β Π±ΡƒΠ΄Π΅Ρ‚ ΠΏΡ€ΠΎΠΏΠΎΡ€Ρ†ΠΈΠΎΠ½Π°Π»ΡŒΠ½Π° массС, ΡƒΠΌΠ½ΠΎΠΆΠ΅Π½Π½ΠΎΠΉ Π½Π° \(Β΅\). ΠŸΡ€ΠΈ Π½Π΅ΠΈΠ·ΠΌΠ΅Π½Π½ΠΎΠΌ коэффициСнтС трСния ΡƒΠΌΠ΅Π½ΡŒΡˆΠ΅Π½ΠΈΠ΅ массы Ρ‚Π΅Π»Π° Π² 2 Ρ€Π°Π·Π° ΠΏΡ€ΠΈΠ²Π΅Π΄Π΅Ρ‚ ΠΊ ΡƒΠΌΠ΅Π½ΡŒΡˆΠ΅Π½ΠΈΡŽ силы трСния скольТСния Ρ‚Π°ΠΊΠΆΠ΅ Π² 2 Ρ€Π°Π·Π°. ΠŸΠΎΡΡ‚ΠΎΠΌΡƒ:

\(10H/2=5H.\)

ΠŸΡ€ΠΈΠΌΠ΅Ρ€ β„–5

Π’Π΅Π»ΠΎ, двиТущССся ΠΏΠΎ Ρ€ΠΎΠ²Π½ΠΎΠΉ Π³ΠΎΡ€ΠΈΠ·ΠΎΠ½Ρ‚Π°Π»ΡŒΠ½ΠΎΠΉ плоскости, Π΄Π°Π²ΠΈΡ‚ Π½Π° Π½Π΅Π΅ с силой 20 Н. Π‘ΠΈΠ»Π° трСния ΠΏΡ€ΠΈ этом составляСт 5 Н. ΠžΠΏΡ€Π΅Π΄Π΅Π»ΠΈΡ‚ΡŒ Π²Π΅Π»ΠΈΡ‡ΠΈΠ½Ρƒ коэффициСнта трСния скольТСния.

РСшСниС: ΠŸΠΎΡΠΊΠΎΠ»ΡŒΠΊΡƒ \(F=\;Β΅\ast P,Β΅=\;FΡ‚Ρ€/P\). ΠŸΠΎΠ΄ΡΡ‚Π°Π²Π»ΡΡ значСния, ΠΏΠΎΠ»ΡƒΡ‡Π°Π΅ΠΌ расчСт: \(5Н/20Н=0,25.\)

ΠžΡ‚Π²Π΅Ρ‚: \(Β΅=0,25\).

ΠŸΠΎΠ»ΡƒΡ‡ΠΈΡ‚ΡŒ знания ΠΈΠ»ΠΈ ΠΏΠΎΠ΄Π³ΠΎΡ‚ΠΎΠ²ΠΈΡ‚ΡŒ ΠΊΠΎΠ½Ρ‚Ρ€ΠΎΠ»ΡŒΠ½ΡƒΡŽ Ρ€Π°Π±ΠΎΡ‚Ρƒ ΠΏΠΎ Ρ‚Π΅ΠΌΠ΅ Β«Π‘ΠΈΠ»Π° трСния» ΠΌΠΎΠΆΠ½ΠΎ быстро ΠΈ Π³Ρ€Π°ΠΌΠΎΡ‚Π½ΠΎ, Ссли ΠΎΠ±Ρ€Π°Ρ‚ΠΈΡ‚ΡŒΡΡ Π·Π° ΠΏΠΎΠΌΠΎΡ‰ΡŒΡŽ Π½Π° ЀСникс.Π₯Π΅Π»ΠΏ.Β 

Π‘ΠΈΠ»Π° трСния


Β 

МнС, ΠΊΠ°ΠΊ ΡƒΡ‡ΠΈΡ‚Π΅Π»ΡŽ, всСгда нСпросто давались ΡƒΡ€ΠΎΠΊΠΈ ΠΏΠΎ Ρ‚Π΅ΠΌΠ΅ Β«Π‘ΠΈΠ»Π° трСния». Π‘Π»ΠΎΠΆΠ½ΠΎΡΡ‚ΡŒ Π½Π° ΡƒΡ€ΠΎΠΊΠ°Ρ… Π·Π°ΠΊΠ»ΡŽΡ‡Π°Π»Π°ΡΡŒ Π² ΠΏΡ€ΠΎΠ²Π΅Π΄Π΅Π½ΠΈΠΈ Π°Π½Π°Π»ΠΈΠ·Π° условий ΠΈ Π²Ρ‹Π±ΠΎΡ€Π΅ Π² Ρ€Π΅ΡˆΠ΅Π½ΠΈΠΈ Π·Π°Π΄Π°Ρ‡ ΠΌΠ΅ΠΆΠ΄Ρƒ силой трСния скольТСния ΠΈ силой трСния покоя.

Поиск ΠΏΡ€ΠΈΠ²Ρ‘Π» ΠΊ ΡΠ»Π΅Π΄ΡƒΡŽΡ‰Π΅ΠΌΡƒ исслСдованию, ΠΊΠΎΡ‚ΠΎΡ€ΠΎΠ΅ ΠΌΡ‹ с ΡƒΡ‡Π΅Π½ΠΈΠΊΠ°ΠΌΠΈ ΠΏΡ€ΠΎΠ²ΠΎΠ΄ΠΈΠΌ Π½Π° ΡƒΡ€ΠΎΠΊΠ΅ Β«Π‘ΠΈΠ»Π° трСния».Β  ВрСмя исслСдования – 7-10 ΠΌΠΈΠ½ΡƒΡ‚.

1.Β Β Β Β  РассматриваСтся Ρ‚Π΅Π»ΠΎ извСстной массы (m=10 ΠΊΠ³) Π½Π° Π³ΠΎΡ€ΠΈΠ·ΠΎΠ½Ρ‚Π°Π»ΡŒΠ½ΠΎΠΉ повСрхности, ΠΊ ΠΊΠΎΡ‚ΠΎΡ€ΠΎΠΌΡƒ ΠΏΡ€ΠΈΠ»ΠΎΠΆΠ΅Π½Π° Π³ΠΎΡ€ΠΈΠ·ΠΎΠ½Ρ‚Π°Π»ΡŒΠ½Π°Ρ сила. Π˜Π·Π²Π΅ΡΡ‚Π΅Π½ коэффициСнт трСния ΠΌΠ΅ΠΆΠ΄Ρƒ Ρ‚Π΅Π»ΠΎΠΌ ΠΈ ΠΏΠΎΠ²Π΅Ρ€Ρ…Π½ΠΎΡΡ‚ΡŒΡŽ (Β΅= 0,1).

2.Β Β Β Β  На доску проСцируСтся ΠΈΒ  Π² тСтрадях выполняСтся  рисунок ΠΈΠ· ΠΌΠ°Ρ‚Π΅Ρ€ΠΈΠ°Π»ΠΎΠ² Ρ€Π°Π·Π΄Π΅Π»Π° рСсурса «Готовимся ΠΊ ΡƒΡ€ΠΎΠΊΡƒ. Π”ΠΈΠ½Π°ΠΌΠΈΠΊΠ°Β»:

Β 

3.Β Β Β Β  По Ρ„ΠΎΡ€ΠΌΡƒΠ»Π΅ опрСдСляСтся Π·Π½Π°Ρ‡Π΅Π½ΠΈΠ΅ силы трСния скольТСния:

FΡ‚Ρ€ с =Β  Β΅N, Β N=mg (Π² Π΄Π°Π½Π½ΠΎΠΉ Π·Π°Π΄Π°Ρ‡Π΅), Ρ‚ΠΎΠ³Π΄Π° FΡ‚Ρ€= 0,1βˆ™ 10ΠΊΠ³ βˆ™ 9,8 Н/ΠΊΠ³ = 9,8Н (запись Π² Ρ‚Π΅Ρ‚Ρ€Π°Π΄ΡŒ).

Β 

4.Β Β Β Β  И прСдлагаСтся Ρ€Π°ΡΡΠΌΠΎΡ‚Ρ€Π΅Ρ‚ΡŒ Ρ‚Ρ€ΠΈ ситуации: прилоТСнная сила мСньшС значСния силы трСния скольТСния, Ρ€Π°Π²Π½Π° силС трСния скольТСния, ΠΈ большС силы трСния скольТСния:

F1 = 4Н,  F2 = 9,8Н,  F3 = 15Н.

5.Β Β Β Β  Π£Ρ‡Π΅Π½ΠΈΠΊΠΈ Π²Ρ‹Π΄Π²ΠΈΠ³Π°ΡŽΡ‚ Π³ΠΈΠΏΠΎΡ‚Π΅Π·Ρƒ: ΠΎ Ρ‚ΠΎΠΌ, Ρ‡Π΅ΠΌΡƒ Π²ΠΎ всСх Ρ‚Ρ€Ρ‘Ρ… случаях Ρ€Π°Π²Π½Π° сила трСния, ΠΈ ΠΊΠ°ΠΊ Π±ΡƒΠ΄Π΅ΠΌ вСсти сСбя Ρ‚Π΅Π»ΠΎ Π² этих ситуациях. ΠΠ½Π°Π»ΠΈΠ·ΠΈΡ€ΡƒΡŽΡ‚ΡΡ всС ситуации.

Β 

ΠŸΡƒΡΡ‚ΡŒ прилоТСнная сила  F = F1 = 4Н. Если ΠΏΡ€ΠΈΠ½ΡΡ‚ΡŒ силу трСния Ρ€Π°Π²Π½ΠΎΠΉ 9,8Н, Ρ‚ΠΎ Ρ‚Π΅Π»ΠΎ Π΄ΠΎΠ»ΠΆΠ½ΠΎ Π½Π°Ρ‡Π°Ρ‚ΡŒ Π΄Π²ΠΈΠ³Π°Ρ‚ΡŒΡΡΒ  с ускорСниСм  Π² сторону дСйствия силы трСния. Π’Π°ΠΊΠΎΠ³ΠΎ Π±Ρ‹Ρ‚ΡŒ Π½Π΅ ΠΌΠΎΠΆΠ΅Ρ‚! Π’Π΅Π»ΠΎ останСтся Π½Π° мСстС. А Ρ‚Π΅Π»ΠΎ остаётся Π² ΠΏΠΎΠΊΠΎΠ΅, Ссли гСомСтричСская сумма сил, ΠΏΡ€ΠΈΠ»ΠΎΠΆΠ΅Π½Π½Ρ‹Ρ… ΠΊ Π½Π΅ΠΌΡƒ Ρ€Π°Π²Π½Π° 0. ΠŸΠΎΡΡ‚ΠΎΠΌΡƒ ΠΌΠΎΠΆΠ½ΠΎ ΡΠ΄Π΅Π»Π°Ρ‚ΡŒ Π²Ρ‹Π²ΠΎΠ΄, Ρ‡Ρ‚ΠΎ ΠΌΠΎΠ΄ΡƒΠ»ΡŒ силы трСния Ρ€Π°Π²Π΅Π½ 4Н.Β  Π’Π°ΠΊΠΈΠΌ ΠΎΠ±Ρ€Π°Π·ΠΎΠΌ, прилоТСнная сила трСния Π΅ΡΡ‚ΡŒ сила трСния покоя.

Β 

ΠŸΠ΅Ρ€Π²Ρ‹ΠΉ Π²Ρ‹Π²ΠΎΠ΄ (Π² Ρ‚Π΅Ρ‚Ρ€Π°Π΄ΡŒ):

Если прилоТСнная сила F = F1 = 4Н, Ρ‚ΠΎ сила трСния Ρ€Π°Π²Π½Π° FΡ‚Ρ€ ΠΏ = 4Н ΠΈ являСтся силой трСния покоя. Π’Π΅Π»ΠΎ ΠΎΡ‚Π½ΠΎΡΠΈΡ‚Π΅Π»ΡŒΠ½ΠΎ ΠΎΠΏΠΎΡ€Ρ‹ Π½Π΅ пСрСмСщаСтся.

Β 

ΠŸΡƒΡΡ‚ΡŒ прилоТСнная сила  F = F1 = 9,8Н. Если ΠΏΡ€ΠΈΠ½ΡΡ‚ΡŒ силу трСния Ρ€Π°Π²Π½ΠΎΠΉ 9,8Н, Ρ‚ΠΎ гСомСтричСская сумма сил, ΠΏΡ€ΠΈΠ»ΠΎΠΆΠ΅Π½Π½Ρ‹Ρ… ΠΊ Ρ‚Π΅Π»Ρƒ Ρ€Π°Π²Π½Π° 0. Π’Π΅Π»ΠΎ останСтся  Π² ΠΏΠΎΠΊΠΎΠ΅ ΠΈΠ»ΠΈ, Ссли ΠΎΠ½ΠΎ двигалось с постоянной ΡΠΊΠΎΡ€ΠΎΡΡ‚ΡŒΡŽ, Ρ‚ΠΎ Π±ΡƒΠ΄Π΅Ρ‚ ΠΏΡ€ΠΎΠ΄ΠΎΠ»ΠΆΠ°Ρ‚ΡŒ Π΄Π²ΠΈΠΆΠ΅Π½ΠΈΠ΅, сохраняя свою ΡΠΊΠΎΡ€ΠΎΡΡ‚ΡŒ (По ΠΏΠ΅Ρ€Π²ΠΎΠΌΡƒ Π·Π°ΠΊΠΎΠ½Ρƒ ΠΡŒΡŽΡ‚ΠΎΠ½Π°). ΠŸΠΎΡΡ‚ΠΎΠΌΡƒ ΠΌΠΎΠΆΠ½ΠΎ ΡΠ΄Π΅Π»Π°Ρ‚ΡŒ Π²Ρ‹Π²ΠΎΠ΄, Ρ‡Ρ‚ΠΎ ΠΌΠΎΠ΄ΡƒΠ»ΡŒ силы трСния Ρ€Π°Π²Π΅Π½ 9,8Н.Β  Π’Π°ΠΊΠΈΠΌ ΠΎΠ±Ρ€Π°Π·ΠΎΠΌ, прилоТСнная сила трСния Π΅ΡΡ‚ΡŒ сила трСния скольТСния ΠΈΠ»ΠΈ максимальная сила трСния покоя.

Β 

Π’Ρ‚ΠΎΡ€ΠΎΠΉ Π²Ρ‹Π²ΠΎΠ΄ (Π² Ρ‚Π΅Ρ‚Ρ€Π°Π΄ΡŒ):

Если прилоТСнная сила F = F2 = 9,8Н, Ρ‚ΠΎ сила трСния Ρ€Π°Π²Π½Π° FΡ‚Ρ€ = 9,8Н ΠΈ являСтся силой трСния скольТСния ΠΈΠ»ΠΈ максимальной силой трСния покоя. Π’Π΅Π»ΠΎ ΠΎΡ‚Π½ΠΎΡΠΈΡ‚Π΅Π»ΡŒΠ½ΠΎ ΠΎΠΏΠΎΡ€Ρ‹ сохраняСт свою ΡΠΊΠΎΡ€ΠΎΡΡ‚ΡŒ (Π’ состоянии покоя ΡΠΊΠΎΡ€ΠΎΡΡ‚ΡŒ Ρ€Π°Π²Π½Π° 0, ΠΈ ΠΎΠ½Π° Ρ‚ΠΎΠΆΠ΅ сохраняСтся).

Β 

ΠŸΡƒΡΡ‚ΡŒ прилоТСнная сила  F = F3 = 15Н. Максимально Π²ΠΎΠ·ΠΌΠΎΠΆΠ½ΠΎΠ΅ Π·Π½Π°Ρ‡Π΅Π½ΠΈΠ΅ силы трСния — 9,8Н. ΠŸΡ€ΠΈ Π·Π°Π΄Π°Π½Π½Ρ‹Ρ… условиях  Π΅Ρ‘ Π·Π½Π°Ρ‡Π΅Π½ΠΈΠ΅ большС Π±Ρ‹Ρ‚ΡŒ Π½Π΅ ΠΌΠΎΠΆΠ΅Ρ‚! ГСомСтричСская сумма сил Π½Π΅ Ρ€Π°Π²Π½Π° Π½ΡƒΠ»ΡŽ. Π‘Π»Π΅Π΄ΠΎΠ²Π°Ρ‚Π΅Π»ΡŒΠ½ΠΎ, Ρ‚Π΅Π»ΠΎ Π±ΡƒΠ΄Π΅Ρ‚ Π΄Π²ΠΈΠ³Π°Ρ‚ΡŒΡΡ с ускорСниСм, Π½Π°ΠΏΡ€Π°Π²Π»Π΅Π½Π½Ρ‹ΠΌ Π² сторону Ρ€Π°Π²Π½ΠΎΠ΄Π΅ΠΉΡΡ‚Π²ΡƒΡŽΡ‰Π΅ΠΉ сил, Ρ‚.Π΅. Π² сторону дСйствия силы F (По Π²Ρ‚ΠΎΡ€ΠΎΠΌΡƒ Π·Π°ΠΊΠΎΠ½Ρƒ ΠΡŒΡŽΡ‚ΠΎΠ½Π°). ΠŸΠΎΡΡ‚ΠΎΠΌΡƒ ΠΌΠΎΠΆΠ½ΠΎ ΡΠ΄Π΅Π»Π°Ρ‚ΡŒ Π²Ρ‹Π²ΠΎΠ΄, Ρ‡Ρ‚ΠΎ ΠΌΠΎΠ΄ΡƒΠ»ΡŒ силы трСния Ρ€Π°Π²Π΅Π½ 9,8 Н, ΠΈ сила трСния Π΅ΡΡ‚ΡŒ сила трСния скольТСния.

Β 

Π’Ρ€Π΅Ρ‚ΠΈΠΉ Β Π²Ρ‹Π²ΠΎΠ΄ (Π² Ρ‚Π΅Ρ‚Ρ€Π°Π΄ΡŒ):

Если прилоТСнная сила F = F3 = 15Н, Ρ‚ΠΎ сила трСния Ρ€Π°Π²Π½Π° FΡ‚Ρ€ = 9,8Н ΠΈ являСтся силой трСния скольТСния. Π’Π΅Π»ΠΎ ΠΎΡ‚Π½ΠΎΡΠΈΡ‚Π΅Π»ΡŒΠ½ΠΎ ΠΎΠΏΠΎΡ€Ρ‹ двиТСтся  с ускорСниСм, Π½Π°ΠΏΡ€Π°Π²Π»Π΅Π½Π½Ρ‹ΠΌ Π² сторону Ρ€Π°Π²Π½ΠΎΠ΄Π΅ΠΉΡΡ‚Π²ΡƒΡŽΡ‰Π΅ΠΉ сил. (УскорСниС Ρ€Π°Π²Π½ΠΎ 0,52 ΠΌ/с2)

Β 

6.Β Β Β Β  Π”Π°Π»Π΅Π΅ выполняСтся Π³Ρ€Π°Ρ„ΠΈΠΊ зависимости значСния силы трСния ΠΎΡ‚ значСния ΠΏΡ€ΠΈΠ»ΠΎΠΆΠ΅Π½Π½ΠΎΠΉ силы ΠΈΠ· ΠΌΠ°Ρ‚Π΅Ρ€ΠΈΠ°Π»ΠΎΠ² Ρ€Π°Π·Π΄Π΅Π»Π° рСсурса «Готовимся ΠΊ ΡƒΡ€ΠΎΠΊΡƒ. Π”ΠΈΠ½Π°ΠΌΠΈΠΊΠ°Β».

Β 

 рис.4

Β 

7.Β Β Β Β  ΠŸΡ€ΠΈ нСобходимости ΠΌΠΎΠΆΠ½ΠΎ Π°Π½Π°Π»ΠΎΠ³ΠΈΡ‡Π½ΡƒΡŽ схСму ΠΏΡ€ΠΈΠΌΠ΅Π½ΠΈΡ‚ΡŒ ΠΊ Π·Π°Π΄Π°Ρ‡Π΅ с Π½Π°ΠΊΠ»ΠΎΠ½Π½ΠΎΠΉ ΠΏΠ»ΠΎΡΠΊΠΎΡΡ‚ΡŒΡŽ.

Если, ΡƒΠ²Π°ΠΆΠ°Π΅ΠΌΡ‹Π΅ ΠΊΠΎΠ»Π»Π΅Π³ΠΈ, Π’Π°ΠΌ эти ΠΏΡ€ΠΈΡ‘ΠΌΡ‹ ΠΏΠΎΠΌΠΎΠ³ΡƒΡ‚, Π±ΡƒΠ΄Ρƒ Ρ€Π°Π΄Π°. Π–Π΅Π»Π°ΡŽ успСха.

Β 

Β 

Β 

Π²Π΅Ρ€Π½ΡƒΡ‚ΡŒΡΡ ΠΊ Ρ€Π°Π·Π΄Π΅Π»Ρƒ «Π˜Π· ΠΎΠΏΡ‹Ρ‚Π° Ρ€Π°Π±ΠΎΡ‚Ρ‹»

Β 

Π‘ΠΈΠ»Π° трСния

Π‘ΠΈΠ»Π°, Π²ΠΎΠ·Π½ΠΈΠΊΠ°ΡŽΡ‰Π°Ρ Π½Π° Π³Ρ€Π°Π½ΠΈΡ†Π΅ соприкосновСния Ρ‚Π΅Π» ΠΏΡ€ΠΈ отсутствии ΠΎΡ‚Π½ΠΎΡΠΈΡ‚Π΅Π»ΡŒΠ½ΠΎΠ³ΠΎ двиТСния Ρ‚Π΅Π», называСтся силой трСния покоя.
Π‘ΠΈΠ»Π° трСния покоя FΡ‚Ρ€ Ρ€Π°Π²Π½Π° ΠΏΠΎ ΠΌΠΎΠ΄ΡƒΠ»ΡŽ внСшнСй силС F, Π½Π°ΠΏΡ€Π°Π²Π»Π΅Π½Π½ΠΎΠΉ ΠΏΠΎ ΠΊΠ°ΡΠ°Ρ‚Π΅Π»ΡŒΠ½ΠΎΠΉ ΠΊ повСрхности соприкосновСния Ρ‚Π΅Π», ΠΈ ΠΏΡ€ΠΎΡ‚ΠΈΠ²ΠΎΠΏΠΎΠ»ΠΎΠΆΠ½Π° Π΅ΠΉ ΠΏΠΎ Π½Π°ΠΏΡ€Π°Π²Π»Π΅Π½ΠΈΡŽ:



ΠŸΡ€ΠΈΠΊΡ€Π΅ΠΏΠΈΠΌ Π΄ΠΈΠ½Π°ΠΌΠΎΠΌΠ΅Ρ‚Ρ€ ΠΊ бруску ΠΈ заставим брусок Π΄Π²ΠΈΠ³Π°Ρ‚ΡŒΡΡ Ρ€Π°Π²Π½ΠΎΠΌΠ΅Ρ€Π½ΠΎ ΠΏΠΎ Π³ΠΎΡ€ΠΈΠ·ΠΎΠ½Ρ‚Π°Π»ΡŒΠ½ΠΎΠΉ повСрхности стола. Π’ΠΎ врСмя Ρ€Π°Π²Π½ΠΎΠΌΠ΅Ρ€Π½ΠΎΠ³ΠΎ двиТСния бруска Π΄ΠΈΠ½Π°ΠΌΠΎΠΌΠ΅Ρ‚Ρ€ ΠΏΠΎΠΊΠ°Π·Ρ‹Π²Π°Π΅Ρ‚, Ρ‡Ρ‚ΠΎ Π½Π° брусок со стороны ΠΏΡ€ΡƒΠΆΠΈΠ½Ρ‹ дСйствуСт постоянная сила упругости FΡƒΠΏΡ€. ΠŸΡ€ΠΈ Ρ€Π°Π²Π½ΠΎΠΌΠ΅Ρ€Π½ΠΎΠΌ Π΄Π²ΠΈΠΆΠ΅Π½ΠΈΠΈ бруска Ρ€Π°Π²Π½ΠΎΠ΄Π΅ΠΉΡΡ‚Π²ΡƒΡŽΡ‰Π°Ρ всСх сил Ρ€Π°Π²Π½Π° Π½ΡƒΠ»ΡŽ. Π‘Π»Π΅Π΄ΠΎΠ²Π°Ρ‚Π΅Π»ΡŒΠ½ΠΎ, ΠΊΡ€ΠΎΠΌΠ΅ силы упругости, Π²ΠΎ врСмя Ρ€Π°Π²Π½ΠΎΠΌΠ΅Ρ€Π½ΠΎΠ³ΠΎ двиТСния Π½Π° брусок дСйствуСт сила, равная ΠΏΠΎ ΠΌΠΎΠ΄ΡƒΠ»ΡŽ силС упругости, Π½ΠΎ направлСнная Π² ΠΏΡ€ΠΎΡ‚ΠΈΠ²ΠΎΠΏΠΎΠ»ΠΎΠΆΠ½ΡƒΡŽ сторону. Π­Ρ‚Π° сила называСтся силой трСния скольТСния FΡ‚Ρ€.
Π’Π΅ΠΊΡ‚ΠΎΡ€ силы трСния скольТСния FΡ‚Ρ€ всСгда Π½Π°ΠΏΡ€Π°Π²Π»Π΅Π½ ΠΏΡ€ΠΎΡ‚ΠΈΠ²ΠΎΠΏΠΎΠ»ΠΎΠΆΠ½ΠΎ Π²Π΅ΠΊΡ‚ΠΎΡ€Ρƒ скорости двиТСния Ρ‚Π΅Π»Π° ΠΎΡ‚Π½ΠΎΡΠΈΡ‚Π΅Π»ΡŒΠ½ΠΎ ΡΠΎΠΏΡ€ΠΈΠΊΠ°ΡΠ°ΡŽΡ‰Π΅Π³ΠΎΡΡ с Π½ΠΈΠΌ Ρ‚Π΅Π»Π°. ΠŸΠΎΡΡ‚ΠΎΠΌΡƒ дСйствиС силы трСния скольТСния всСгда ΠΏΡ€ΠΈΠ²ΠΎΠ΄ΠΈΡ‚ ΠΊ ΡƒΠΌΠ΅Π½ΡŒΡˆΠ΅Π½ΠΈΡŽ модуля ΠΎΡ‚Π½ΠΎΡΠΈΡ‚Π΅Π»ΡŒΠ½ΠΎΠΉ скорости Ρ‚Π΅Π».
Π‘ΠΈΠ»Ρ‹ трСния Π²ΠΎΠ·Π½ΠΈΠΊΠ°ΡŽΡ‚ благодаря ΡΡƒΡ‰Π΅ΡΡ‚Π²ΠΎΠ²Π°Π½ΠΈΡŽ сил взаимодСйствия ΠΌΠ΅ΠΆΠ΄Ρƒ ΠΌΠΎΠ»Π΅ΠΊΡƒΠ»Π°ΠΌΠΈ ΠΈ Π°Ρ‚ΠΎΠΌΠ°ΠΌΠΈ ΡΠΎΠΏΡ€ΠΈΠΊΠ°ΡΠ°ΡŽΡ‰ΠΈΡ…ΡΡ Ρ‚Π΅Π». ПослСдниС обусловлСны взаимодСйствиСм элСктричСских зарядов, ΠΊΠΎΡ‚ΠΎΡ€Ρ‹ΠΌΠΈ ΠΎΠ±Π»Π°Π΄Π°ΡŽΡ‚ частицы, входящиС Π² состав Π°Ρ‚ΠΎΠΌΠΎΠ². На основании ΠΎΠΏΡ‹Ρ‚ΠΎΠ² ΠΌΠΎΠΆΠ½ΠΎ ΡΠ΄Π΅Π»Π°Ρ‚ΡŒ Π²Ρ‹Π²ΠΎΠ΄, Ρ‡Ρ‚ΠΎ максимальноС Π·Π½Π°Ρ‡Π΅Π½ΠΈΠ΅ модуля силы трСния покоя прямо ΠΏΡ€ΠΎΠΏΠΎΡ€Ρ†ΠΈΠΎΠ½Π°Π»ΡŒΠ½ΠΎ силС давлСния.
ВзаимодСйствиС Ρ‚Π΅Π»Π° ΠΈ ΠΎΠΏΠΎΡ€Ρ‹ Π²Ρ‹Π·Ρ‹Π²Π°Π΅Ρ‚ Π΄Π΅Ρ„ΠΎΡ€ΠΌΠ°Ρ†ΠΈΡŽ ΠΈ Ρ‚Π΅Π»Π°, ΠΈ ΠΎΠΏΠΎΡ€Ρ‹. Π‘ΠΈΠ»Ρƒ упругости N, Π²ΠΎΠ·Π½ΠΈΠΊΠ°ΡŽΡ‰ΡƒΡŽ Π² Ρ€Π΅Π·ΡƒΠ»ΡŒΡ‚Π°Ρ‚Π΅ Π΄Π΅Ρ„ΠΎΡ€ΠΌΠ°Ρ†ΠΈΠΈ ΠΎΠΏΠΎΡ€Ρ‹ ΠΈ Π΄Π΅ΠΉΡΡ‚Π²ΡƒΡŽΡ‰ΡƒΡŽ Π½Π° Ρ‚Π΅Π»ΠΎ, Π½Π°Π·Ρ‹Π²Π°ΡŽΡ‚ силой Ρ€Π΅Π°ΠΊΡ†ΠΈΠΈ ΠΎΠΏΠΎΡ€Ρ‹.



По Ρ‚Ρ€Π΅Ρ‚ΡŒΠ΅ΠΌΡƒ Π·Π°ΠΊΠΎΠ½Ρƒ ΠΡŒΡŽΡ‚ΠΎΠ½Π° сила давлСния ΠΈ сила Ρ€Π΅Π°ΠΊΡ†ΠΈΠΈ ΠΎΠΏΠΎΡ€Ρ‹ Ρ€Π°Π²Π½Ρ‹ ΠΏΠΎ ΠΌΠΎΠ΄ΡƒΠ»ΡŽ ΠΈ ΠΏΡ€ΠΎΡ‚ΠΈΠ²ΠΎΠΏΠΎΠ»ΠΎΠΆΠ½Ρ‹ ΠΏΠΎ Π½Π°ΠΏΡ€Π°Π²Π»Π΅Π½ΠΈΡŽ. ΠŸΠΎΡΡ‚ΠΎΠΌΡƒ ΠΏΡ€Π΅Π΄Ρ‹Π΄ΡƒΡ‰ΠΈΠΉ Π²Ρ‹Π²ΠΎΠ΄ ΠΌΠΎΠΆΠ½ΠΎ ΡΡ„ΠΎΡ€ΠΌΡƒΠ»ΠΈΡ€ΠΎΠ²Π°Ρ‚ΡŒ Ρ‚Π°ΠΊ: ΠΌΠΎΠ΄ΡƒΠ»ΡŒ максимальной силы трСния покоя ΠΏΡ€ΠΎΠΏΠΎΡ€Ρ†ΠΈΠΎΠ½Π°Π»Π΅Π½ силС Ρ€Π΅Π°ΠΊΡ†ΠΈΠΈ ΠΎΠΏΠΎΡ€Ρ‹:

FΡ‚Ρ€max = ΞΌ Β· N

ГрСчСской Π±ΡƒΠΊΠ²ΠΎΠΉ ΞΌ (мю) ΠΎΠ±ΠΎΠ·Π½Π°Ρ‡Π΅Π½ коэффициСнт ΠΏΡ€ΠΎΠΏΠΎΡ€Ρ†ΠΈΠΎΠ½Π°Π»ΡŒΠ½ΠΎΡΡ‚ΠΈ, Π½Π°Π·Ρ‹Π²Π°Π΅ΠΌΡ‹ΠΉ коэффициСнтом трСния.
ΠœΠΎΠ΄ΡƒΠ»ΡŒ силы трСния скольТСния FΡ‚Ρ€, ΠΊΠ°ΠΊ ΠΈ ΠΌΠΎΠ΄ΡƒΠ»ΡŒ максимальной силы трСния покоя, ΠΏΡ€ΠΎΠΏΠΎΡ€Ρ†ΠΈΠΎΠ½Π°Π»Π΅Π½ ΠΌΠΎΠ΄ΡƒΠ»ΡŽ силы Ρ€Π΅Π°ΠΊΡ†ΠΈΠΈ ΠΎΠΏΠΎΡ€Ρ‹:

FΡ‚Ρ€ = ΞΌ Β· N

МаксимальноС Π·Π½Π°Ρ‡Π΅Π½ΠΈΠ΅ силы трСния покоя ΠΏΡ€ΠΈΠΌΠ΅Ρ€Π½ΠΎ Ρ€Π°Π²Π½ΠΎ силС трСния скольТСния, ΠΏΡ€ΠΈΠ±Π»ΠΈΠΆΠ΅Π½Π½ΠΎ Ρ€Π°Π²Π½Ρ‹ Ρ‚Π°ΠΊΠΆΠ΅ коэффициСнты трСния покоя ΠΈ скольТСния.
Π‘ΠΈΠ»Ρ‹ трСния Π²ΠΎΠ·Π½ΠΈΠΊΠ°ΡŽΡ‚ Ρ‚Π°ΠΊ ΠΆΠ΅ ΠΈ ΠΏΡ€ΠΈ ΠΊΠ°Ρ‡Π΅Π½ΠΈΠΈ Ρ‚Π΅Π»Π°. ΠŸΡ€ΠΈ ΠΎΠ΄ΠΈΠ½Π°ΠΊΠΎΠ²ΠΎΠΉ Π½Π°Π³Ρ€ΡƒΠ·ΠΊΠ΅ сила трСния качСния Π·Π½Π°Ρ‡ΠΈΡ‚Π΅Π»ΡŒΠ½ΠΎ мСньшС силы трСния скольТСния. ΠŸΠΎΡΡ‚ΠΎΠΌΡƒ для ΡƒΠΌΠ΅Π½ΡŒΡˆΠ΅Π½ΠΈΡ сил трСния Π² Ρ‚Π΅Ρ…Π½ΠΈΠΊΠ΅ ΠΏΡ€ΠΈΠΌΠ΅Π½ΡΡŽΡ‚ΡΡ колСса, ΡˆΠ°Ρ€ΠΈΠΊΠΎΠ²Ρ‹Π΅ ΠΈ Ρ€ΠΎΠ»ΠΈΠΊΠΎΠ²Ρ‹Π΅ подшипники.

Π”Ρ€ΡƒΠ³ΠΈΠ΅ Π·Π°ΠΌΠ΅Ρ‚ΠΊΠΈ ΠΏΠΎ Ρ„ΠΈΠ·ΠΈΠΊΠ΅

Π‘ΠΎΠΏΡ€ΠΎΡ‚ΠΈΠ²Π»Π΅Π½ΠΈΠ΅ ΠΊΠ°Ρ‡Π΅Π½ΠΈΡŽ ΠΈ ΠΏΡ€ΠΎΠΌΡ‹ΡˆΠ»Π΅Π½Π½Ρ‹Π΅ колёса

Π’Ρ€Π΅Π½ΠΈΠ΅ ΠΈ сопротивлСниС ΠΊΠ°Ρ‡Π΅Π½ΠΈΡŽ

ΠŸΡ€ΠΎΡ†Π΅ΡΡ трСния (Ρ„Ρ€ΠΈΠΊΡ†ΠΈΠΎΠ½Π½ΠΎΠ΅ взаимодСйствиС) ΠΈΠ³Ρ€Π°Π΅Ρ‚ Π²Π°ΠΆΠ½ΡƒΡŽ Ρ€ΠΎΠ»ΡŒ Π² ΠΏΡ€ΠΎΠΌΡ‹ΡˆΠ»Π΅Π½Π½ΠΎΠΌ ΠΌΠΈΡ€Π΅ ΠΈ повсСднСвной ΠΆΠΈΠ·Π½ΠΈ. Π‘ΠΈΠ»Π° трСния ΠΎΠΊΠ°Π·Ρ‹Π²Π°Π΅Ρ‚ сопротивлСниС скольТСнию, Π²Ρ€Π°Ρ‰Π΅Π½ΠΈΡŽ, ΠΊΠ°Ρ‡Π΅Π½ΠΈΡŽ, ΠΏΠΎΠ»Ρ‘Ρ‚Ρƒ ΠΎΠ±ΡŠΠ΅ΠΊΡ‚Π° ΠΈΠ·-Π·Π° Π΅Π³ΠΎ ΠΊΠΎΠ½Ρ‚Π°ΠΊΡ‚Π° с Π΄Ρ€ΡƒΠ³ΠΈΠΌ ΠΎΠ±ΡŠΠ΅ΠΊΡ‚ΠΎΠΌ. Она ΠΌΠΎΠΆΠ΅Ρ‚ Π±Ρ‹Ρ‚ΡŒ ΠΏΠΎΠ»Π΅Π·Π½ΠΎΠΉ (ΠΊ ΠΏΡ€ΠΈΠΌΠ΅Ρ€Ρƒ, ΠΊΠΎΠ³Π΄Π° Π½ΡƒΠΆΠ½ΠΎ Π·Π°Π΄Π΅ΠΉΡΡ‚Π²ΠΎΠ²Π°Ρ‚ΡŒ Ρ‚ΠΎΡ€ΠΌΠΎΠ·Π°, Ρ‡Ρ‚ΠΎΠ±Ρ‹ ΠΎΡΡ‚Π°Π½ΠΎΠ²ΠΈΡ‚ΡŒ Π°Π²Ρ‚ΠΎΠΌΠΎΠ±ΠΈΠ»ΡŒ), ΠΈΠ»ΠΈ Π²Ρ€Π΅Π΄Π½ΠΎΠΉ (ΠΏΡ€ΠΈ ΠΏΠΎΠΏΡ‹Ρ‚ΠΊΠ΅ Π΅Ρ…Π°Ρ‚ΡŒ с Π½ΠΎΠ³ΠΎΠΉ Π½Π° ΠΏΠ΅Π΄Π°Π»ΠΈ Ρ‚ΠΎΡ€ΠΌΠΎΠ·Π°). Π­Ρ‚Π° ΡΡ‚Π°Ρ‚ΡŒΡ расскаТСт ΠΎ Π²Π°ΠΆΠ½ΠΎΠΌ аспСктС ΠΏΡ€ΠΎΠΌΡ‹ΡˆΠ»Π΅Π½Π½Ρ‹Ρ… колёс – ΠΎ сопротивлСнии ΠΊΠ°Ρ‡Π΅Π½ΠΈΡŽ.

Π‘ΠΎΠΏΡ€ΠΎΡ‚ΠΈΠ²Π»Π΅Π½ΠΈΠ΅ ΠΊΠ°Ρ‡Π΅Π½ΠΈΡŽ – ΠΏΡ€ΠΈΡ‚ΠΎΡ€ΠΌΠ°ΠΆΠΈΠ²Π°ΡŽΡ‰Π΅Π΅ дСйствиС, ΠΊΠΎΡ‚ΠΎΡ€ΠΎΠ΅ ΠΎΠΊΠ°Π·Ρ‹Π²Π°Π΅Ρ‚ ΠΏΠΎΠ²Π΅Ρ€Ρ…Π½ΠΎΡΡ‚ΡŒ ΠΏΠΎΠ»Π° Π½Π° ΡˆΠΈΠ½ΠΊΡƒ (ΠΊΠΎΠ½Ρ‚Π°ΠΊΡ‚Π½Ρ‹ΠΉ слой) катящСгося колСса. Оно являСтся ΠΌΠ΅Ρ€ΠΎΠΉ энСргии, потСрянной Π½Π° ΠΎΠΏΡ€Π΅Π΄Π΅Π»Ρ‘Π½Π½ΠΎΠΌ расстоянии.Β 

Рассмотрим катящССся ΠΏΠΎ плоской повСрхности колСсо. Π•Π³ΠΎ шинка дСформируСтся, Ρ‡Ρ‚ΠΎ Π²Ρ‹Π·Ρ‹Π²Π°Π΅Ρ‚ Π½Π΅ΠΊΠΎΡ‚ΠΎΡ€ΠΎΠ΅ сопротивлСниС двиТСнию качСния. Плоская ΠΏΠΎΠ²Π΅Ρ€Ρ…Π½ΠΎΡΡ‚ΡŒ Ρ‚Π°ΠΊΠΆΠ΅ ΠΌΠΎΠΆΠ΅Ρ‚ Π΄Π΅Ρ„ΠΎΡ€ΠΌΠΈΡ€ΠΎΠ²Π°Ρ‚ΡŒΡΡ, особСнно Ссли ΠΎΠ½Π° мягкая. Π₯ΠΎΡ€ΠΎΡˆΠΈΠ΅ ΠΏΡ€ΠΈΠΌΠ΅Ρ€Ρ‹ сильно ΡΠΎΠΏΡ€ΠΎΡ‚ΠΈΠ²Π»ΡΡŽΡ‰ΠΈΡ…ΡΡ Π²Ρ€Π°Ρ‰Π΅Π½ΠΈΡŽ повСрхностСй  – Π³Ρ€ΡΠ·ΡŒ ΠΈΠ»ΠΈ пСсок. ΠšΠ°Ρ‚ΠΈΡ‚ΡŒ Ρ‚Π΅Π»Π΅ΠΆΠΊΡƒ ΠΏΠΎ Π°ΡΡ„Π°Π»ΡŒΡ‚Ρƒ Π·Π½Π°Ρ‡ΠΈΡ‚Π΅Π»ΡŒΠ½ΠΎ Π»Π΅Π³Ρ‡Π΅, Ρ‡Π΅ΠΌ ΠΏΠΎ пСску.Β 

Π€Π°ΠΊΡ‚ΠΎΡ€Ρ‹, Π²Π»ΠΈΡΡŽΡ‰ΠΈΠ΅ Π½Π° рассСиваниС энСргии катящСгося ΠΏΡ€ΠΎΠΌΡ‹ΡˆΠ»Π΅Π½Π½ΠΎΠ³ΠΎ колСса:

  • Β Β  Β Ρ‚Ρ€Π΅Π½ΠΈΠ΅ ΠΊΠΎΠ½Ρ‚Π°ΠΊΡ‚ΠΈΡ€ΡƒΡŽΡ‰ΠΈΡ… повСрхностСй;
  • Β Β  Β ΡƒΠΏΡ€ΡƒΠ³ΠΈΠ΅ свойства ΠΌΠ°Ρ‚Π΅Ρ€ΠΈΠ°Π»ΠΎΠ²;
  • Β Β  Β Π³Ρ€ΡƒΠ±ΠΎΡΡ‚ΡŒ повСрхностСй.
На рисункС 1: ДСформация повСрхностСй происходит Π΄ΠΎ стСпСни, ΠΎΠΏΡ€Π΅Π΄Π΅Π»Ρ‘Π½Π½ΠΎΠΉ ΠΈΡ… ΡƒΠΏΡ€ΡƒΠ³ΠΈΠΌΠΈ свойствами.

Π’Ρ€Π΅Π½ΠΈΠ΅ качСния ΠΈ Ρ‚Ρ€Π΅Π½ΠΈΠ΅ скольТСния 

ΠšΠΎΡΡ„Ρ„ΠΈΡ†ΠΈΠ΅Π½Ρ‚ трСния качСния Π½Π΅ слСдуСт ΠΏΡƒΡ‚Π°Ρ‚ΡŒ с коэффициСнтом трСния скольТСния. ΠšΠΎΡΡ„Ρ„ΠΈΡ†ΠΈΠ΅Π½Ρ‚ трСния скольТСния Π²Ρ‹Ρ€Π°ΠΆΠ°Π΅Ρ‚ ΠΎΡ‚Π½ΠΎΡˆΠ΅Π½ΠΈΠ΅ силы трСния ΠΌΠ΅ΠΆΠ΄Ρƒ Ρ‚Π΅Π»Π°ΠΌΠΈ ΠΈ силы, ΠΏΡ€ΠΈΠΆΠΈΠΌΠ°ΡŽΡ‰Π΅ΠΉ Ρ‚Π΅Π»Π° Π΄Ρ€ΡƒΠ³ ΠΊ Π΄Ρ€ΡƒΠ³Ρƒ. Π”Π°Π½Π½Ρ‹ΠΉ коэффициСнт зависит ΠΎΡ‚ Ρ‚ΠΈΠΏΠ° ΠΈΡΠΏΠΎΠ»ΡŒΠ·ΡƒΠ΅ΠΌΡ‹Ρ… ΠΌΠ°Ρ‚Π΅Ρ€ΠΈΠ°Π»ΠΎΠ². К ΠΏΡ€ΠΈΠΌΠ΅Ρ€Ρƒ, ΡΡ‚Π°Π»ΡŒ Π½Π° Π»ΡŒΠ΄Ρƒ ΠΈΠΌΠ΅Π΅Ρ‚ Π½ΠΈΠ·ΠΊΠΈΠΉ коэффициСнт трСния, Π° Ρ€Π΅Π·ΠΈΠ½Π° Π½Π° Π°ΡΡ„Π°Π»ΡŒΡ‚Π΅ ΠΈΠΌΠ΅Π΅Ρ‚ высокий коэффициСнт трСния.Β 

Рисунок 2 поясняСт понятиС трСния скольТСния. ΠŸΡ€Π΅Π΄ΡΡ‚Π°Π²ΡŒΡ‚Π΅ силу, ΠΊΠΎΡ‚ΠΎΡ€ΡƒΡŽ Π½ΡƒΠΆΠ½ΠΎ ΠΏΡ€ΠΈΠΌΠ΅Π½ΠΈΡ‚ΡŒ, Ρ‡Ρ‚ΠΎΠ±Ρ‹ ΠΏΡ€ΠΎΡ‚ΡΠ½ΡƒΡ‚ΡŒ тяТёлый ящик ΠΏΠΎ ΠΏΠΎΠ»Ρƒ. БтатичСскоС Ρ‚Ρ€Π΅Π½ΠΈΠ΅ Ρ‚Ρ€Π΅Π±ΡƒΠ΅Ρ‚ примСнСния ΠΎΠΏΡ€Π΅Π΄Π΅Π»Ρ‘Π½Π½ΠΎΠΉ силы, Ρ‡Ρ‚ΠΎΠ±Ρ‹ ΡΠ΄Π²ΠΈΠ½ΡƒΡ‚ΡŒ ящик с мСста. Π‘ Π½Π°Ρ‡Π°Π»ΠΎΠΌ двиТСния, Π²ΠΎΠ·Π½ΠΈΠΊΠ°Π΅Ρ‚ динамичСскоС Ρ‚Ρ€Π΅Π½ΠΈΠ΅, Ρ‚Ρ€Π΅Π±ΡƒΡŽΡ‰Π΅Π΅ постоянного прилоТСния ΠΎΠΏΡ€Π΅Π΄Π΅Π»Π΅Π½Π½ΠΎΠΉ силы для поддСрТания двиТСния. Π’ этом ΠΏΡ€ΠΈΠΌΠ΅Ρ€Π΅, Ρ‡Π΅Π»ΠΎΠ²Π΅ΠΊ, Ρ‚ΠΎΠ»ΠΊΠ°ΡŽΡ‰ΠΈΠΉ ящик, ΠΏΡ€ΠΈΠΊΠ»Π°Π΄Ρ‹Π²Π°Π΅Ρ‚ силу Fapp, ящик вСсит N, Π° ΠΏΠΎΠ» создаСт силу трСния f, которая сопротивляСтся двиТСнию. Β 

ΠŸΡ€ΠΈΡ‡ΠΈΠ½Π°, ΠΏΠΎ ΠΊΠΎΡ‚ΠΎΡ€ΠΎΠΉ ΠΌΡ‹ ΠΈΡΠΏΠΎΠ»ΡŒΠ·ΡƒΠ΅ΠΌ колёса для пСрСмСщСния ΠΌΠ°Ρ‚Π΅Ρ€ΠΈΠ°Π»ΠΎΠ² Π² Ρ‚ΠΎΠΌ, Ρ‡Ρ‚ΠΎ ΠΎΠ½ΠΈ ΠΏΠΎΠ·Π²ΠΎΠ»ΡΡŽΡ‚ Ρ‚Ρ€Π°Ρ‚ΠΈΡ‚ΡŒ Π·Π½Π°Ρ‡ΠΈΡ‚Π΅Π»ΡŒΠ½ΠΎ мСньшС силы. ΠŸΡ€Π΅Π΄ΡΡ‚Π°Π²ΡŒΡ‚Π΅, Ρ‡Ρ‚ΠΎ приходится Π²ΠΎΠ»ΠΎΡ‡ΡŒ Ρ…ΠΎΠ»ΠΎΠ΄ΠΈΠ»ΡŒΠ½ΠΈΠΊ ΠΈΠ»ΠΈ ΠΏΠΈΠ°Π½ΠΈΠ½ΠΎ! Π‘ΠΎΠ»Π΅Π΅ Ρ‚ΠΎΠ³ΠΎ, ΠΏΠΎΠ΄ΡƒΠΌΠ°ΠΉΡ‚Π΅, насколько Π»Π΅Π³Ρ‡Π΅ Π±Ρ‹Π»ΠΎ Π±Ρ‹ ΠΏΠ΅Ρ€Π΅Π΄Π²ΠΈΠ½ΡƒΡ‚ΡŒ Π²Ρ‹ΡˆΠ΅ΡƒΠΏΠΎΠΌΡΠ½ΡƒΡ‚Ρ‹ΠΉ ящик, Ссли Π±Ρ‹ ΠΏΡ€ΠΈΠΌΠ΅Π½ΡΠ»ΠΈΡΡŒ колёса.Β 

Π‘ΠΈΠ»Π°, трСбуСмая для пСрСдвиТСния оборудования Π½Π° колёсах, Π²Π΅Π»ΠΈΠΊΠ° Ρ‚ΠΎΠ»ΡŒΠΊΠΎ ΠΏΡ€ΠΈ стартС. Π•Π΅ часто Π½Π°Π·Ρ‹Π²Π°ΡŽΡ‚ Β«ΠΏΠ΅Ρ€Π²ΠΎΠ½Π°Ρ‡Π°Π»ΡŒΠ½ΠΎΠΉ ΠΈΠ»ΠΈ «стартовой» силой. Как Ρ‚ΠΎΠ»ΡŒΠΊΠΎ ΠΏΠΎΠ»ΡƒΡ‡Π΅Π½ΠΎ Π½ΡƒΠΆΠ½ΠΎΠ΅ ускорСниС, для продолТСния двиТСния Π½Π΅ΠΎΠ±Ρ…ΠΎΠ΄ΠΈΠΌΠ° Π³ΠΎΡ€Π°Π·Π΄ΠΎ мСньшая сила, ΠΊΠΎΡ‚ΠΎΡ€ΡƒΡŽ Π½Π°Π·Ρ‹Π²Π°ΡŽΡ‚ Β«ΠΏΠ΅Ρ€ΠΌΠ°Π½Π΅Π½Ρ‚Π½ΠΎΠΉΒ» ΠΈΠ»ΠΈ «катящСй». Как ΠΏΡ€Π°Π²ΠΈΠ»ΠΎ «стартовая» сила ΠΏΡ€Π΅Π²Ρ‹ΡˆΠ°Π΅Ρ‚ Π΅Π΅ Π² 2-2.5 Ρ€Π°Π·Π°.Β 

Расчёт силы трСния качСния

ΠŸΠΎΠΌΠΎΡ‡ΡŒ ΡƒΠ·Π½Π°Ρ‚ΡŒ сопротивлСниС ΠΊΠ°Ρ‡Π΅Π½ΠΈΡŽ ΠΏΡ€ΠΎΠΌΡ‹ΡˆΠ»Π΅Π½Π½Ρ‹Ρ… колёс ΠΏΠΎΠΌΠΎΠ³Π°Π΅Ρ‚ коэффициСнт трСния качСния. Π•Π³ΠΎ Π·Π½Π°Ρ‡Π΅Π½ΠΈΠ΅ для Ρ€Π°Π·Π»ΠΈΡ‡Π½Ρ‹Ρ… ΠΌΠ°Ρ‚Π΅Ρ€ΠΈΠ°Π»ΠΎΠ² ΠΏΠΎΠ»ΡƒΡ‡Π΅Π½ΠΎ эмпиричСским ΠΏΡƒΡ‚Π΅ΠΌ ΠΈ ΠΌΠΎΠΆΠ΅Ρ‚ Π²Π°Ρ€ΡŒΠΈΡ€ΠΎΠ²Π°Ρ‚ΡŒΡΡ Π² зависимости ΠΎΡ‚ скорости вращСния колСса, Π½Π°Π³Ρ€ΡƒΠ·ΠΊΠΈ Π½Π° колСсо, ΠΌΠ°Ρ‚Π΅Ρ€ΠΈΠ°Π»Π° ΠΎΠΏΠΎΡ€Π½ΠΎΠΉ повСрхности.

Π’ Ρ‚Π°Π±Π»ΠΈΡ†Π΅ Π½ΠΈΠΆΠ΅ ΠΏΡ€ΠΈΠ²Π΅Π΄Π΅Π½Ρ‹ коэффициСнты трСния качСния Π½Π°ΠΈΠ±ΠΎΠ»Π΅Π΅ распространСнных ΠΌΠ°Ρ‚Π΅Ρ€ΠΈΠ°Π»ΠΎΠ², ΠΈΠ· ΠΊΠΎΡ‚ΠΎΡ€Ρ‹Ρ… ΠΈΠ·Π³ΠΎΡ‚Π°Π²Π»ΠΈΠ²Π°ΡŽΡ‚ ΠΏΡ€ΠΎΠΌΡ‹ΡˆΠ»Π΅Π½Π½Ρ‹Π΅ колСса. Β ΠΠ΅ΡƒΠ΄ΠΈΠ²ΠΈΡ‚Π΅Π»ΡŒΠ½ΠΎ, Ρ‡Ρ‚ΠΎ самый мягкий, Π»Π΅Π³ΠΊΠΎ Π΄Π΅Ρ„ΠΎΡ€ΠΌΠΈΡ€ΡƒΡŽΡ‰ΠΈΠΉΡΡ ΠΌΠ°Ρ‚Π΅Ρ€ΠΈΠ°Π» (Ρ€Π΅Π·ΠΈΠ½Π°) ΠΎΠ±Π»Π°Π΄Π°Π΅Ρ‚ самым высоким коэффициСнтом трСния качСния, Π° самый Ρ‚Π²Ρ‘Ρ€Π΄Ρ‹ΠΉ ΠΌΠ°Ρ‚Π΅Ρ€ΠΈΠ°Π» (кованая ΡΡ‚Π°Π»ΡŒ) – самым Π½ΠΈΠ·ΠΊΠΈΠΌ. Β 

ΠœΠ°Ρ‚Π΅Ρ€ΠΈΠ°Π»  шинки (ΠΊΠΎΠ½Ρ‚Π°ΠΊΡ‚Π½ΠΎΠ³ΠΎ слоя колСса)

ΠœΠ°Ρ‚Π΅Ρ€ΠΈΠ°Π» ΠΏΠΎΠ»Π°

ΠšΠΎΡΡ„Ρ„ΠΈΡ†ΠΈΠ΅Π½Ρ‚ трСния качСния

(масса Π³Ρ€ΡƒΠ·Π° – 600 ΠΊΠ³, ΡΠΊΠΎΡ€ΠΎΡΡ‚ΡŒ – 5 ΠΊΠΌ/Ρ‡)

Кованая ΡΡ‚Π°Π»ΡŒ

ΡΡ‚Π°Π»ΡŒ

0.019

Π§ΡƒΠ³ΡƒΠ½

ΡΡ‚Π°Π»ΡŒ

0.021

Ввёрдая Ρ€Π΅Π·ΠΈΠ½Π°

ΡΡ‚Π°Π»ΡŒ

0.303

ΠŸΠΎΠ»ΠΈΡƒΡ€Π΅Ρ‚Π°Π½

ΡΡ‚Π°Π»ΡŒ

0.03–0.057

Π›ΠΈΡ‚ΠΎΠΉ Π½Π΅ΠΉΠ»ΠΎΠ½

ΡΡ‚Π°Π»ΡŒ

0.027

Π€Π΅Π½ΠΎΠ»

ΡΡ‚Π°Π»ΡŒ

0.026

Π€ΠΎΡ€ΠΌΡƒΠ»Π° для расчётов

F = f Ρ… F/R

Β Β FΒ = сила трСния качСния 
Β Β fΒ = коэффициСнт трСния качСния

Β Β WΒ = сила давлСния Π½Π° ΠΎΠΏΠΎΡ€Ρƒ (вСс)
Β Β RΒ = радиус колСса

Из Ρ„ΠΎΡ€ΠΌΡƒΠ»Ρ‹ Π²ΠΈΠ΄Π½ΠΎ, Ρ‡Ρ‚ΠΎ сила трСния качСния F ΠΏΡ€ΠΎΠΏΠΎΡ€Ρ†ΠΈΠΎΠ½Π°Π»ΡŒΠ½Π° силС давлСния Π½Π° ΠΎΠΏΠΎΡ€Ρƒ W ΠΈ ΠΎΠ±Ρ€Π°Ρ‚Π½ΠΎ ΠΏΡ€ΠΎΠΏΠΎΡ€Ρ†ΠΈΠΎΠ½Π°Π»ΡŒΠ½Π° радиусу R колСса. Π’Π°ΠΊΠΈΠΌ ΠΎΠ±Ρ€Π°Π·ΠΎΠΌ, Π΄ΠΈΠ°ΠΌΠ΅Ρ‚Ρ€ колСс ΠΈΠ³Ρ€Π°Π΅Ρ‚ Π²Π°ΠΆΠ½ΡƒΡŽ Ρ€ΠΎΠ»ΡŒ ΠΏΡ€ΠΈ транспортировкС тяТёлых Π³Ρ€ΡƒΠ·ΠΎΠ².Β 

Π£Π·Π½Π°Π² силу трСния качСния ΠΊΠ°ΠΆΠ΄ΠΎΠ³ΠΎ ΠΈ ΡƒΠΌΠ½ΠΎΠΆΠΈΠ² Π΅Π΅ Π½Π° число, ΠΌΠΎΠΆΠ½ΠΎ ΡƒΠ·Π½Π°Ρ‚ΡŒ ΠΏΡ€ΠΈΠΌΠ΅Ρ€Π½ΡƒΡŽ силу сопротивлСния двиТСнию. Однако Π²Ρ‹ΡˆΠ΅ΠΏΡ€ΠΈΠ²Π΅Π΄Π΅Π½Π½Π°Ρ Ρ„ΠΎΡ€ΠΌΡƒΠ»Π° Π½Π΅Ρ‚ΠΎΡ‡Π½Π°, ΠΏΠΎΡ‚ΠΎΠΌΡƒ Ρ‡Ρ‚ΠΎ Π½Π΅ ΡƒΡ‡ΠΈΡ‚Ρ‹Π²Π°Π΅Ρ‚ Π΄Ρ€ΡƒΠ³ΠΈΠ΅ Ρ„Π°ΠΊΡ‚ΠΎΡ€Ρ‹, Π²Π»ΠΈΡΡŽΡ‰ΠΈΠ΅ Π½Π° Π»Ρ‘Π³ΠΊΠΎΡΡ‚ΡŒ качСния (ΠΊ ΠΏΡ€ΠΈΠΌΠ΅Ρ€Ρƒ, силу Π°Π΄Π³Π΅Π·ΠΈΠΈ).Β 

Как Π²Ρ‹Π±Ρ€Π°Ρ‚ΡŒ ΠΏΡ€ΠΎΠΌΡ‹ΡˆΠ»Π΅Π½Π½Ρ‹Π΅ колёса для Π»Ρ‘Π³ΠΊΠΎΠ³ΠΎ пСрСдвиТСния?

Π§Ρ‚ΠΎΠ±Ρ‹ ΡΠ½ΠΈΠ·ΠΈΡ‚ΡŒ сопротивлСниС ΠΊΠ°Ρ‡Π΅Π½ΠΈΡŽ, Π½Π΅ΠΎΠ±Ρ…ΠΎΠ΄ΠΈΠΌΠΎ Π²Ρ‹Π±ΠΈΡ€Π°Ρ‚ΡŒ колёса большого Π΄ΠΈΠ°ΠΌΠ΅Ρ‚Ρ€Π° ΠΈ ΠΈΠ· ΠΌΠ°Ρ‚Π΅Ρ€ΠΈΠ°Π»ΠΎΠ² с Π½ΠΈΠ·ΠΊΠΈΠΌ коэффициСнтом трСния.Β 
Π’Ρ‹Π±ΠΎΡ€ подшипников Π½Π΅ ΡΡ‚ΠΎΠ»ΡŒ ΠΊΡ€ΠΈΡ‚ΠΈΡ‡Π΅Π½ для лёгкости Ρ…ΠΎΠ΄Π° Ρ‚Π΅Π»Π΅ΠΆΠΊΠΈ, ΠΊΠ°ΠΊ Π΄ΠΈΠ°ΠΌΠ΅Ρ‚Ρ€ ΠΈ ΠΌΠ°Ρ‚Π΅Ρ€ΠΈΠ°Π» шинки. ΠŸΠΎΠ½ΡΡ‚Π½ΠΎ, Ρ‡Ρ‚ΠΎ подшипники качСния ΠΏΡ€Π΅Π΄ΠΏΠΎΡ‡Ρ‚ΠΈΡ‚Π΅Π»ΡŒΠ½Π΅Π΅ подшипников скольТСния. Π’Π°ΠΊΠΆΠ΅ стоит ΡƒΡ‡ΠΈΡ‚Ρ‹Π²Π°Ρ‚ΡŒ, Ρ‡Ρ‚ΠΎ ΡˆΠ°Ρ€ΠΈΠΊΠΎΠ²Ρ‹Π΅ ΠΈ Ρ€ΠΎΠ»ΠΈΠΊΠΎΠ²Ρ‹Π΅ подшипники Π»ΡƒΡ‡ΡˆΠ΅ Π²Ρ‹Π΄Π΅Ρ€ΠΆΠΈΠ²Π°ΡŽΡ‚ Π½Π°Π³Ρ€ΡƒΠ·ΠΊΠΈ, мСньшС ΠΈΠ·Π½Π°ΡˆΠΈΠ²Π°ΡŽΡ‚ΡΡ ΠΈ дольшС слуТат.Β 

Π“Π»Π°Π²Π½Ρ‹Π΅ Ρ„Π°ΠΊΡ‚ΠΎΡ€Ρ‹, Π²Π»ΠΈΡΡŽΡ‰ΠΈΠ΅ Π½Π° сопротивлСниС ΠΊΠ°Ρ‡Π΅Π½ΠΈΡŽ:

  • Β Β   масса;
  • Β Β  Β Π΄ΠΈΠ°ΠΌΠ΅Ρ‚Ρ€ колСс;
  • Β Β  Β ΠΌΠ°Ρ‚Π΅Ρ€ΠΈΠ°Π» ΠΈ ΠΌΡΠ³ΠΊΠΎΡΡ‚ΡŒ шинки;
  • Β Β  Β ΠΌΠ°Ρ‚Π΅Ρ€ΠΈΠ°Π» ΠΈ качСство повСрхности ΠΏΠΎΠ»Π°;
  • Β Β   условия Π½Π° ΠΏΠΎΠ»Ρƒ (Π³Ρ€ΡƒΠ±ΠΎΡΡ‚ΡŒ повСрхности, чистота, Π½Π°ΠΊΠ»ΠΎΠ½ ΠΈ Ρ‚.Π΄.).

Π€Π°ΠΊΡ‚ΠΎΡ€Ρ‹, ΠΊΠΎΡ‚ΠΎΡ€Ρ‹Π΅ ΠΎΠ±Ρ‹Ρ‡Π½ΠΎ ΠΈΠ³Π½ΠΎΡ€ΠΈΡ€ΡƒΡŽΡ‚:

  • Β Β  Β Ρ‚ΠΈΠΏ подшипников;
  • Β Β   рисунок ΠΏΡ€ΠΎΡ‚Π΅ΠΊΡ‚ΠΎΡ€Π°;
  • Β Β   эффСкт скольТСния ΠΈΠ»ΠΈ Π°Π΄Π³Π΅Π·ΠΈΠΈ;
  • Β Β  Β Ρ‚Π΅ΠΌΠΏΠ΅Ρ€Π°Ρ‚ΡƒΡ€Π° ΠΎΠΊΡ€ΡƒΠΆΠ°ΡŽΡ‰Π΅ΠΉ срСды;
  • Β Β  Β ΡƒΠΊΠ»ΠΎΠ½Ρ‹ повСрхности.

Β  Β  ΠžΠ±Ρ‰ΠΈΠ΅ Ρ€Π΅ΠΊΠΎΠΌΠ΅Π½Π΄Π°Ρ†ΠΈΠΈ:

  1. ΠŸΠΎΠΊΡƒΠΏΠ°ΠΉΡ‚Π΅Β ΠΊΠΎΠ»Π΅ΡΠΎ ΠΏΡ€ΠΎΠΌΡ‹ΡˆΠ»Π΅Π½Π½ΠΎΠ΅, ΠΎΡΠ½ΠΎΠ²Ρ‹Π²Π°ΡΡΡŒ Π½Π° Π³Ρ€ΡƒΠ·ΠΎΠΏΠΎΠ΄ΡŠΡ‘ΠΌΠ½ΠΎΡΡ‚ΠΈ ΠΈ состоянии ΠΏΠΎΠ»ΠΎΠ².
  2. Π”ΠΎΠΏΠΎΠ»Π½ΠΈΡ‚Π΅Π»ΡŒΠ½ΠΎ ΠΏΡ€ΠΈΠ½ΠΈΠΌΠ°ΠΉΡ‚Π΅ Π²ΠΎ Π²Π½ΠΈΠΌΠ°Π½ΠΈΠ΅: Π΄ΠΈΠ°ΠΏΠ°Π·ΠΎΠ½ Ρ‚Π΅ΠΌΠΏΠ΅Ρ€Π°Ρ‚ΡƒΡ€, ΡƒΠ΄Π°Ρ€ΠΎΠΏΡ€ΠΎΡ‡Π½ΠΎΡΡ‚ΡŒ, ΡƒΡΡ‚ΠΎΠΉΡ‡ΠΈΠ²ΠΎΡΡ‚ΡŒ ΠΊ Π²Π»Π°Π³Π΅, ΡΡ‚ΠΎΠΉΠΊΠΎΡΡ‚ΡŒ ΠΊ свСту ΠΈ Ρ…ΠΈΠΌΠΈΠΊΠ°Ρ‚Π°ΠΌ, Π²ΠΎΠ·ΠΌΠΎΠΆΠ½ΠΎΡΡ‚ΡŒ восстановлСния.
  3. Π’Ρ‹Π±ΠΈΡ€Π°ΠΉΡ‚Π΅ максимально большой ΠΈΠ· Π²ΠΎΠ·ΠΌΠΎΠΆΠ½Ρ‹Ρ… Π΄ΠΈΠ°ΠΌΠ΅Ρ‚Ρ€ΠΎΠ².
  4. ΠžΡΡ‚Π°Π½ΠΎΠ²ΠΈΡ‚Π΅ Π²Ρ‹Π±ΠΎΡ€ Π½Π° шинкС с ΠΌΠΈΠ½ΠΈΠΌΠ°Π»ΡŒΠ½Ρ‹ΠΌ сопротивлСниСм ΠΊΠ°Ρ‡Π΅Π½ΠΈΡŽ.
  5. ΠŸΠΎΠ΄ΡΡ‡ΠΈΡ‚Π°ΠΉΡ‚Π΅ силу сопротивлСния ΠΊΠ°Ρ‡Π΅Π½ΠΈΡŽ, принимая Π²ΠΎ Π²Π½ΠΈΠΌΠ°Π½ΠΈΠ΅ Π²Π΅Π»ΠΈΡ‡ΠΈΠ½Ρƒ «стартовой» силы.
  6. Π£Ρ‡ΠΈΡ‚Ρ‹Π²Π°ΠΉΡ‚Π΅ Ρ„Π°ΠΊΡ‚ΠΎΡ€ бСзопасности.
  7. ΠŸΠΎΠΌΠ½ΠΈΡ‚Π΅ ΠΏΡ€ΠΎ ΡƒΠΊΠ»ΠΎΠ½Ρ‹ повСрхности. Π‘ΠΎΠΏΡ€ΠΎΡ‚ΠΈΠ²Π»Π΅Π½ΠΈΠ΅ ΠΊΠ°Ρ‡Π΅Π½ΠΈΡŽ возрастаСт Π½Π° ΠΏΠΎΠ΄ΡŠΡ‘ΠΌΠ°Ρ… ΠΈ сниТаСтся Π½Π° спусках. F = Fx/cosa.Β 
  8. Для буксировки самоходным транспортом Π»ΡƒΡ‡ΡˆΠ΅ Π²Ρ‹Π±ΠΈΡ€Π°Ρ‚ΡŒ ΠΏΡ€ΠΎΠΌΡ‹ΡˆΠ»Π΅Π½Π½Ρ‹Π΅ колёса с ΡˆΠ°Ρ€ΠΈΠΊΠΎΠ²Ρ‹ΠΌΠΈ подшипниками Π² оси. Волько ΠΎΠ½ΠΈ обСспСчат большой ΠΏΡ€ΠΎΠ±Π΅Π³, Π²Ρ‹Π΄Π΅Ρ€ΠΆΠ°Ρ‚ высокиС скорости ΠΈ Π½Π°Π³Ρ€ΡƒΠ·ΠΊΠΈ.

Π›Π΅Π³ΠΊΠΎΠΉ Π²Π°ΠΌ Ρ€Π°Π±ΠΎΡ‚Ρ‹!

Π—Π°Π΄Π°Ρ‡Π° ΠΏΠΎ Ρ„ΠΈΠ·ΠΈΠΊΠ΅. Надо Π½Π°ΠΉΡ‚ΠΈ силу трСния зная массу, силу тяги

Π’Π΅Π»ΠΎ Π±Ρ‹Π»ΠΎ Π²Ρ‹ΠΏΡƒΡ‰Π΅Π½ΠΎ Π²Π΅Ρ€Ρ‚ΠΈΠΊΠ°Π»ΡŒΠ½ΠΎ Π²Π²Π΅Ρ€Ρ… со ΡΠΊΠΎΡ€ΠΎΡΡ‚ΡŒΡŽ 30 ΠΌ / с.2?​

Ρ€Π°ΡΠΏΠΈΡˆΠΈΡ‚Π΅ поТалуйста Ρ€Π΅ΡˆΠ΅Π½ΠΈΠ΅, ΠΎΡ‡Π΅Π½ΡŒ Π½ΡƒΠΆΠ½ΠΎ

Бколько элСктронов дСйствуСт Π½Π° заряд , ΠΏΠΎΠΌΠ΅Ρ‰Ρ‘Π½Π½Ρ‹Ρ… Π² ΠΎΠ΄Π½ΠΎΡ€ΠΎΠ΄Π½ΠΎΠ΅ ΠΏΠΎΠ»Π΅ Π½Π°ΠΏΡ€ΡΠΆΡ‘Π½Π½ΠΎΡΡ‚ΡŒΡŽ 20000 Н/Кл ΠΏΠΎ дСйствиСм силы 0.32 нН (ΠžΡ‚Π²Π΅Ρ‚ Π²Ρ‹Ρ€Π°Π·ΠΈΡ‚Π΅ Π² стСпСни 5)* … ​ А)1 Π’)8 Π‘)32 Π’)16

Π”Π°ΡŽ 50 Π±Π°Π»ΠΎΠ²!!!! Π‘Ρ€ΠΎΡ‡Π½ΠΎ!!!!2. ΠŸΠΎΡ€ΠΈΡΡ‚Π΅ Ρ‚Ρ–Π»ΠΎ Ρ‚Π²Π°Ρ€ΠΈΠ½Π½ΠΎΠ³ΠΎ походТСння, якС ΠΌΡ–ΠΆ Π²ΠΎΠ»ΠΎΠΊΠ½Π°ΠΌΠΈ ΠΌΡ–ΡΡ‚ΠΈΡ‚ΡŒ повітря, Π° Ρ‚ΠΎΠΌΡƒ мас ΠΏΠΎΠ³Π°Π½Ρƒ Ρ‚Π΅ΠΏΠ»ΠΎΠΏΡ€ΠΎΠ²Ρ–Π΄Π½Ρ–ΡΡ‚ΡŒ. 4. Π’Π²Π΅Ρ€Π΄Π° Ρ€Π΅Ρ‡ … ΠΎΠ²ΠΈΠ½Π°, Ρ‰ΠΎ ΠΌΠ°Ρ” ΠΏΠΈΡ‚ΠΎΠΌΡƒ Ρ‚Π΅ΠΏΠ»ΠΎΡ‚Ρƒ плавлСння 0,59-10 Π”ΠΆ/ΠΊΠ³ 5. Π’Π²Π΅Ρ€Π΄Π° Ρ€Π΅Ρ‡ΠΎΠ²ΠΈΠ½Π°, Ρ‰ΠΎ ΠΌΠ°Π΅ Ρ‚Π΅ΠΌΠΏΠ΅Ρ€Π°Ρ‚ΡƒΡ€Ρƒ плавлСння 1200Β° Π‘ 11. Наочний спосіб ΠΏΠΎΠΊΠ°Π·Ρƒ залСТності ΠΌΡ–ΠΆ Π΄Π²ΠΎΠΌΠ° Ρ„Ρ–Π·ΠΈΡ‡Π½ΠΈΠΌΠΈ Π²Π΅Π»ΠΈΠ½ΠΈΠ½Π°ΠΌΠΈ 12 Π—Π°Π³Π°Π»ΡŒΠ½Π° Π½Π°Π·Π²Π° машин, Ρ‰ΠΎ ΠΏΠ΅Ρ€Π΅Ρ‚Π²ΠΎΡ€ΡŽΡŽΡ‚ΡŒ ΠΏΠ΅Π²Π½ΠΈΠΉ Π²ΠΈΠ΄ Π΅Π½Π΅Ρ€Π³Ρ–Ρ— Π² ΠΌΠ΅Ρ…Π°Π½Ρ–Ρ‡Π½Ρƒ Π΅Π½Π΅Ρ€Π³Ρ–ΡŽ 13. Бпосіб Π·ΠΌiΠ½ΠΈ Π²Π½ΡƒΡ‚Ρ€Ρ–ΡˆΠ½ΡŒΠΎΡ— Π΅Π½Π΅Ρ€Π³Ρ–Ρ— Ρ‚iΠ»Π° Π±Π΅Π· виконання Ρ€ΠΎΠ±ΠΎΡ‚ΠΈ Π½Π°Π΄ Ρ‚Ρ–Π»ΠΎΠΌ Π°Π±ΠΎ самим 16. ΠŸΡ€ΠΈΠ»Π°Π΄, Ρ‰ΠΎ Π²ΠΈΠΊΠΎΡ€ΠΈΡΡ‚ΠΎΠ²ΡƒΡŽΡ‚ΡŒ Ρƒ Π±Π°Π³Π°Ρ‚ΡŒΠΎΡ… дослідах ΠΏΡ€ΠΈ Π²ΠΈΠ²Ρ‡Π΅Π½Π½Ρ– Ρ‚Π΅ΠΏΠ»ΠΎΠ²ΠΈΡ… явищ, Ρ‚iΠ»ΠΎΠΌ основна частина якого-Π΄Π²Ρ– посудини Ρ€Ρ–Π·Π½ΠΈΡ… Ρ€ΠΎΠ·ΠΌΡ–Ρ€Ρ–Π² 17. Паливо, Ρ‰ΠΎ ΠΌΠ°Π΅ ΠΏΠΈΡ‚ΠΎΠΌΡƒ Ρ‚Π΅ΠΏΠ»ΠΎΡ‚Ρƒ згорання 4,6-10 Π”ΠΆ/ΠΊΠ³ 18. Π―Π²ΠΈΡ‰Π΅ пСрСтворСння Ρ€Ρ–Π΄ΠΈΠ½ΠΈ Π² Ρ‚Π²Π΅Ρ€Π΄ΠΈΠΉ стан 19. ΠšΡ€ΠΈΡΡ‚Π°Π»Ρ–Ρ‡Π½Π΅ Ρ‚Π²Π΅Ρ€Π΄Π΅ Ρ‚Ρ–Π»ΠΎ, Ρ‰ΠΎ ΠΌΠ°Ρ” Ρ‚Π΅ΠΌΠΏΠ΅Ρ€Π°Ρ‚ΡƒΡ€Ρƒ кристалізації 0Β°Π‘ 20. МалСнька частинка Ρ‚Π²Π΅Ρ€Π΄ΠΎΠ³ΠΎ Ρ‚Ρ–Π»Π°, Ρ‰ΠΎ ΠΌΠ°Ρ” ΠΏΡ€Π°Π²ΠΈΠ»ΡŒΠ½Ρƒ Π³Π΅ΠΎΠΌΠ΅Ρ‚Ρ€ΠΈΡ‡Π½Ρƒ Ρ„ΠΎΡ€ΠΌΡƒ 21. ΠœΠ΅Ρ‚Π°Π», Ρ‰ΠΎ ΠΌΠ°Ρ” ΠΏΠΈΡ‚ΠΎΠΌΡƒ Ρ‚Π΅ΠΏΠ»ΠΎΡ”ΠΌΠ½Ρ–ΡΡ‚ΡŒ 140 Π”ΠΆ/ΠΊΠ³ К Ρ– Ρ‚Π΅ΠΌΠΏΠ΅Ρ€Π°Ρ‚ΡƒΡ€Ρƒ плавлСння 327°Б​

Π˜ΡΡ‚ΠΎΡ‡Π½ΠΈΠΊ напряТСния Π² Π²ΠΈΠ΄Π΅ аккумулятора ΠΈΠΌΠ΅Π΅Ρ‚ напряТСниС Π±Π΅Π· Π½Π°Π³Ρ€ΡƒΠ·ΠΊΠΈ 4,2 Π’. КакоС Π²Π½ΡƒΡ‚Ρ€Π΅Π½Π½Π΅Π΅ сопротивлСниС напряТСниС ΠΈΠΌΠ΅Π΅Ρ‚ этот аккумулятор, Ссли ΠΏΡ€ … ΠΈ ΠΏΠΎΠ΄ΠΊΠ»ΡŽΡ‡Π΅Π½ΠΈΠΈ Π½Π°Π³Ρ€ΡƒΠ·ΠΊΠΈ, ΠΏΠΎΡ‚Ρ€Π΅Π±Π»ΡΡŽΡ‰Π΅ΠΉ Ρ‚ΠΎΠΊ 0,5 А, напряТСниС Π½Π° аккумуляторС составляСт 4 Π’?

Π— Π΄Π½Π° ΠΎΠ·Π΅Ρ€Π° ΠΏΡ–Π΄Π½Ρ–ΠΌΠ°ΡŽΡ‚ΡŒ Π±Π΅Ρ‚ΠΎΠ½Π½Ρƒ Π±Π°Π»ΠΊΡƒ Ρ€ΠΎΠ·ΠΌΡ–Ρ€Π°ΠΌΠΈ : Π΄ΠΎΠ²ΠΆΠΈΠ½Π° 40см, висота 50 см, ΡˆΠΈΡ€ΠΈΠ½Π° 75 см. Π―ΠΊΡƒ ΠΌΡ–Π½Ρ–ΠΌΠ°Π»ΡŒΠ½Ρƒ Ρ€ΠΎΠ±ΠΎΡ‚Ρƒ ΠΏΠΎΡ‚Ρ€Ρ–Π±Π½ΠΎ ΠΏΡ€ΠΈ Ρ†ΡŒΠΎΠΌΡƒ Π²ΠΈΠΊΠΎΠ½Π°Ρ‚ΠΈ, якщо Π³Π»ΠΈ … Π±ΠΈΠ½Π° ΠΎΠ·Π΅Ρ€Π° Π΄ΠΎΡ€Ρ–Π²Π½ΡŽΡ” 12 ΠΌ? ΠžΠΏΠΎΡ€ΠΎΠΌ Π²ΠΎΠ΄ΠΈ Π·Π½Π΅Ρ…Ρ‚ΡƒΠΉΡ‚Π΅. ΠŸΠΎΠΆΠ°Π»ΡƒΠΉΡΡ‚Π°!!! Π‘Ρ€ΠΎΡ‡Π½ΠΎ!!!​

ΠšΠ°Π»ΡŒΠΊΡƒΠ»ΡΡ‚ΠΎΡ€ трСния

Π˜ΡΠΏΠΎΠ»ΡŒΠ·ΡƒΠΉΡ‚Π΅ этот ΠΊΠ°Π»ΡŒΠΊΡƒΠ»ΡΡ‚ΠΎΡ€ трСния для расчСта силы трСния ΠΌΠ΅ΠΆΠ΄Ρƒ ΠΎΠ±ΡŠΠ΅ΠΊΡ‚ΠΎΠΌ ΠΈ Π·Π΅ΠΌΠ»Π΅ΠΉ. Он основан Π½Π° простом ΠΏΡ€ΠΈΠ½Ρ†ΠΈΠΏΠ΅: Ρ‚Ρ€Π΅Π½ΠΈΠ΅ ΠΏΡ€ΠΎΠΏΠΎΡ€Ρ†ΠΈΠΎΠ½Π°Π»ΡŒΠ½ΠΎ Π½ΠΎΡ€ΠΌΠ°Π»ΡŒΠ½ΠΎΠΉ силС, Π΄Π΅ΠΉΡΡ‚Π²ΡƒΡŽΡ‰Π΅ΠΉ ΠΌΠ΅ΠΆΠ΄Ρƒ ΠΎΠ±ΡŠΠ΅ΠΊΡ‚ΠΎΠΌ ΠΈ Π·Π΅ΠΌΠ»Π΅ΠΉ. ΠŸΡ€ΠΎΡ‡Ρ‚ΠΈΡ‚Π΅, Ρ‡Ρ‚ΠΎΠ±Ρ‹ ΡƒΠ·Π½Π°Ρ‚ΡŒ, ΠΊΠ°ΠΊ ΠΏΡ€ΠΈΠΌΠ΅Π½ΠΈΡ‚ΡŒ ΡƒΡ€Π°Π²Π½Π΅Π½ΠΈΠ΅ силы трСния ΠΈ Ρ€Π°Π·Π½ΠΈΡ†Ρƒ ΠΌΠ΅ΠΆΠ΄Ρƒ статичСским ΠΈ кинСтичСским Ρ‚Ρ€Π΅Π½ΠΈΠ΅ΠΌ. Π’Ρ‹ Ρ‚Π°ΠΊΠΆΠ΅ ΠΌΠΎΠΆΠ΅Ρ‚Π΅ ΠΈΡΠΏΠΎΠ»ΡŒΠ·ΠΎΠ²Π°Ρ‚ΡŒ этот инструмСнт Π² качСствС ΠΊΠ°Π»ΡŒΠΊΡƒΠ»ΡΡ‚ΠΎΡ€Π° коэффициСнта трСния.

Π£Ρ€Π°Π²Π½Π΅Π½ΠΈΠ΅ силы трСния

Π€ΠΎΡ€ΠΌΡƒΠ»Π°, ΠΏΠΎΠ·Π²ΠΎΠ»ΡΡŽΡ‰Π°Ρ Ρ€Π°ΡΡΡ‡ΠΈΡ‚Π°Ρ‚ΡŒ силу трСния, ΠΎΡ‡Π΅Π½ΡŒ проста:

F = мкН

Π³Π΄Π΅:

  • F — сила трСния, измСряСмая Π² ΠΡŒΡŽΡ‚ΠΎΠ½Π°Ρ…;
  • ΞΌ — Π±Π΅Π·Ρ€Π°Π·ΠΌΠ΅Ρ€Π½Ρ‹ΠΉ коэффициСнт трСния; ΠΈ
  • Н — Π½ΠΎΡ€ΠΌΠ°Π»ΡŒΠ½Π°Ρ сила (пСрпСндикулярная повСрхности Π·Π΅ΠΌΠ»ΠΈ), выраТСнная Π² ΠΡŒΡŽΡ‚ΠΎΠ½Π°Ρ….

БтатичСскоС Ρ‚Ρ€Π΅Π½ΠΈΠ΅ ΠΈ кинСтичСскоС Ρ‚Ρ€Π΅Π½ΠΈΠ΅

БтатичСскоС Ρ‚Ρ€Π΅Π½ΠΈΠ΅ дСйствуСт, ΠΊΠΎΠ³Π΄Π° ΠΎΠ±ΡŠΠ΅ΠΊΡ‚ остаСтся Π½Π΅ΠΏΠΎΠ΄Π²ΠΈΠΆΠ½Ρ‹ΠΌ. ΠŸΡ€Π΅Π΄ΡΡ‚Π°Π²ΡŒΡ‚Π΅, Ρ‡Ρ‚ΠΎ Π²Ρ‹ ΠΏΡ‹Ρ‚Π°Π΅Ρ‚Π΅ΡΡŒ Π²Ρ‹Ρ‚Π°Ρ‰ΠΈΡ‚ΡŒ тяТСлый ящик. Если ΠΌΡ‹ Π½Π΅ ΠΏΡ€ΠΈΠ½ΠΈΠΌΠ°Π΅ΠΌ Π²ΠΎ Π²Π½ΠΈΠΌΠ°Π½ΠΈΠ΅ Ρ‚Ρ€Π΅Π½ΠΈΠ΅, Π΄Π°ΠΆΠ΅ самая малСнькая сила Π΄ΠΎΠ»ΠΆΠ½Π° Π²Ρ‹Π·Ρ‹Π²Π°Ρ‚ΡŒ Π½Π΅ΠΊΠΎΡ‚ΠΎΡ€ΠΎΠ΅ ускорСниС ΠΊΠΎΡ€ΠΎΠ±ΠΊΠΈ согласно Π²Ρ‚ΠΎΡ€ΠΎΠΌΡƒ Π·Π°ΠΊΠΎΠ½Ρƒ ΠΡŒΡŽΡ‚ΠΎΠ½Π°. На самом Π΄Π΅Π»Π΅ Π²Π°ΠΌ Π½ΡƒΠΆΠ½ΠΎ довольно сильно Ρ‚ΡΠ½ΡƒΡ‚ΡŒ, Ρ‡Ρ‚ΠΎΠ±Ρ‹ ΠΊΠΎΡ€ΠΎΠ±ΠΊΠ° Π½Π°Ρ‡Π°Π»Π° Π΄Π²ΠΈΠ³Π°Ρ‚ΡŒΡΡ ΠΈΠ·-Π·Π° силы статичСского трСния.

ΠšΠΈΠ½Π΅Ρ‚ΠΈΡ‡Π΅ΡΠΊΠΎΠ΅ Ρ‚Ρ€Π΅Π½ΠΈΠ΅ дСйствуСт Π½Π° двиТущийся ΠΎΠ±ΡŠΠ΅ΠΊΡ‚ ΠΈΠ»ΠΈ, Π΄Ρ€ΡƒΠ³ΠΈΠΌΠΈ словами, Π½Π° ΠΎΠ±ΡŠΠ΅ΠΊΡ‚ с Π½Π΅Π½ΡƒΠ»Π΅Π²ΠΎΠΉ кинСтичСской энСргиСй.Если Π±Ρ‹ Π½Π΅ Π±Ρ‹Π»ΠΎ кинСтичСского трСния, любой ΠΎΠ±ΡŠΠ΅ΠΊΡ‚, ΠΊΠΎΡ‚ΠΎΡ€Ρ‹ΠΉ Π²Ρ‹ Ρ‚ΠΎΠ»ΠΊΠ°Π΅Ρ‚Π΅ (Π½Π°ΠΏΡ€ΠΈΠΌΠ΅Ρ€, ΠΈΠ³Ρ€ΡƒΡˆΠ΅Ρ‡Π½Π°Ρ машинка), Π½ΠΈΠΊΠΎΠ³Π΄Π° Π½Π΅ пСрСставал Π±Ρ‹ Π΄Π²ΠΈΠ³Π°Ρ‚ΡŒΡΡ, ΠΏΠΎΡΠΊΠΎΠ»ΡŒΠΊΡƒ, согласно ΠΏΠ΅Ρ€Π²ΠΎΠΌΡƒ Π·Π°ΠΊΠΎΠ½Ρƒ ΠΡŒΡŽΡ‚ΠΎΠ½Π°, Π½Π° Π½Π΅Π³ΠΎ Π½Π΅ дСйствовала Π±Ρ‹ никакая сила, поэтому ΠΎΠ½ ΠΏΡ€ΠΎΠ΄ΠΎΠ»ΠΆΠ°Π» Π±Ρ‹ Π΄Π²ΠΈΠ³Π°Ρ‚ΡŒΡΡ с постоянной ΡΠΊΠΎΡ€ΠΎΡΡ‚ΡŒΡŽ. .

НСсмотря Π½Π° Ρ‚ΠΎ, Ρ‡Ρ‚ΠΎ Ρ„ΠΎΡ€ΠΌΡƒΠ»Ρ‹ для статичСского ΠΈ кинСтичСского трСния ΠΎΠ΄ΠΈΠ½Π°ΠΊΠΎΠ²Ρ‹, Π²Ρ‹ Π΄ΠΎΠ»ΠΆΠ½Ρ‹ ΠΏΠΎΠΌΠ½ΠΈΡ‚ΡŒ, Ρ‡Ρ‚ΠΎ коэффициСнты трСния Ρ€Π°Π·Π½Ρ‹Π΅. ΠšΠΎΡΡ„Ρ„ΠΈΡ†ΠΈΠ΅Π½Ρ‚ кинСтичСского трСния ΠΎΠ±Ρ‹Ρ‡Π½ΠΎ Π½ΠΈΠΆΠ΅, Ρ‡Π΅ΠΌ коэффициСнт трСния покоя.

Как ΠΈΠ·ΠΌΠ΅Ρ€ΠΈΡ‚ΡŒ коэффициСнт трСния?

БущСствуСт Π΄Π²Π° простых ΠΌΠ΅Ρ‚ΠΎΠ΄Π° ΠΎΡ†Π΅Π½ΠΊΠΈ коэффициСнта трСния : ΠΏΡƒΡ‚Π΅ΠΌ измСрСния ΡƒΠ³Π»Π° пСрСмСщСния ΠΈ с ΠΏΠΎΠΌΠΎΡ‰ΡŒΡŽ Π΄Π°Ρ‚Ρ‡ΠΈΠΊΠ° силы.ΠšΠΎΡΡ„Ρ„ΠΈΡ†ΠΈΠ΅Π½Ρ‚ трСния Ρ€Π°Π²Π΅Π½ tan (ΞΈ), Π³Π΄Π΅ ΞΈ — ΡƒΠ³ΠΎΠ» ΠΎΡ‚ Π³ΠΎΡ€ΠΈΠ·ΠΎΠ½Ρ‚Π°Π»ΠΈ, ΠΏΡ€ΠΈ ΠΊΠΎΡ‚ΠΎΡ€ΠΎΠΌ ΠΎΠ±ΡŠΠ΅ΠΊΡ‚, ΠΏΠΎΠΌΠ΅Ρ‰Π΅Π½Π½Ρ‹ΠΉ ΠΏΠΎΠ²Π΅Ρ€Ρ… Π΄Ρ€ΡƒΠ³ΠΎΠ³ΠΎ , Π½Π°Ρ‡ΠΈΠ½Π°Π΅Ρ‚ ΠΏΠ΅Ρ€Π΅ΠΌΠ΅Ρ‰Π°Ρ‚ΡŒΡΡ Π½Π° . Для плоской повСрхности Π²Ρ‹ ΠΌΠΎΠΆΠ΅Ρ‚Π΅ ΠΏΠΎΡ‚ΡΠ½ΡƒΡ‚ΡŒ ΠΎΠ±ΡŠΠ΅ΠΊΡ‚ ΠΏΠΎ повСрхности с ΠΏΠΎΠΌΠΎΡ‰ΡŒΡŽ измСритСля силы . Π Π°Π·Π΄Π΅Π»ΠΈΡ‚Π΅ Π½ΡŒΡŽΡ‚ΠΎΠ½Ρ‹, Π½Π΅ΠΎΠ±Ρ…ΠΎΠ΄ΠΈΠΌΡ‹Π΅ для пСрСмСщСния ΠΎΠ±ΡŠΠ΅ΠΊΡ‚Π°, Π½Π° массу ΠΎΠ±ΡŠΠ΅ΠΊΡ‚Π°, Ρ‡Ρ‚ΠΎΠ±Ρ‹ ΠΏΠΎΠ»ΡƒΡ‡ΠΈΡ‚ΡŒ коэффициСнт трСния.

Π§Ρ‚ΠΎ ΡΠ»ΡƒΡ‡ΠΈΠ»ΠΎΡΡŒ Π±Ρ‹ Π² ΠΌΠΈΡ€Π΅ Π±Π΅Π· трСния?

Если Π±Ρ‹ Π½Π΅ Π±Ρ‹Π»ΠΎ трСния, Π½Π΅ Π±Ρ‹Π»ΠΎ Π±Ρ‹ Π½ΠΈΡ‡Π΅Π³ΠΎ .Π’Ρ€Π΅Π½ΠΈΠ΅ , ΠΊΠΎΡ‚ΠΎΡ€ΠΎΠ΅ ΡƒΠ΄Π΅Ρ€ΠΆΠΈΠ²Π°Π΅Ρ‚ Π°Ρ‚ΠΎΠΌΡ‹ вмСстС, исчСзло Π±Ρ‹, , Ρ‚Π°ΠΊ Ρ‡Ρ‚ΠΎ Π½ΠΈΡ‡Ρ‚ΠΎ Π½Π΅ ΠΌΠΎΠ³Π»ΠΎ Π±Ρ‹ ΠΎΠ±Ρ€Π°Π·ΠΎΠ²Π°Ρ‚ΡŒΡΡ. Π–ΠΈΠ·Π½ΠΈ Π½Π΅ сущСствовало Π±Ρ‹, ΠΏΠΎΡΠΊΠΎΠ»ΡŒΠΊΡƒ Π°Ρ‚ΠΎΠΌΡ‹ Π½Π΅ ΠΌΠΎΠ³Π»ΠΈ Π±Ρ‹ Π½Π°Ρ…ΠΎΠ΄ΠΈΡ‚ΡŒΡΡ рядом Π΄Ρ€ΡƒΠ³ с Π΄Ρ€ΡƒΠ³ΠΎΠΌ достаточно Π΄ΠΎΠ»Π³ΠΎ, Ρ‡Ρ‚ΠΎΠ±Ρ‹ ΠΎΠ±Ρ€Π°Π·ΠΎΠ²Π°Ρ‚ΡŒ простыС ΠΌΠΎΠ»Π΅ΠΊΡƒΠ»Ρ‹. ΠœΠΈΡ€ станСт опасным мСстом для ΠΆΠΈΠ·Π½ΠΈ, ΠΏΠΎΡΠΊΠΎΠ»ΡŒΠΊΡƒ двиТущиСся ΠΌΠ°ΡˆΠΈΠ½Ρ‹ ΠΏΠΎΡ‚Π΅Ρ€ΡΡŽΡ‚ ΡΠΏΠΎΡΠΎΠ±Π½ΠΎΡΡ‚ΡŒ ΠΎΡΡ‚Π°Π½Π°Π²Π»ΠΈΠ²Π°Ρ‚ΡŒΡΡ. Π’Ρ‹, вСроятно, Ρ‚ΠΎΠΆΠ΅ ΡƒΠΌΡ€Π΅Ρ‚Π΅, ΠΏΠΎΡΠΊΠΎΠ»ΡŒΠΊΡƒ ваша ΠΊΡ€ΠΎΠ²ΡŒ Π±ΡƒΠ΄Π΅Ρ‚ постСпСнно ΠΏΠ΅Ρ€Π΅ΠΌΠ΅Ρ‰Π°Ρ‚ΡŒΡΡ всС быстрСС ΠΈ быстрСС . Π₯ΠΎΡ€ΠΎΡˆΠΎ, Ρ‡Ρ‚ΠΎ Ρ‚Π°ΠΊΠΎΠΉ сцСнарий физичСски Π½Π΅Π²ΠΎΠ·ΠΌΠΎΠΆΠ΅Π½!

Как Ρ€Π°ΡΡΡ‡ΠΈΡ‚Π°Ρ‚ΡŒ ΠΏΠΎΡ‚Π΅Ρ€ΡŽ энСргии Π½Π° Ρ‚Ρ€Π΅Π½ΠΈΠ΅?

ЭнСргия, потСрянная ΠΈΠ·-Π·Π° трСния, составляСт , равная Ρ€Π°Π±ΠΎΡ‚Π΅, ΡΠΎΠ²Π΅Ρ€ΡˆΠ°Π΅ΠΌΠΎΠΉ Ρ‚Ρ€Π΅Π½ΠΈΠ΅ΠΌ , ΠΈΠ»ΠΈ F Ρ‚Ρ€Π΅Π½ΠΈΠ΅ d , Π³Π΄Π΅ d — это ΠΏΡ€ΠΎΠΉΠ΄Π΅Π½Π½ΠΎΠ΅ расстояниС. F fric ΠΌΠΎΠΆΠ½ΠΎ Ρ€Π°Π·Π±ΠΈΡ‚ΡŒ Π½Π° ΞΌ k F N , Π³Π΄Π΅ ΞΌ k — коэффициСнт трСния, Π° F N — Π½ΠΎΡ€ΠΌΠ°Π»ΡŒΠ½Π°Ρ сила. F N ΠΌΠΎΠΆΠ½ΠΎ Π΄Π°Π»Π΅Π΅ Ρ€Π°Π·Π±ΠΈΡ‚ΡŒ Π½Π° mgcos (ΞΈ) . Π˜Ρ‚Π°ΠΊ, Π² ΠΈΡ‚ΠΎΠ³Π΅ E = ΞΌ k (mgcos (ΞΈ)) d .

КакиС Π±Ρ‹Π²Π°ΡŽΡ‚ 4 Ρ‚ΠΈΠΏΠ° трСния?

Π§Π΅Ρ‚Ρ‹Ρ€Π΅ Ρ‚ΠΈΠΏΠ° трСния: статичСскоС, скольТСниС, ΠΊΠ°Ρ‡Π΅Π½ΠΈΠ΅ ΠΈ тСкучая срСда . БтатичСский находится ΠΌΠ΅ΠΆΠ΄Ρƒ двумя повСрхностями, ΠΊΠΎΠ³Π΄Π° Π½ΠΈ ΠΎΠ΄Π½Π° ΠΈΠ· Π½ΠΈΡ… Π½Π΅ двиТСтся (ΠΎΡ‚Π½ΠΎΡΠΈΡ‚Π΅Π»ΡŒΠ½ΠΎ Π΄Ρ€ΡƒΠ³ Π΄Ρ€ΡƒΠ³Π°). БкольТСниС происходит ΠΌΠ΅ΠΆΠ΄Ρƒ двумя ΠΎΠ±ΡŠΠ΅ΠΊΡ‚Π°ΠΌΠΈ, ΡΠΊΠΎΠ»ΡŒΠ·ΡΡ‰ΠΈΠΌΠΈ Π΄Ρ€ΡƒΠ³ ΠΏΠΎ Π΄Ρ€ΡƒΠ³Ρƒ (ΠΊΠ°ΠΊ Π½ΠΈ странно) — Π½Π°ΠΏΡ€ΠΈΠΌΠ΅Ρ€, ΠΊΠΎΠ³Π΄Π° Π²Ρ‹ ΡΠΊΠΎΠ»ΡŒΠ·ΠΈΡ‚Π΅ ΠΏΠΎ дСрСвянному ΠΏΠΎΠ»Ρƒ Π² носках. ΠšΠ°Ρ‚ΠΎΠΊ находится ΠΌΠ΅ΠΆΠ΄Ρƒ ΠΏΠΎΠ²Π΅Ρ€Ρ…Π½ΠΎΡΡ‚ΡŒΡŽ ΠΈ катящимся ΠΏΡ€Π΅Π΄ΠΌΠ΅Ρ‚ΠΎΠΌ (колСсом, мячом ΠΈ Ρ‚. Π”.). Π’Ρ€Π΅Π½ΠΈΠ΅ Тидкости — это Ρ‚Ρ€Π΅Π½ΠΈΠ΅ ΠΌΠ΅ΠΆΠ΄Ρƒ двиТущимся ΠΎΠ±ΡŠΠ΅ΠΊΡ‚ΠΎΠΌ ΠΈ срСдой, Ρ‡Π΅Ρ€Π΅Π· ΠΊΠΎΡ‚ΠΎΡ€ΡƒΡŽ ΠΎΠ½ двиТСтся, Π½Π°ΠΏΡ€ΠΈΠΌΠ΅Ρ€, самолСтом Π² Π²ΠΎΠ·Π΄ΡƒΡ…Π΅ ΠΈΠ»ΠΈ Ρ€Ρ‹Π±ΠΎΠΉ Ρ‡Π΅Ρ€Π΅Π· Π²ΠΎΠ΄Ρƒ.

Π§Ρ‚ΠΎ Ρ‚Π°ΠΊΠΎΠ΅ Π΅Π΄ΠΈΠ½ΠΈΡ†Π° трСния Π² систСмС БИ?

Как ΠΈ всС силы, Π΅Π΄ΠΈΠ½ΠΈΡ†Π΅ΠΉ трСния являСтся ΠΡŒΡŽΡ‚ΠΎΠ½ , Ρ‡Ρ‚ΠΎ Ρ€Π°Π²Π½ΠΎ 1 ΠΊΠ³ Β· ΠΌ Β· с -2 .Π’ британских Π΅Π΄ΠΈΠ½ΠΈΡ†Π°Ρ… измСрСния силы — Ρ„ΡƒΠ½Ρ‚ΠΎΠ² силы , Ρ„ΡƒΠ½Ρ‚-сила, 1 ΠΈΠ· ΠΊΠΎΡ‚ΠΎΡ€Ρ‹Ρ… составляСт ΠΏΡ€ΠΈΠΌΠ΅Ρ€Π½ΠΎ 4,45 Н. ΠšΠΎΡΡ„Ρ„ΠΈΡ†ΠΈΠ΅Π½Ρ‚ трСния Π±Π΅Π·Ρ€Π°Π·ΠΌΠ΅Ρ€Π΅Π½ ΠΈ, ΡΠ»Π΅Π΄ΠΎΠ²Π°Ρ‚Π΅Π»ΡŒΠ½ΠΎ, Π½Π΅ ΠΈΠΌΠ΅Π΅Ρ‚ Π΅Π΄ΠΈΠ½ΠΈΡ†.

Π’ Ρ‡Π΅ΠΌ Ρ€Π°Π·Π½ΠΈΡ†Π° ΠΌΠ΅ΠΆΠ΄Ρƒ статичСским Ρ‚Ρ€Π΅Π½ΠΈΠ΅ΠΌ ΠΈ динамичСским Ρ‚Ρ€Π΅Π½ΠΈΠ΅ΠΌ?

БтатичСскоС ΠΈ динамичСскоС Ρ‚Ρ€Π΅Π½ΠΈΠ΅ Ρ€Π°Π·Π»ΠΈΡ‡Π°ΡŽΡ‚ΡΡ ΠΏΠΎ двиТСнию . БтатичСскоС Ρ‚Ρ€Π΅Π½ΠΈΠ΅ — это Ρ‚Ρ€Π΅Π½ΠΈΠ΅ ΠΌΠ΅ΠΆΠ΄Ρƒ двумя повСрхностями , ΠΊΠΎΡ‚ΠΎΡ€Ρ‹Π΅ Π½Π΅ двиТутся ΠΎΡ‚Π½ΠΎΡΠΈΡ‚Π΅Π»ΡŒΠ½ΠΎ Π΄Ρ€ΡƒΠ³ Π΄Ρ€ΡƒΠ³Π°. Если Π±Ρ‹ эти Π΄Π²Π΅ повСрхности двигались ΠΎΡ‚Π½ΠΎΡΠΈΡ‚Π΅Π»ΡŒΠ½ΠΎ Π΄Ρ€ΡƒΠ³ Π΄Ρ€ΡƒΠ³Π° , это Π±Ρ‹Π»ΠΎ Π±Ρ‹ динамичСскоС Ρ‚Ρ€Π΅Π½ΠΈΠ΅.НапримСр, Π±Π»ΠΎΠΊ Π½Π° столС испытываСт статичСскоС Ρ‚Ρ€Π΅Π½ΠΈΠ΅. Π­Ρ‚ΠΎ продолТаСтся, ΠΏΠΎΠΊΠ° Π²Ρ‹ наклоняСтС стол, ΠΏΠΎΠΊΠ° Π±Π»ΠΎΠΊ Π½Π΅ пСрСмСстится, ΠΈ Π² этот ΠΌΠΎΠΌΠ΅Π½Ρ‚ Ρ‚Ρ€Π΅Π½ΠΈΠ΅ станСт динамичСским.

Как Ρ‚Ρ€Π΅Π½ΠΈΠ΅ влияСт Π½Π° Π΄Π²ΠΈΠΆΠ΅Π½ΠΈΠ΅?

Π’Ρ€Π΅Π½ΠΈΠ΅ — это Ρ‚ΠΎ, Ρ‡Ρ‚ΠΎ Π΄Π΅Π»Π°Π΅Ρ‚ Π²ΠΎΠ·ΠΌΠΎΠΆΠ½Ρ‹ΠΌ Π΄Π²ΠΈΠΆΠ΅Π½ΠΈΠ΅ . Когда ΠΎΠ΄ΠΈΠ½ ΠΎΠ±ΡŠΠ΅ΠΊΡ‚ двиТСтся ΠΎΡ‚Π½ΠΎΡΠΈΡ‚Π΅Π»ΡŒΠ½ΠΎ Π΄Ρ€ΡƒΠ³ΠΎΠ³ΠΎ, Π΄Π²Π΅ ΠΏΡ€ΠΎΡ‚ΠΈΠ²ΠΎΠΏΠΎΠ»ΠΎΠΆΠ½Ρ‹Π΅, Π½ΠΎ Ρ€Π°Π²Π½Ρ‹Π΅ силы ΠΎΠ±Ρ€Π°Π·ΡƒΡŽΡ‚ΡΡ ΠΈΠ·-Π·Π° трСния , Π±Π΅Π· ΠΊΠΎΡ‚ΠΎΡ€ΠΎΠ³ΠΎ Π²Ρ‹ Π±Ρ‹ Π±Π΅ΠΆΠ°Π»ΠΈ Π½Π° мСстС. Π’Ρ€Π΅Π½ΠΈΠ΅ Ρ‚Π°ΠΊΠΆΠ΅ позволяСт Π½Π°ΠΌ ΠΎΡΡ‚Π°Π½ΠΎΠ²ΠΈΡ‚ΡŒ — ΠΊΠΎΠ³Π΄Π° ΠΎΠ±ΡŠΠ΅ΠΊΡ‚ замСдляСтся, Ρ‚Ρ€Π΅Π½ΠΈΠ΅ ΠΏΡ€Π΅Π²Ρ€Π°Ρ‰Π°Π΅Ρ‚ ΠΊΠΈΠ½Π΅Ρ‚ΠΈΡ‡Π΅ΡΠΊΡƒΡŽ ΡΠ½Π΅Ρ€Π³ΠΈΡŽ Π² Ρ‚Π΅ΠΏΠ»ΠΎΠ²ΡƒΡŽ ΠΈΠ»ΠΈ ΠΊΠΎΠ»Π΅Π±Π°Ρ‚Π΅Π»ΡŒΠ½ΡƒΡŽ ΡΠ½Π΅Ρ€Π³ΠΈΡŽ.Π§Π΅ΠΌ Π±ΠΎΠ»Π΅Π΅ скользкая ΠΏΠΎΠ²Π΅Ρ€Ρ…Π½ΠΎΡΡ‚ΡŒ (Ρ‡Π΅ΠΌ Π²Ρ‹ΡˆΠ΅ коэффициСнт трСния), Ρ‚Π΅ΠΌ ΠΌΠ΅Π΄Π»Π΅Π½Π½Π΅Π΅ ΡΠΊΠΎΡ€ΠΎΡΡ‚ΡŒ ΠΏΠ΅Ρ€Π΅Π΄Π°Ρ‡ΠΈ этой энСргии.

Как Ρ€Π°ΡΡΡ‡ΠΈΡ‚Π°Ρ‚ΡŒ силу трСния

ОбновлСно 8 дСкабря 2020 Π³.

Π›ΠΈ ДТонсон

ΠŸΠΎΠ²Π΅Ρ€Ρ…Π½ΠΎΡΡ‚ΠΈ ΡΠΎΠ·Π΄Π°ΡŽΡ‚ силу трСния, которая сопротивляСтся ΡΠΊΠΎΠ»ΡŒΠ·ΡΡ‰ΠΈΠΌ двиТСниям, ΠΈ Π²Π°ΠΌ Π½Π΅ΠΎΠ±Ρ…ΠΎΠ΄ΠΈΠΌΠΎ Ρ€Π°ΡΡΡ‡ΠΈΡ‚Π°Ρ‚ΡŒ Π²Π΅Π»ΠΈΡ‡ΠΈΠ½Ρƒ этой силы ΠΊΠ°ΠΊ Ρ‡Π°ΡΡ‚ΡŒ ΠΌΠ½ΠΎΠ³ΠΈΡ… физичСских Π·Π°Π΄Π°Ρ‡. Π’Π΅Π»ΠΈΡ‡ΠΈΠ½Π° трСния Π² основном зависит ΠΎΡ‚ Β«Π½ΠΎΡ€ΠΌΠ°Π»ΡŒΠ½ΠΎΠΉ силы», ΠΊΠΎΡ‚ΠΎΡ€ΡƒΡŽ повСрхности ΠΎΠΊΠ°Π·Ρ‹Π²Π°ΡŽΡ‚ Π½Π° сидящиС Π½Π° Π½ΠΈΡ… ΠΏΡ€Π΅Π΄ΠΌΠ΅Ρ‚Ρ‹, Π° Ρ‚Π°ΠΊΠΆΠ΅ ΠΎΡ‚ характСристик рассматриваСмой ΠΊΠΎΠ½ΠΊΡ€Π΅Ρ‚Π½ΠΎΠΉ повСрхности.Π’ Π±ΠΎΠ»ΡŒΡˆΠΈΠ½ΡΡ‚Π²Π΅ случаСв Π²Ρ‹ ΠΌΠΎΠΆΠ΅Ρ‚Π΅ ΠΈΡΠΏΠΎΠ»ΡŒΠ·ΠΎΠ²Π°Ρ‚ΡŒ Ρ„ΠΎΡ€ΠΌΡƒΠ»Ρƒ:

F = \ mu N

для расчСта трСния, Π³Π΄Π΅ N ΠΎΠ·Π½Π°Ρ‡Π°Π΅Ρ‚ Β«Π½ΠΎΡ€ΠΌΠ°Π»ΡŒΠ½ΡƒΡŽΒ» силу, Π° Β« ΞΌ Β» Π²ΠΊΠ»ΡŽΡ‡Π°Π΅Ρ‚ характСристики повСрхности. .

Π§Ρ‚ΠΎ Ρ‚Π°ΠΊΠΎΠ΅ Ρ‚Ρ€Π΅Π½ΠΈΠ΅?

Π’Ρ€Π΅Π½ΠΈΠ΅ описываСт силу ΠΌΠ΅ΠΆΠ΄Ρƒ двумя повСрхностями, ΠΊΠΎΠ³Π΄Π° Π²Ρ‹ ΠΏΡ‹Ρ‚Π°Π΅Ρ‚Π΅ΡΡŒ ΠΏΠ΅Ρ€Π΅ΠΌΠ΅ΡΡ‚ΠΈΡ‚ΡŒ ΠΎΠ΄Π½Ρƒ ΠΏΠΎΠ²Π΅Ρ€Ρ… Π΄Ρ€ΡƒΠ³ΠΎΠΉ. Π‘ΠΈΠ»Π° сопротивляСтся двиТСнию, ΠΈ Π² Π±ΠΎΠ»ΡŒΡˆΠΈΠ½ΡΡ‚Π²Π΅ случаСв сила дСйствуСт Π² Π½Π°ΠΏΡ€Π°Π²Π»Π΅Π½ΠΈΠΈ, ΠΏΡ€ΠΎΡ‚ΠΈΠ²ΠΎΠΏΠΎΠ»ΠΎΠΆΠ½ΠΎΠΌ двиТСнию. На молСкулярном ΡƒΡ€ΠΎΠ²Π½Π΅, ΠΊΠΎΠ³Π΄Π° Π²Ρ‹ ΠΏΡ€ΠΈΠΆΠΈΠΌΠ°Π΅Ρ‚Π΅ Π΄Π²Π΅ повСрхности вмСстС, нСбольшиС Π΄Π΅Ρ„Π΅ΠΊΡ‚Ρ‹ Π½Π° ΠΊΠ°ΠΆΠ΄ΠΎΠΉ повСрхности ΠΌΠΎΠ³ΡƒΡ‚ ΡΡ†Π΅ΠΏΠ»ΡΡ‚ΡŒΡΡ, ΠΈ ΠΌΠ΅ΠΆΠ΄Ρƒ ΠΌΠΎΠ»Π΅ΠΊΡƒΠ»Π°ΠΌΠΈ ΠΎΠ΄Π½ΠΎΠ³ΠΎ ΠΌΠ°Ρ‚Π΅Ρ€ΠΈΠ°Π»Π° ΠΈ Π΄Ρ€ΡƒΠ³ΠΎΠ³ΠΎ ΠΌΠΎΠ³ΡƒΡ‚ Π²ΠΎΠ·Π½ΠΈΠΊΠ°Ρ‚ΡŒ силы притяТСния.Π­Ρ‚ΠΈ Ρ„Π°ΠΊΡ‚ΠΎΡ€Ρ‹ Π·Π°Ρ‚Ρ€ΡƒΠ΄Π½ΡΡŽΡ‚ ΠΈΡ… ΠΎΠ±Ρ…ΠΎΠ΄ Π΄Ρ€ΡƒΠ³ Π΄Ρ€ΡƒΠ³Π°. Однако Π²Ρ‹ Π½Π΅ Ρ€Π°Π±ΠΎΡ‚Π°Π΅Ρ‚Π΅ Π½Π° этом ΡƒΡ€ΠΎΠ²Π½Π΅, ΠΊΠΎΠ³Π΄Π° вычисляСтС силу трСния. Для повсСднСвных ситуаций Ρ„ΠΈΠ·ΠΈΠΊΠΈ Π³Ρ€ΡƒΠΏΠΏΠΈΡ€ΡƒΡŽΡ‚ всС эти Ρ„Π°ΠΊΡ‚ΠΎΡ€Ρ‹ вмСстС Π² «коэффициСнт» ΞΌ .

РасчСт силы трСния

    Β«ΠΠΎΡ€ΠΌΠ°Π»ΡŒΠ½Π°ΡΒ» сила описываСт силу, ΠΊΠΎΡ‚ΠΎΡ€ΡƒΡŽ ΠΏΠΎΠ²Π΅Ρ€Ρ…Π½ΠΎΡΡ‚ΡŒ, Π½Π° ΠΊΠΎΡ‚ΠΎΡ€ΡƒΡŽ опираСтся (ΠΈΠ»ΠΈ Π½Π° ΠΊΠΎΡ‚ΠΎΡ€ΡƒΡŽ приТимаСтся) ΠΎΠ±ΡŠΠ΅ΠΊΡ‚, ΠΎΠΊΠ°Π·Ρ‹Π²Π°Π΅Ρ‚ Π½Π° ΠΎΠ±ΡŠΠ΅ΠΊΡ‚. Для Π½Π΅ΠΏΠΎΠ΄Π²ΠΈΠΆΠ½ΠΎΠ³ΠΎ ΠΎΠ±ΡŠΠ΅ΠΊΡ‚Π° Π½Π° плоской повСрхности сила Π΄ΠΎΠ»ΠΆΠ½Π° Ρ‚ΠΎΡ‡Π½ΠΎ ΠΏΡ€ΠΎΡ‚ΠΈΠ²ΠΎΠ΄Π΅ΠΉΡΡ‚Π²ΠΎΠ²Π°Ρ‚ΡŒ силС Π³Ρ€Π°Π²ΠΈΡ‚Π°Ρ†ΠΈΠΈ, ΠΈΠ½Π°Ρ‡Π΅ ΠΎΠ±ΡŠΠ΅ΠΊΡ‚ Π±ΡƒΠ΄Π΅Ρ‚ Π΄Π²ΠΈΠ³Π°Ρ‚ΡŒΡΡ Π² соотвСтствии с Π·Π°ΠΊΠΎΠ½Π°ΠΌΠΈ двиТСния ΠΡŒΡŽΡ‚ΠΎΠ½Π°.Β«ΠΠΎΡ€ΠΌΠ°Π»ΡŒΠ½Π°ΡΒ» сила ( Н, ) — это Π½Π°Π·Π²Π°Π½ΠΈΠ΅ силы, которая это Π΄Π΅Π»Π°Π΅Ρ‚.

    Он всСгда дСйствуСт пСрпСндикулярно повСрхности. Π­Ρ‚ΠΎ ΠΎΠ·Π½Π°Ρ‡Π°Π΅Ρ‚, Ρ‡Ρ‚ΠΎ Π½Π° Π½Π°ΠΊΠ»ΠΎΠ½Π½ΠΎΠΉ повСрхности Π½ΠΎΡ€ΠΌΠ°Π»ΡŒΠ½Π°Ρ сила ΠΏΠΎ-ΠΏΡ€Π΅ΠΆΠ½Π΅ΠΌΡƒ Π±ΡƒΠ΄Π΅Ρ‚ Π½Π°ΠΏΡ€Π°Π²Π»Π΅Π½Π° ​​прямо ΠΎΡ‚ повСрхности, Π² Ρ‚ΠΎ врСмя ΠΊΠ°ΠΊ сила тяТСсти Π±ΡƒΠ΄Π΅Ρ‚ Π½Π°ΠΏΡ€Π°Π²Π»Π΅Π½Π° ​​прямо Π²Π½ΠΈΠ·.

    ΠΠΎΡ€ΠΌΠ°Π»ΡŒΠ½ΡƒΡŽ силу Π² Π±ΠΎΠ»ΡŒΡˆΠΈΠ½ΡΡ‚Π²Π΅ случаСв ΠΌΠΎΠΆΠ½ΠΎ просто ΠΎΠΏΠΈΡΠ°Ρ‚ΡŒ ΡΠ»Π΅Π΄ΡƒΡŽΡ‰ΠΈΠΌ ΠΎΠ±Ρ€Π°Π·ΠΎΠΌ:

    N = mg

    Π—Π΄Π΅ΡΡŒ m прСдставляСт массу ΠΎΠ±ΡŠΠ΅ΠΊΡ‚Π°, Π° g ΠΎΠ±ΠΎΠ·Π½Π°Ρ‡Π°Π΅Ρ‚ ускорСниС свободного падСния, ΠΊΠΎΡ‚ΠΎΡ€ΠΎΠ΅ это 9.8 ΠΌΠ΅Ρ‚Ρ€ΠΎΠ² Π² сСкунду Π² сСкунду (ΠΌ / с 2 ) ΠΈΠ»ΠΈ чистых Π²Ρ‹ΠΈΠ³Ρ€Ρ‹ΡˆΠ΅ΠΉ Π½Π° ΠΊΠΈΠ»ΠΎΠ³Ρ€Π°ΠΌΠΌ (Н / ΠΊΠ³). Π­Ρ‚ΠΎ просто соотвСтствуСт «вСсу» ΠΎΠ±ΡŠΠ΅ΠΊΡ‚Π°.

    Для Π½Π°ΠΊΠ»ΠΎΠ½Π½Ρ‹Ρ… повСрхностСй сила Π½ΠΎΡ€ΠΌΠ°Π»ΡŒΠ½ΠΎΠΉ силы ΡƒΠΌΠ΅Π½ΡŒΡˆΠ°Π΅Ρ‚ΡΡ с ΡƒΠ²Π΅Π»ΠΈΡ‡Π΅Π½ΠΈΠ΅ΠΌ Π½Π°ΠΊΠ»ΠΎΠ½Π° повСрхности, поэтому Ρ„ΠΎΡ€ΠΌΡƒΠ»Π° ΠΏΡ€ΠΈΠ½ΠΈΠΌΠ°Π΅Ρ‚ ΡΠ»Π΅Π΄ΡƒΡŽΡ‰ΠΈΠΉ Π²ΠΈΠ΄:

    N = mg \ cos {\ theta}

    ΠŸΡ€ΠΈ ΞΈ , ΠΎΠ±ΠΎΠ·Π½Π°Ρ‡Π°ΡŽΡ‰Π΅ΠΌ ΡƒΠ³ΠΎΠ», ΠΏΠΎΠ²Π΅Ρ€Ρ…Π½ΠΎΡΡ‚ΡŒ Π½Π°ΠΊΠ»ΠΎΠ½Π΅Π½Π° ΠΊ.

    Π’ качСствС простого ΠΏΡ€ΠΈΠΌΠ΅Ρ€Π° расчСта рассмотрим ΠΏΠ»ΠΎΡΠΊΡƒΡŽ ΠΏΠΎΠ²Π΅Ρ€Ρ…Π½ΠΎΡΡ‚ΡŒ с 2-ΠΊΠΈΠ»ΠΎΠ³Ρ€Π°ΠΌΠΌΠΎΠ²Ρ‹ΠΌ дСрСвянным бруском, Π»Π΅ΠΆΠ°Ρ‰ΠΈΠΌ Π½Π° Π½Π΅ΠΉ.ΠΠΎΡ€ΠΌΠ°Π»ΡŒΠ½Π°Ρ сила Π±ΡƒΠ΄Π΅Ρ‚ Π½Π°ΠΏΡ€Π°Π²Π»Π΅Π½Π° ​​прямо Π²Π²Π΅Ρ€Ρ… (Ρ‡Ρ‚ΠΎΠ±Ρ‹ Π²Ρ‹Π΄Π΅Ρ€ΠΆΠ°Ρ‚ΡŒ вСс Π±Π»ΠΎΠΊΠ°), ΠΈ Π²Ρ‹ Π΄ΠΎΠ»ΠΆΠ½Ρ‹ Π²Ρ‹Ρ‡ΠΈΡΠ»ΠΈΡ‚ΡŒ:

    N = 2 \ times 9,8 = 19,6 \ text {N}

    ΠšΠΎΡΡ„Ρ„ΠΈΡ†ΠΈΠ΅Π½Ρ‚ зависит ΠΎΡ‚ ΠΎΠ±ΡŠΠ΅ΠΊΡ‚Π° ΠΈ ΠΊΠΎΠ½ΠΊΡ€Π΅Ρ‚Π½ΠΎΠΉ ситуации, Π² ΠΊΠΎΡ‚ΠΎΡ€ΠΎΠΉ Π²Ρ‹ Π½Π°Ρ…ΠΎΠ΄ΠΈΡ‚Π΅ΡΡŒ. Ρ€Π°Π±ΠΎΡ‚Π°Π΅ΠΌ с. Если ΠΎΠ±ΡŠΠ΅ΠΊΡ‚ Π΅Ρ‰Π΅ Π½Π΅ двиТСтся ΠΏΠΎ повСрхности, Π²Ρ‹ ΠΈΡΠΏΠΎΠ»ΡŒΠ·ΡƒΠ΅Ρ‚Π΅ коэффициСнт трСния покоя ΞΌ static , Π½ΠΎ Ссли ΠΎΠ½ двиТСтся, Π²Ρ‹ ΠΈΡΠΏΠΎΠ»ΡŒΠ·ΡƒΠ΅Ρ‚Π΅ коэффициСнт трСния скольТСния ΞΌ slide .

    ΠžΠ±Ρ‹Ρ‡Π½ΠΎ коэффициСнт трСния скольТСния мСньшС, Ρ‡Π΅ΠΌ коэффициСнт трСния покоя. Π”Ρ€ΡƒΠ³ΠΈΠΌΠΈ словами, Π»Π΅Π³Ρ‡Π΅ ΡΠ΄Π²ΠΈΠ½ΡƒΡ‚ΡŒ Ρ‚ΠΎ, Ρ‡Ρ‚ΠΎ ΡƒΠΆΠ΅ ΡΠΊΠΎΠ»ΡŒΠ·ΠΈΡ‚, Ρ‡Π΅ΠΌ ΡΠ΄Π²ΠΈΠ½ΡƒΡ‚ΡŒ Ρ‚ΠΎ, Ρ‡Ρ‚ΠΎ Π½Π΅ΠΏΠΎΠ΄Π²ΠΈΠΆΠ½ΠΎ.

    ΠœΠ°Ρ‚Π΅Ρ€ΠΈΠ°Π»Ρ‹, ΠΊΠΎΡ‚ΠΎΡ€Ρ‹Π΅ Π²Ρ‹ рассматриваСтС, Ρ‚Π°ΠΊΠΆΠ΅ Π²Π»ΠΈΡΡŽΡ‚ Π½Π° коэффициСнт. НапримСр, Ссли Ρ€Π°Π½Π΅Π΅ дСрСвянный брусок находился Π½Π° ΠΊΠΈΡ€ΠΏΠΈΡ‡Π½ΠΎΠΉ повСрхности, коэффициСнт Π±ΡƒΠ΄Π΅Ρ‚ 0,6, Π° для чистой дрСвСсины ΠΎΠ½ ΠΌΠΎΠΆΠ΅Ρ‚ Π±Ρ‹Ρ‚ΡŒ ΠΎΡ‚ 0,25 Π΄ΠΎ 0,5. Для льда Π½Π° Π»ΡŒΠ΄Ρƒ статичСский коэффициСнт Ρ€Π°Π²Π΅Π½ 0.1. И снова коэффициСнт скольТСния сниТаСт Π΅Π³ΠΎ Π΅Ρ‰Π΅ большС, Π΄ΠΎ 0,03 для льда ΠΏΠΎ Π»ΡŒΠ΄Ρƒ ΠΈ 0,2 для Π΄Π΅Ρ€Π΅Π²Π° ΠΏΠΎ Π΄Π΅Ρ€Π΅Π²Ρƒ. НайдитС ΠΈΡ… для своСй повСрхности с ΠΏΠΎΠΌΠΎΡ‰ΡŒΡŽ ΠΎΠ½Π»Π°ΠΉΠ½-Ρ‚Π°Π±Π»ΠΈΡ†Ρ‹ (см. РСсурсы).

    Π€ΠΎΡ€ΠΌΡƒΠ»Π° силы трСния состояния:

    F = \ mu N

    Для ΠΏΡ€ΠΈΠΌΠ΅Ρ€Π° рассмотрим дСрСвянный брусок массой 2 ΠΊΠ³ Π½Π° дСрСвянном столС, ΠΊΠΎΡ‚ΠΎΡ€Ρ‹ΠΉ Ρ‚ΠΎΠ»ΠΊΠ°ΡŽΡ‚ с мСста. Π’ этом случаС Π²Ρ‹ ΠΈΡΠΏΠΎΠ»ΡŒΠ·ΡƒΠ΅Ρ‚Π΅ статичСский коэффициСнт с ΞΌ static = ΠΎΡ‚ 0,25 Π΄ΠΎ 0,5 для дрСвСсины. ΠŸΡ€ΠΈΠ½ΠΈΠΌΠ°Ρ ΞΌ статичСский = 0.5, Ρ‡Ρ‚ΠΎΠ±Ρ‹ ΠΌΠ°ΠΊΡΠΈΠΌΠΈΠ·ΠΈΡ€ΠΎΠ²Π°Ρ‚ΡŒ ΠΏΠΎΡ‚Π΅Π½Ρ†ΠΈΠ°Π»ΡŒΠ½Ρ‹ΠΉ эффСкт трСния, ΠΈ вспомнив N = 19,6 Н ΠΈΠ· ΠΏΡ€Π΅Π΄Ρ‹Π΄ΡƒΡ‰Π΅Π³ΠΎ, сила Π±ΡƒΠ΄Π΅Ρ‚:

    F = 0,5 \ times19,6 = 9,8 \ text {N}

    ΠŸΠΎΠΌΠ½ΠΈΡ‚Π΅, Ρ‡Ρ‚ΠΎ Ρ‚Ρ€Π΅Π½ΠΈΠ΅ Ρ‚ΠΎΠ»ΡŒΠΊΠΎ обСспСчиваСт силу для сопротивлСния двиТСнию, поэтому, Ссли Π²Ρ‹ Π½Π°Ρ‡Π½Π΅Ρ‚Π΅ остороТно Ρ‚ΠΎΠ»ΠΊΠ°Ρ‚ΡŒ Π΅Π³ΠΎ ΠΈ станСтС Π±ΠΎΠ»Π΅Π΅ Ρ‚Π²Π΅Ρ€Π΄Ρ‹ΠΌ, сила трСния увСличится Π΄ΠΎ максимального значСния, ΠΊΠΎΡ‚ΠΎΡ€ΠΎΠ΅ Π²Ρ‹ Ρ‚ΠΎΠ»ΡŒΠΊΠΎ Ρ‡Ρ‚ΠΎ рассчитали. Π€ΠΈΠ·ΠΈΠΊΠΈ ΠΈΠ½ΠΎΠ³Π΄Π° ΠΏΠΈΡˆΡƒΡ‚ F max , Ρ‡Ρ‚ΠΎΠ±Ρ‹ ΠΏΡ€ΠΎΡΡΠ½ΠΈΡ‚ΡŒ этот ΠΌΠΎΠΌΠ΅Π½Ρ‚.

    Когда Π±Π»ΠΎΠΊ пСрСмСщаСтся, Π²Ρ‹ ΠΈΡΠΏΠΎΠ»ΡŒΠ·ΡƒΠ΅Ρ‚Π΅ ΞΌ slide = 0.2, Π² Π΄Π°Π½Π½ΠΎΠΌ случаС:

    F_ {slide} = \ mu_ {slide} N = 0,2 \ times 19,6 = 3,92 \ text {N}

Как Ρ€Π°ΡΡΡ‡ΠΈΡ‚Π°Ρ‚ΡŒ ускорСниС с Ρ‚Ρ€Π΅Π½ΠΈΠ΅ΠΌ

ОбновлСно 5 дСкабря 2020 Π³.

Автор ΠšΡ€ΠΈΡ Π”Π΅Π·ΠΈΠ»

Как сила, ΠΏΡ€Π΅ΠΏΡΡ‚ΡΡ‚Π²ΡƒΡŽΡ‰Π°Ρ двиТСнию, Ρ‚Ρ€Π΅Π½ΠΈΠ΅ всСгда сниТаСт ускорСниС. Π’Ρ€Π΅Π½ΠΈΠ΅ Π²ΠΎΠ·Π½ΠΈΠΊΠ°Π΅Ρ‚ ΠΌΠ΅ΠΆΠ΄Ρƒ взаимодСйствиСм ΠΎΠ±ΡŠΠ΅ΠΊΡ‚Π° с ΠΏΠΎΠ²Π΅Ρ€Ρ…Π½ΠΎΡΡ‚ΡŒΡŽ. Π•Π³ΠΎ Π²Π΅Π»ΠΈΡ‡ΠΈΠ½Π° зависит ΠΎΡ‚ характСристик повСрхности ΠΈ ΠΎΠ±ΡŠΠ΅ΠΊΡ‚Π°, Π° Ρ‚Π°ΠΊΠΆΠ΅ ΠΎΡ‚ Ρ‚ΠΎΠ³ΠΎ, двиТСтся ΠΎΠ±ΡŠΠ΅ΠΊΡ‚ ΠΈΠ»ΠΈ Π½Π΅Ρ‚.Π’Ρ€Π΅Π½ΠΈΠ΅ ΠΌΠΎΠΆΠ΅Ρ‚ Π±Ρ‹Ρ‚ΡŒ Ρ€Π΅Π·ΡƒΠ»ΡŒΡ‚Π°Ρ‚ΠΎΠΌ взаимодСйствия Π΄Π²ΡƒΡ… Ρ‚Π²Π΅Ρ€Π΄Ρ‹Ρ… ΠΎΠ±ΡŠΠ΅ΠΊΡ‚ΠΎΠ², Π½ΠΎ Π½Π΅ ΠΎΠ±ΡΠ·Π°Ρ‚Π΅Π»ΡŒΠ½ΠΎ. Π‘ΠΎΠΏΡ€ΠΎΡ‚ΠΈΠ²Π»Π΅Π½ΠΈΠ΅ Π²ΠΎΠ·Π΄ΡƒΡ…Π° — это Ρ‚ΠΈΠΏ силы трСния, ΠΈ Π²Ρ‹ Π΄Π°ΠΆΠ΅ ΠΌΠΎΠΆΠ΅Ρ‚Π΅ Ρ€Π°ΡΡΠΌΠ°Ρ‚Ρ€ΠΈΠ²Π°Ρ‚ΡŒ взаимодСйствиС Ρ‚Π²Π΅Ρ€Π΄ΠΎΠ³ΠΎ Ρ‚Π΅Π»Π°, двиТущСгося ΠΏΠΎ Π²ΠΎΠ΄Π΅ ΠΈΠ»ΠΈ сквозь Π²ΠΎΠ΄Ρƒ, ΠΊΠ°ΠΊ Ρ„Ρ€ΠΈΠΊΡ†ΠΈΠΎΠ½Π½ΠΎΠ΅ взаимодСйствиС.

TL; DR (слишком Π΄ΠΎΠ»Π³ΠΎ; Π½Π΅ Ρ‡ΠΈΡ‚Π°Π»)

Π‘ΠΈΠ»Π° трСния зависит ΠΎΡ‚ массы ΠΎΠ±ΡŠΠ΅ΠΊΡ‚Π° плюс коэффициСнт трСния скольТСния ΠΌΠ΅ΠΆΠ΄Ρƒ ΠΎΠ±ΡŠΠ΅ΠΊΡ‚ΠΎΠΌ ΠΈ ΠΏΠΎΠ²Π΅Ρ€Ρ…Π½ΠΎΡΡ‚ΡŒΡŽ, ΠΏΠΎ ΠΊΠΎΡ‚ΠΎΡ€ΠΎΠΉ ΠΎΠ½ ΡΠΊΠΎΠ»ΡŒΠ·ΠΈΡ‚. Π’Ρ‹Ρ‡Ρ‚ΠΈΡ‚Π΅ эту силу ΠΈΠ· ΠΏΡ€ΠΈΠ»ΠΎΠΆΠ΅Π½Π½ΠΎΠΉ силы, Ρ‡Ρ‚ΠΎΠ±Ρ‹ Π½Π°ΠΉΡ‚ΠΈ ускорСниС ΠΎΠ±ΡŠΠ΅ΠΊΡ‚Π°.

Как Ρ€Π°ΡΡΡ‡ΠΈΡ‚Π°Ρ‚ΡŒ силу трСния

Π‘ΠΈΠ»Π° — это вСкторная Π²Π΅Π»ΠΈΡ‡ΠΈΠ½Π°, Ρ‡Ρ‚ΠΎ ΠΎΠ·Π½Π°Ρ‡Π°Π΅Ρ‚, Ρ‡Ρ‚ΠΎ Π²Ρ‹ Π΄ΠΎΠ»ΠΆΠ½Ρ‹ ΡƒΡ‡ΠΈΡ‚Ρ‹Π²Π°Ρ‚ΡŒ Π½Π°ΠΏΡ€Π°Π²Π»Π΅Π½ΠΈΠ΅, Π² ΠΊΠΎΡ‚ΠΎΡ€ΠΎΠΌ ΠΎΠ½Π° дСйствуСт. Π‘ΡƒΡ‰Π΅ΡΡ‚Π²ΡƒΡŽΡ‚ Π΄Π²Π° основных Ρ‚ΠΈΠΏΠ° сил трСния: статичСская сила (F st ) ΠΈ сила скольТСния (F sl ). НСсмотря Π½Π° Ρ‚ΠΎ, Ρ‡Ρ‚ΠΎ ΠΎΠ½ΠΈ Π΄Π΅ΠΉΡΡ‚Π²ΡƒΡŽΡ‚ Π² Π½Π°ΠΏΡ€Π°Π²Π»Π΅Π½ΠΈΠΈ, ΠΏΡ€ΠΎΡ‚ΠΈΠ²ΠΎΠΏΠΎΠ»ΠΎΠΆΠ½ΠΎΠΌ Π½Π°ΠΏΡ€Π°Π²Π»Π΅Π½ΠΈΡŽ двиТСния ΠΎΠ±ΡŠΠ΅ΠΊΡ‚Π°, Π½ΠΎΡ€ΠΌΠ°Π»ΡŒΠ½Π°Ρ сила (F N ) создаСт эти силы, ΠΊΠΎΡ‚ΠΎΡ€Ρ‹Π΅ Π΄Π΅ΠΉΡΡ‚Π²ΡƒΡŽΡ‚ пСрпСндикулярно Π½Π°ΠΏΡ€Π°Π²Π»Π΅Π½ΠΈΡŽ двиТСния. F N Ρ€Π°Π²Π΅Π½ вСсу ΠΎΠ±ΡŠΠ΅ΠΊΡ‚Π° плюс Π»ΡŽΠ±Ρ‹Π΅ Π΄ΠΎΠΏΠΎΠ»Π½ΠΈΡ‚Π΅Π»ΡŒΠ½Ρ‹Π΅ вСса.НапримСр, Ссли Π²Ρ‹ Π½Π°Π΄Π°Π²ΠΈΡ‚Π΅ Π½Π° дСрСвянный брусок Π½Π° столС, Π²Ρ‹ ΡƒΠ²Π΅Π»ΠΈΡ‡ΠΈΡ‚Π΅ Π½ΠΎΡ€ΠΌΠ°Π»ΡŒΠ½ΡƒΡŽ силу ΠΈ, Ρ‚Π°ΠΊΠΈΠΌ ΠΎΠ±Ρ€Π°Π·ΠΎΠΌ, ΡƒΠ²Π΅Π»ΠΈΡ‡ΠΈΡ‚Π΅ силу трСния.

Как статичСскоС, Ρ‚Π°ΠΊ ΠΈ ΡΠΊΠΎΠ»ΡŒΠ·ΡΡ‰Π΅Π΅ Ρ‚Ρ€Π΅Π½ΠΈΠ΅ зависят ΠΎΡ‚ характСристик двиТущСгося Ρ‚Π΅Π»Π° ΠΈ повСрхности, ΠΏΠΎ ΠΊΠΎΡ‚ΠΎΡ€ΠΎΠΉ ΠΎΠ½ΠΎ двиТСтся. Π­Ρ‚ΠΈ характСристики Π²Ρ‹Ρ€Π°ΠΆΠ°ΡŽΡ‚ΡΡ Π² коэффициСнтах статичСского (Β΅ st ) ΠΈ скольТСния (Β΅ sl ) трСния. Π­Ρ‚ΠΈ коэффициСнты Π±Π΅Π·Ρ€Π°Π·ΠΌΠ΅Ρ€Π½Ρ‹ ΠΈ свСдСны Π² Ρ‚Π°Π±Π»ΠΈΡ†Ρƒ для ΠΌΠ½ΠΎΠ³ΠΈΡ… ΠΎΠ±Ρ‹Ρ‡Π½Ρ‹Ρ… ΠΏΡ€Π΅Π΄ΠΌΠ΅Ρ‚ΠΎΠ² ΠΈ повСрхностСй. Как Ρ‚ΠΎΠ»ΡŒΠΊΠΎ Π²Ρ‹ Π½Π°ΠΉΠ΄Π΅Ρ‚Π΅ Ρ‚ΠΎΡ‚, ΠΊΠΎΡ‚ΠΎΡ€Ρ‹ΠΉ ΠΏΡ€ΠΈΠΌΠ΅Π½ΠΈΠΌ Π² вашСй ситуации, Π²Ρ‹ рассчитываСтС силы трСния, ΠΈΡΠΏΠΎΠ»ΡŒΠ·ΡƒΡ ΡΠ»Π΅Π΄ΡƒΡŽΡ‰ΠΈΠ΅ уравнСния:

F_ {st} \ leq \ mu_ {st} F_N \\\ text {} \\ F_ {sl} = \ mu_ { sl} F_N

РасчСт ускорСния

Π’Ρ‚ΠΎΡ€ΠΎΠΉ Π·Π°ΠΊΠΎΠ½ ΠΡŒΡŽΡ‚ΠΎΠ½Π° гласит, Ρ‡Ρ‚ΠΎ ускорСниС ΠΎΠ±ΡŠΠ΅ΠΊΡ‚Π° (a) ΠΏΡ€ΠΎΠΏΠΎΡ€Ρ†ΠΈΠΎΠ½Π°Π»ΡŒΠ½ΠΎ силС (F), ΠΏΡ€ΠΈΠ»ΠΎΠΆΠ΅Π½Π½ΠΎΠΉ ΠΊ Π½Π΅ΠΌΡƒ, Π° коэффициСнт ΠΏΡ€ΠΎΠΏΠΎΡ€Ρ†ΠΈΠΎΠ½Π°Π»ΡŒΠ½ΠΎΡΡ‚ΠΈ — это масса ΠΎΠ±ΡŠΠ΅ΠΊΡ‚Π° (m).Если вас интСрСсуСт ускорСниС, ΠΈΠ·ΠΌΠ΅Π½ΠΈΡ‚Π΅ ΡƒΡ€Π°Π²Π½Π΅Π½ΠΈΠ΅ ΡΠ»Π΅Π΄ΡƒΡŽΡ‰ΠΈΠΌ ΠΎΠ±Ρ€Π°Π·ΠΎΠΌ:

a = \ frac {F} {m}

Π‘ΠΈΠ»Π° — это вСкторная Π²Π΅Π»ΠΈΡ‡ΠΈΠ½Π°, Ρ‡Ρ‚ΠΎ ΠΎΠ·Π½Π°Ρ‡Π°Π΅Ρ‚, Ρ‡Ρ‚ΠΎ Π²Ρ‹ Π΄ΠΎΠ»ΠΆΠ½Ρ‹ ΡƒΡ‡ΠΈΡ‚Ρ‹Π²Π°Ρ‚ΡŒ Π½Π°ΠΏΡ€Π°Π²Π»Π΅Π½ΠΈΠ΅, Π² ΠΊΠΎΡ‚ΠΎΡ€ΠΎΠΌ ΠΎΠ½Π° дСйствуСт. Π‘ΡƒΡ‰Π΅ΡΡ‚Π²ΡƒΡŽΡ‚ Π΄Π²Π° основных Ρ‚ΠΈΠΏΠ° сил трСния: статичСская сила (F st ) ΠΈ сила скольТСния (F sl ). НСсмотря Π½Π° Ρ‚ΠΎ, Ρ‡Ρ‚ΠΎ ΠΎΠ½ΠΈ Π΄Π΅ΠΉΡΡ‚Π²ΡƒΡŽΡ‚ Π² Π½Π°ΠΏΡ€Π°Π²Π»Π΅Π½ΠΈΠΈ, ΠΏΡ€ΠΎΡ‚ΠΈΠ²ΠΎΠΏΠΎΠ»ΠΎΠΆΠ½ΠΎΠΌ Π½Π°ΠΏΡ€Π°Π²Π»Π΅Π½ΠΈΡŽ двиТСния ΠΎΠ±ΡŠΠ΅ΠΊΡ‚Π°, Π½ΠΎΡ€ΠΌΠ°Π»ΡŒΠ½Π°Ρ сила (F N ) создаСт эти силы, ΠΊΠΎΡ‚ΠΎΡ€Ρ‹Π΅ Π΄Π΅ΠΉΡΡ‚Π²ΡƒΡŽΡ‚ пСрпСндикулярно Π½Π°ΠΏΡ€Π°Π²Π»Π΅Π½ΠΈΡŽ двиТСния.F N Ρ€Π°Π²Π΅Π½ вСсу ΠΎΠ±ΡŠΠ΅ΠΊΡ‚Π° плюс Π»ΡŽΠ±Ρ‹Π΅ Π΄ΠΎΠΏΠΎΠ»Π½ΠΈΡ‚Π΅Π»ΡŒΠ½Ρ‹Π΅ вСса. НапримСр, Ссли Π²Ρ‹ Π½Π°Π΄Π°Π²ΠΈΡ‚Π΅ Π½Π° дСрСвянный брусок Π½Π° столС, Π²Ρ‹ ΡƒΠ²Π΅Π»ΠΈΡ‡ΠΈΡ‚Π΅ Π½ΠΎΡ€ΠΌΠ°Π»ΡŒΠ½ΡƒΡŽ силу ΠΈ, Ρ‚Π°ΠΊΠΈΠΌ ΠΎΠ±Ρ€Π°Π·ΠΎΠΌ, ΡƒΠ²Π΅Π»ΠΈΡ‡ΠΈΡ‚Π΅ силу трСния.

ΠžΠ±Ρ‰Π°Ρ сила (F), Π΄Π΅ΠΉΡΡ‚Π²ΡƒΡŽΡ‰Π°Ρ Π½Π° ΠΎΠ±ΡŠΠ΅ΠΊΡ‚, ΠΏΠΎΠ΄Π²Π΅Ρ€ΠΆΠ΅Π½Π½Ρ‹ΠΉ Ρ‚Ρ€Π΅Π½ΠΈΡŽ, Ρ€Π°Π²Π½Π° суммС ΠΏΡ€ΠΈΠ»ΠΎΠΆΠ΅Π½Π½ΠΎΠΉ силы (F ΠΏΡ€ΠΈΠ»ΠΎΠΆΠ΅Π½ΠΈΠ΅ ) ΠΈ силы трСния (F fr ). Но ΠΏΠΎΡΠΊΠΎΠ»ΡŒΠΊΡƒ сила трСния противодСйствуСт двиТСнию, ΠΎΠ½Π° ΠΎΡ‚Ρ€ΠΈΡ†Π°Ρ‚Π΅Π»ΡŒΠ½Π° ΠΏΠΎ ΠΎΡ‚Π½ΠΎΡˆΠ΅Π½ΠΈΡŽ ΠΊ прямой силС, поэтому:

F = F_ {app} -F_ {fr}

Π‘ΠΈΠ»Π° трСния являСтся ΠΏΡ€ΠΎΠΈΠ·Π²Π΅Π΄Π΅Π½ΠΈΠ΅ΠΌ коэффициСнта трСния ΠΈ Π½ΠΎΡ€ΠΌΠ°Π»ΡŒΠ½ΠΎΠΉ силы, которая ΠΏΡ€ΠΈ отсутствии Π΄ΠΎΠΏΠΎΠ»Π½ΠΈΡ‚Π΅Π»ΡŒΠ½Ρ‹Ρ… нисходящих сил — это вСс ΠΎΠ±ΡŠΠ΅ΠΊΡ‚Π°.ВСс (w) опрСдСляСтся ΠΊΠ°ΠΊ масса (ΠΌ) ΠΎΠ±ΡŠΠ΅ΠΊΡ‚Π°, умноТСнная Π½Π° силу тяТСсти (Π³):

F_N = w = mg

Π’Π΅ΠΏΠ΅Ρ€ΡŒ Π²Ρ‹ Π³ΠΎΡ‚ΠΎΠ²Ρ‹ Ρ€Π°ΡΡΡ‡ΠΈΡ‚Π°Ρ‚ΡŒ ускорСниС ΠΎΠ±ΡŠΠ΅ΠΊΡ‚Π° массой (ΠΌ). ΠΏΠΎΠ΄Π²Π΅Ρ€ΠΆΠ΅Π½ ΠΏΡ€ΠΈΠ»ΠΎΠΆΠ΅Π½Π½ΠΎΠΉ силС F app ΠΈ силС трСния. ΠŸΠΎΡΠΊΠΎΠ»ΡŒΠΊΡƒ ΠΎΠ±ΡŠΠ΅ΠΊΡ‚ двиТСтся, Π²Ρ‹ ΠΈΡΠΏΠΎΠ»ΡŒΠ·ΡƒΠ΅Ρ‚Π΅ коэффициСнт трСния скольТСния, Ρ‡Ρ‚ΠΎΠ±Ρ‹ ΠΏΠΎΠ»ΡƒΡ‡ΠΈΡ‚ΡŒ этот Ρ€Π΅Π·ΡƒΠ»ΡŒΡ‚Π°Ρ‚:

a = \ frac {F_ {app} — \ mu_ {sl} mg} {m}

ΠšΠΈΠ½Π΅Ρ‚ΠΈΡ‡Π΅ΡΠΊΠΎΠ΅ Ρ‚Ρ€Π΅Π½ΠΈΠ΅: ΠΎΠΏΡ€Π΅Π΄Π΅Π»Π΅Π½ΠΈΠ΅, коэффициСнт, Ρ„ΠΎΡ€ΠΌΡƒΠ»Π° (с ΠΏΡ€ΠΈΠΌΠ΅Ρ€Π°ΠΌΠΈ)

ОбновлСно 22 дСкабря 2020 Π³.

Π›ΠΈ ДТонсон

Π‘ΠΎΠ»ΡŒΡˆΠΈΠ½ΡΡ‚Π²ΠΎ ΠΎΠ±ΡŠΠ΅ΠΊΡ‚ΠΎΠ² Π½Π° самом Π΄Π΅Π»Π΅ Π½Π΅ Ρ‚Π°ΠΊΠΈΠ΅ Π³Π»Π°Π΄ΠΊΠΈΠ΅, ΠΊΠ°ΠΊ Π²Ρ‹ Π΄ΡƒΠΌΠ°Π΅Ρ‚Π΅.На микроскопичСском ΡƒΡ€ΠΎΠ²Π½Π΅ Π΄Π°ΠΆΠ΅ каТущиСся Π³Π»Π°Π΄ΠΊΠΈΠΌΠΈ повСрхности Π½Π° самом Π΄Π΅Π»Π΅ ΠΏΡ€Π΅Π΄ΡΡ‚Π°Π²Π»ΡΡŽΡ‚ собой Π»Π°Π½Π΄ΡˆΠ°Ρ„Ρ‚ ΠΊΡ€ΠΎΡˆΠ΅Ρ‡Π½Ρ‹Ρ… Ρ…ΠΎΠ»ΠΌΠΎΠ² ΠΈ Π΄ΠΎΠ»ΠΈΠ½, слишком ΠΌΠ°Π»Π΅Π½ΡŒΠΊΠΈΡ…, Ρ‡Ρ‚ΠΎΠ±Ρ‹ ΠΈΡ… ΠΌΠΎΠΆΠ½ΠΎ Π±Ρ‹Π»ΠΎ ΡƒΠ²ΠΈΠ΄Π΅Ρ‚ΡŒ, Π½ΠΎ ΠΊΠΎΡ‚ΠΎΡ€Ρ‹Π΅ ΠΈΠΌΠ΅ΡŽΡ‚ ΠΎΠ³Ρ€ΠΎΠΌΠ½ΠΎΠ΅ Π·Π½Π°Ρ‡Π΅Π½ΠΈΠ΅, ΠΊΠΎΠ³Π΄Π° Π΄Π΅Π»ΠΎ Π΄ΠΎΡ…ΠΎΠ΄ΠΈΡ‚ Π΄ΠΎ расчСта ΠΎΡ‚Π½ΠΎΡΠΈΡ‚Π΅Π»ΡŒΠ½ΠΎΠ³ΠΎ двиТСния ΠΌΠ΅ΠΆΠ΄Ρƒ двумя ΠΊΠΎΠ½Ρ‚Π°ΠΊΡ‚ΠΈΡ€ΡƒΡŽΡ‰ΠΈΠΌΠΈ повСрхностями.

Π­Ρ‚ΠΈ ΠΊΡ€ΠΎΡˆΠ΅Ρ‡Π½Ρ‹Π΅ Π΄Π΅Ρ„Π΅ΠΊΡ‚Ρ‹ повСрхностСй ΡΡ†Π΅ΠΏΠ»ΡΡŽΡ‚ΡΡ Π΄Ρ€ΡƒΠ³ с Π΄Ρ€ΡƒΠ³ΠΎΠΌ, создавая силу трСния, которая дСйствуСт Π² Π½Π°ΠΏΡ€Π°Π²Π»Π΅Π½ΠΈΠΈ, ΠΏΡ€ΠΎΡ‚ΠΈΠ²ΠΎΠΏΠΎΠ»ΠΎΠΆΠ½ΠΎΠΌ Π»ΡŽΠ±ΠΎΠΌΡƒ двиТСнию, ΠΈ Π΄ΠΎΠ»ΠΆΠ½Π° Π±Ρ‹Ρ‚ΡŒ рассчитана для опрСдСлСния Ρ€Π΅Π·ΡƒΠ»ΡŒΡ‚ΠΈΡ€ΡƒΡŽΡ‰Π΅ΠΉ силы, Π΄Π΅ΠΉΡΡ‚Π²ΡƒΡŽΡ‰Π΅ΠΉ Π½Π° ΠΎΠ±ΡŠΠ΅ΠΊΡ‚.

БущСствуСт нСсколько Ρ€Π°Π·Π»ΠΈΡ‡Π½Ρ‹Ρ… Ρ‚ΠΈΠΏΠΎΠ² трСния, Π½ΠΎ кинСтичСскоС Ρ‚Ρ€Π΅Π½ΠΈΠ΅ ΠΈΠ½Π°Ρ‡Π΅ извСстно ΠΊΠ°ΠΊ Ρ‚Ρ€Π΅Π½ΠΈΠ΅ скольТСния , Π° Ρ‚Ρ€Π΅Π½ΠΈΠ΅ покоя воздСйствуСт Π½Π° ΠΎΠ±ΡŠΠ΅ΠΊΡ‚ Π΄ΠΎ Ρ‚ΠΎΠ³ΠΎ, ΠΊΠ°ΠΊ Π½Π°Ρ‡ΠΈΠ½Π°Π΅Ρ‚ Π΄Π²ΠΈΠΆΠ΅Π½ΠΈΠ΅, ΠΈ Ρ‚Ρ€Π΅Π½ΠΈΠ΅ качСния ΠΊΠΎΠ½ΠΊΡ€Π΅Ρ‚Π½ΠΎ относится ΠΊ катящимся ΠΎΠ±ΡŠΠ΅ΠΊΡ‚Π°ΠΌ, Ρ‚Π°ΠΊΠΈΠΌ ΠΊΠ°ΠΊ колСса.

Π˜Π·ΡƒΡ‡Π΅Π½ΠΈΠ΅ Ρ‚ΠΎΠ³ΠΎ, Ρ‡Ρ‚ΠΎ Ρ‚Π°ΠΊΠΎΠ΅ кинСтичСскоС Ρ‚Ρ€Π΅Π½ΠΈΠ΅, ΠΊΠ°ΠΊ Π½Π°ΠΉΡ‚ΠΈ подходящий коэффициСнт трСния ΠΈ ΠΊΠ°ΠΊ Π΅Π³ΠΎ Π²Ρ‹Ρ‡ΠΈΡΠ»ΠΈΡ‚ΡŒ, расскаТСт Π²Π°ΠΌ всС, Ρ‡Ρ‚ΠΎ Π²Π°ΠΌ Π½ΡƒΠΆΠ½ΠΎ Π·Π½Π°Ρ‚ΡŒ для Ρ€Π΅ΡˆΠ΅Π½ΠΈΡ физичСских Π·Π°Π΄Π°Ρ‡, связанных с силой трСния.

ΠžΠΏΡ€Π΅Π΄Π΅Π»Π΅Π½ΠΈΠ΅ кинСтичСского трСния

НаиболСС прямоС ΠΎΠΏΡ€Π΅Π΄Π΅Π»Π΅Π½ΠΈΠ΅ кинСтичСского трСния: сопротивлСниС двиТСнию, Π²Ρ‹Π·Π²Π°Π½Π½ΠΎΠ΅ ΠΊΠΎΠ½Ρ‚Π°ΠΊΡ‚ΠΎΠΌ ΠΌΠ΅ΠΆΠ΄Ρƒ ΠΏΠΎΠ²Π΅Ρ€Ρ…Π½ΠΎΡΡ‚ΡŒΡŽ ΠΈ двиТущимся ΠΏΠΎ Π½Π΅ΠΉ ΠΎΠ±ΡŠΠ΅ΠΊΡ‚ΠΎΠΌ. Π‘ΠΈΠ»Π° кинСтичСского трСния дСйствуСт Ρ‚Π°ΠΊ, Ρ‡Ρ‚ΠΎ противодСйствуСт двиТСнию ΠΎΠ±ΡŠΠ΅ΠΊΡ‚Π°, поэтому, Ссли Π²Ρ‹ Ρ‚ΠΎΠ»ΠΊΠ°Π΅Ρ‚Π΅ Ρ‡Ρ‚ΠΎ-Ρ‚ΠΎ Π²ΠΏΠ΅Ρ€Π΅Π΄, Ρ‚Ρ€Π΅Π½ΠΈΠ΅ Ρ‚ΠΎΠ»ΠΊΠ°Π΅Ρ‚ Π΅Π³ΠΎ Π½Π°Π·Π°Π΄.

ΠšΠΈΠ½Π΅Ρ‚ΠΈΡ‡Π΅ΡΠΊΠ°Ρ фантастичСская сила примСняСтся Ρ‚ΠΎΠ»ΡŒΠΊΠΎ ΠΊ двиТущСмуся ΠΎΠ±ΡŠΠ΅ΠΊΡ‚Ρƒ (ΠΎΡ‚ΡΡŽΠ΄Π° «кинСтичСский») ΠΈ ΠΈΠ½Π°Ρ‡Π΅ извСстна ΠΊΠ°ΠΊ Ρ‚Ρ€Π΅Π½ΠΈΠ΅ скольТСния. Π­Ρ‚ΠΎ сила, которая противодСйствуСт двиТСнию скольТСния (Ρ‚ΠΎΠ»ΠΊΠ°Π½ΠΈΡŽ ΠΊΠΎΡ€ΠΎΠ±ΠΊΠΈ ΠΏΠΎ ΠΏΠΎΠ»ΠΎΠ²ΠΈΡ†Π°ΠΌ), ΠΈ ΡΡƒΡ‰Π΅ΡΡ‚Π²ΡƒΡŽΡ‚ ΡΠΏΠ΅Ρ†ΠΈΠ°Π»ΡŒΠ½Ρ‹Π΅ коэффициСнты трСния для этого ΠΈ Π΄Ρ€ΡƒΠ³ΠΈΡ… Ρ‚ΠΈΠΏΠΎΠ² трСния (Π½Π°ΠΏΡ€ΠΈΠΌΠ΅Ρ€, трСния качСния).

Π”Ρ€ΡƒΠ³ΠΈΠΌ основным Ρ‚ΠΈΠΏΠΎΠΌ трСния ΠΌΠ΅ΠΆΠ΄Ρƒ Ρ‚Π²Π΅Ρ€Π΄Ρ‹ΠΌΠΈ Ρ‚Π΅Π»Π°ΠΌΠΈ являСтся Ρ‚Ρ€Π΅Π½ΠΈΠ΅ покоя, ΠΈ это сопротивлСниС двиТСнию, Π²Ρ‹Π·Π²Π°Π½Π½ΠΎΠ΅ Ρ‚Ρ€Π΅Π½ΠΈΠ΅ΠΌ ΠΌΠ΅ΠΆΠ΄Ρƒ Π½Π΅ΠΏΠΎΠ΄Π²ΠΈΠΆΠ½Ρ‹ΠΌ ΠΎΠ±ΡŠΠ΅ΠΊΡ‚ΠΎΠΌ ΠΈ ΠΏΠΎΠ²Π΅Ρ€Ρ…Π½ΠΎΡΡ‚ΡŒΡŽ.ΠšΠΎΡΡ„Ρ„ΠΈΡ†ΠΈΠ΅Π½Ρ‚ трСния покоя ΠΎΠ±Ρ‹Ρ‡Π½ΠΎ большС, Ρ‡Π΅ΠΌ коэффициСнт кинСтичСского трСния, Ρ‡Ρ‚ΠΎ ΡƒΠΊΠ°Π·Ρ‹Π²Π°Π΅Ρ‚ Π½Π° Ρ‚ΠΎ, Ρ‡Ρ‚ΠΎ сила трСния слабСС для ΠΎΠ±ΡŠΠ΅ΠΊΡ‚ΠΎΠ², ΠΊΠΎΡ‚ΠΎΡ€Ρ‹Π΅ ΡƒΠΆΠ΅ находятся Π² Π΄Π²ΠΈΠΆΠ΅Π½ΠΈΠΈ.

Π£Ρ€Π°Π²Π½Π΅Π½ΠΈΠ΅ кинСтичСского трСния

Π‘ΠΈΠ»Ρƒ трСния Π»ΡƒΡ‡ΡˆΠ΅ всСго ΠΎΠΏΡ€Π΅Π΄Π΅Π»ΠΈΡ‚ΡŒ с ΠΏΠΎΠΌΠΎΡ‰ΡŒΡŽ уравнСния. Π‘ΠΈΠ»Π° трСния зависит ΠΎΡ‚ коэффициСнта трСния для рассматриваСмого Ρ‚ΠΈΠΏΠ° трСния ΠΈ Π²Π΅Π»ΠΈΡ‡ΠΈΠ½Ρ‹ Π½ΠΎΡ€ΠΌΠ°Π»ΡŒΠ½ΠΎΠΉ силы, ΠΊΠΎΡ‚ΠΎΡ€ΡƒΡŽ ΠΏΠΎΠ²Π΅Ρ€Ρ…Π½ΠΎΡΡ‚ΡŒ ΠΎΠΊΠ°Π·Ρ‹Π²Π°Π΅Ρ‚ Π½Π° ΠΎΠ±ΡŠΠ΅ΠΊΡ‚.Для трСния скольТСния сила трСния опрСдСляСтся ΠΊΠ°ΠΊ:

F_k = ΞΌ_k F_n

Π“Π΄Π΅ F k — сила кинСтичСского трСния, ΞΌ k — коэффициСнт трСния скольТСния ( ΠΈΠ»ΠΈ кинСтичСскоС Ρ‚Ρ€Π΅Π½ΠΈΠ΅) ΠΈ F n — Π½ΠΎΡ€ΠΌΠ°Π»ΡŒΠ½Π°Ρ сила, равная вСсу ΠΎΠ±ΡŠΠ΅ΠΊΡ‚Π°, Ссли ΠΏΡ€ΠΎΠ±Π»Π΅ΠΌΠ° связана с Π³ΠΎΡ€ΠΈΠ·ΠΎΠ½Ρ‚Π°Π»ΡŒΠ½ΠΎΠΉ ΠΏΠΎΠ²Π΅Ρ€Ρ…Π½ΠΎΡΡ‚ΡŒΡŽ ΠΈ Π½ΠΈΠΊΠ°ΠΊΠΈΠ΅ Π΄Ρ€ΡƒΠ³ΠΈΠ΅ Π²Π΅Ρ€Ρ‚ΠΈΠΊΠ°Π»ΡŒΠ½Ρ‹Π΅ силы Π½Π΅ Π΄Π΅ΠΉΡΡ‚Π²ΡƒΡŽΡ‚ (Ρ‚. Π΅. F n = ΠΌΠ³ , Π³Π΄Π΅ ΠΌ, — масса ΠΎΠ±ΡŠΠ΅ΠΊΡ‚Π°, Π° Π³, — ускорСниС свободного падСния).ΠŸΠΎΡΠΊΠΎΠ»ΡŒΠΊΡƒ Ρ‚Ρ€Π΅Π½ΠΈΠ΅ — это сила, Π΅Π΄ΠΈΠ½ΠΈΡ†Π΅ΠΉ силы трСния являСтся Π½ΡŒΡŽΡ‚ΠΎΠ½ (Н). ΠšΠΎΡΡ„Ρ„ΠΈΡ†ΠΈΠ΅Π½Ρ‚ кинСтичСского трСния Π±Π΅Π·Ρ€Π°Π·ΠΌΠ΅Ρ€Π΅Π½.

Π£Ρ€Π°Π²Π½Π΅Π½ΠΈΠ΅ статичСского трСния Π² основном Ρ‚Π°ΠΊΠΎΠ΅ ΠΆΠ΅, Π·Π° ΠΈΡΠΊΠ»ΡŽΡ‡Π΅Π½ΠΈΠ΅ΠΌ Ρ‚ΠΎΠ³ΠΎ, Ρ‡Ρ‚ΠΎ коэффициСнт трСния скольТСния Π·Π°ΠΌΠ΅Π½Π΅Π½ коэффициСнтом статичСского трСния ( ΞΌ s ). Π­Ρ‚ΠΎ Π΄Π΅ΠΉΡΡ‚Π²ΠΈΡ‚Π΅Π»ΡŒΠ½ΠΎ Π»ΡƒΡ‡ΡˆΠ΅ всСго Ρ€Π°ΡΡΠΌΠ°Ρ‚Ρ€ΠΈΠ²Π°Ρ‚ΡŒ ΠΊΠ°ΠΊ максимальноС Π·Π½Π°Ρ‡Π΅Π½ΠΈΠ΅, ΠΏΠΎΡ‚ΠΎΠΌΡƒ Ρ‡Ρ‚ΠΎ ΠΎΠ½ΠΎ увСличиваСтся Π΄ΠΎ ΠΎΠΏΡ€Π΅Π΄Π΅Π»Π΅Π½Π½ΠΎΠΉ Ρ‚ΠΎΡ‡ΠΊΠΈ, Π° Π·Π°Ρ‚Π΅ΠΌ, Ссли Π²Ρ‹ ΠΏΡ€ΠΈΠ»ΠΎΠΆΠΈΡ‚Π΅ большС силы ΠΊ ΠΎΠ±ΡŠΠ΅ΠΊΡ‚Ρƒ, ΠΎΠ½ Π½Π°Ρ‡Π½Π΅Ρ‚ Π΄Π²ΠΈΠ³Π°Ρ‚ΡŒΡΡ:

F_s \ leq ΞΌ_s F_n

РасчСты с кинСтичСским Ρ‚Ρ€Π΅Π½ΠΈΠ΅ΠΌ

ΠžΠΏΡ€Π΅Π΄Π΅Π»ΠΈΡ‚ΡŒ ΠΊΠΈΠ½Π΅Ρ‚ΠΈΡ‡Π΅ΡΠΊΡƒΡŽ силу трСния нСслоТно Π½Π° Π³ΠΎΡ€ΠΈΠ·ΠΎΠ½Ρ‚Π°Π»ΡŒΠ½ΠΎΠΉ повСрхности, Π½ΠΎ Π½Π΅ΠΌΠ½ΠΎΠ³ΠΎ слоТнСС Π½Π° Π½Π°ΠΊΠ»ΠΎΠ½Π½ΠΎΠΉ повСрхности.2 \\ & = 7.85 \; \ text {N} \ end {align}

Π’Π΅ΠΏΠ΅Ρ€ΡŒ ΠΏΡ€Π΅Π΄ΡΡ‚Π°Π²ΡŒΡ‚Π΅ Ρ‚Ρƒ ΠΆΠ΅ ΡΠΈΡ‚ΡƒΠ°Ρ†ΠΈΡŽ, Π·Π° ΠΈΡΠΊΠ»ΡŽΡ‡Π΅Π½ΠΈΠ΅ΠΌ Ρ‚ΠΎΠ³ΠΎ, Ρ‡Ρ‚ΠΎ ΠΏΠΎΠ²Π΅Ρ€Ρ…Π½ΠΎΡΡ‚ΡŒ Π½Π°ΠΊΠ»ΠΎΠ½Π΅Π½Π° ΠΏΠΎΠ΄ ΡƒΠ³Π»ΠΎΠΌ 20 градусов ΠΊ Π³ΠΎΡ€ΠΈΠ·ΠΎΠ½Ρ‚Π°Π»ΠΈ. ΠΠΎΡ€ΠΌΠ°Π»ΡŒΠ½Π°Ρ сила зависит ΠΎΡ‚ ΡΠΎΡΡ‚Π°Π²Π»ΡΡŽΡ‰Π΅ΠΉ вСса ΠΎΠ±ΡŠΠ΅ΠΊΡ‚Π°, Π½Π°ΠΏΡ€Π°Π²Π»Π΅Π½Π½ΠΎΠ³ΠΎ пСрпСндикулярно повСрхности, ΠΊΠΎΡ‚ΠΎΡ€Ρ‹ΠΉ задаСтся Ρ„ΠΎΡ€ΠΌΡƒΠ»ΠΎΠΉ ΠΌΠ³ cos ( ΞΈ ), Π³Π΄Π΅ ΞΈ — это ΡƒΠ³ΠΎΠ» Π½Π°ΠΊΠ»ΠΎΠ½Π°. ΠžΠ±Ρ€Π°Ρ‚ΠΈΡ‚Π΅ Π²Π½ΠΈΠΌΠ°Π½ΠΈΠ΅, Ρ‡Ρ‚ΠΎ mg sin ( ΞΈ ) сообщаСт Π²Π°ΠΌ силу тяТСсти, Ρ‚ΡΠ½ΡƒΡ‰ΡƒΡŽ Π΅Π³ΠΎ Π²Π½ΠΈΠ· ΠΏΠΎ склону.2 Γ— \ cos (20 Β°) \\ & = 7.37 \; \ text {N} \ end {align}

Π’Ρ‹ Ρ‚Π°ΠΊΠΆΠ΅ ΠΌΠΎΠΆΠ΅Ρ‚Π΅ Ρ€Π°ΡΡΡ‡ΠΈΡ‚Π°Ρ‚ΡŒ коэффициСнт статичСского трСния с ΠΏΠΎΠΌΠΎΡ‰ΡŒΡŽ простого экспСримСнта. ΠŸΡ€Π΅Π΄ΡΡ‚Π°Π²ΡŒΡ‚Π΅, Ρ‡Ρ‚ΠΎ Π²Ρ‹ ΠΏΡ‹Ρ‚Π°Π΅Ρ‚Π΅ΡΡŒ Ρ‚ΠΎΠ»ΠΊΠ°Ρ‚ΡŒ ΠΈΠ»ΠΈ Ρ‚ΡΠ½ΡƒΡ‚ΡŒ 5-ΠΊΠΈΠ»ΠΎΠ³Ρ€Π°ΠΌΠΌΠΎΠ²Ρ‹ΠΉ дСрСвянный брусок ΠΏΠΎ Π±Π΅Ρ‚ΠΎΠ½Ρƒ. Если Π²Ρ‹ записываСтС ΠΏΡ€ΠΈΠ»ΠΎΠΆΠ΅Π½Π½ΡƒΡŽ силу Π² Ρ‚ΠΎΡ‚ ΠΌΠΎΠΌΠ΅Π½Ρ‚, ΠΊΠΎΠ³Π΄Π° ΠΊΠΎΡ€ΠΎΠ±ΠΊΠ° Π½Π°Ρ‡ΠΈΠ½Π°Π΅Ρ‚ Π΄Π²ΠΈΠ³Π°Ρ‚ΡŒΡΡ, Π²Ρ‹ ΠΌΠΎΠΆΠ΅Ρ‚Π΅ ΠΈΠ·ΠΌΠ΅Π½ΠΈΡ‚ΡŒ ΡƒΡ€Π°Π²Π½Π΅Π½ΠΈΠ΅ статичСского трСния, Ρ‡Ρ‚ΠΎΠ±Ρ‹ Π½Π°ΠΉΡ‚ΠΈ подходящий коэффициСнт трСния для Π΄Π΅Ρ€Π΅Π²Π° ΠΈ камня. Если для пСрСмСщСния Π±Π»ΠΎΠΊΠ° трСбуСтся сила 30 Н, Ρ‚ΠΎ максимум для F с = 30 Н, поэтому:

F_s = ΞΌ_s F_n

\ begin {align} ΞΌ_s & = \ frac {F_s } {F_n} \\ & = \ frac {F_s} {mg} \\ & = \ frac {30 \; \ text {N}} {5 \; \ text {kg} Γ— 9.2} \\ & = \ frac {30 \; \ text {N}} {49,05 \; \ text {N}} \\ & = 0,61 \ end {align}

Π’Π°ΠΊΠΈΠΌ ΠΎΠ±Ρ€Π°Π·ΠΎΠΌ, коэффициСнт составляСт ΠΎΠΊΠΎΠ»ΠΎ 0,61.

Как Ρ€Π°ΡΡΡ‡ΠΈΡ‚Π°Ρ‚ΡŒ силу трСния — x-engineer.org

Когда Π»ΡŽΠ±Ρ‹Π΅ Π΄Π²Π° Ρ‚Π΅Π»Π° Π²ΡΡ‚ΡƒΠΏΠ°ΡŽΡ‚ Π² ΠΊΠΎΠ½Ρ‚Π°ΠΊΡ‚ ΠΈ ΠΌΠ΅ΠΆΠ΄Ρƒ Π½ΠΈΠΌΠΈ происходит ΠΎΡ‚Π½ΠΎΡΠΈΡ‚Π΅Π»ΡŒΠ½ΠΎΠ΅ Π΄Π²ΠΈΠΆΠ΅Π½ΠΈΠ΅, Π²ΠΎΠ·Π½ΠΈΠΊΠ°Π΅Ρ‚ Ρ‚Ρ€Π΅Π½ΠΈΠ΅ . Π’Ρ€Π΅Π½ΠΈΠ΅ ΠΌΠΎΠΆΠ½ΠΎ ΠΎΠΏΡ€Π΅Π΄Π΅Π»ΠΈΡ‚ΡŒ ΠΊΠ°ΠΊ силу, которая противодСйствуСт двиТСнию Π΄Π²ΡƒΡ… ΠΊΠΎΠ½Ρ‚Π°ΠΊΡ‚ΠΈΡ€ΡƒΡŽΡ‰ΠΈΡ… повСрхностСй, ΠΊΠΎΡ‚ΠΎΡ€Ρ‹Π΅ ΡΠΊΠΎΠ»ΡŒΠ·ΡΡ‚ ΠΎΡ‚Π½ΠΎΡΠΈΡ‚Π΅Π»ΡŒΠ½ΠΎ Π΄Ρ€ΡƒΠ³ Π΄Ρ€ΡƒΠ³Π°.

Π’Ρ€Π΅Π½ΠΈΠ΅ являСтся Ρ‡Π°ΡΡ‚ΡŒΡŽ нашСй повсСднСвной ΠΆΠΈΠ·Π½ΠΈ ΠΈ позволяСт Π²Ρ‹ΠΏΠΎΠ»Π½ΡΡ‚ΡŒ ΠΎΠΏΡ€Π΅Π΄Π΅Π»Π΅Π½Π½Ρ‹Π΅ дСйствия.НапримСр, для Ρ…ΠΎΠ΄ΡŒΠ±Ρ‹ ΠΈ Π±Π΅Π³Π° трСбуСтся Ρ‚Ρ€Π΅Π½ΠΈΠ΅, Π° для ускорСния ΠΈΠ»ΠΈ тормоТСния автомобиля Π½Π° Π΄ΠΎΡ€ΠΎΠ³Π΅ трСбуСтся Ρ‚Ρ€Π΅Π½ΠΈΠ΅. Π’ΠΎΡ€ΠΌΠΎΠ·Π½Ρ‹Π΅ систСмы, систСмы сцСплСния, ΠΈΡΠΏΠΎΠ»ΡŒΠ·ΡƒΠ΅ΠΌΡ‹Π΅ Π² транспортных срСдствах, Ρ‚Π°ΠΊΠΆΠ΅ основаны Π½Π° ΠΏΡ€ΠΈΠ½Ρ†ΠΈΠΏΠ°Ρ… трСния.

Π‘ΠΈΠ»Π° трСния зависит ΠΎΡ‚ Ρ‚ΠΎΠ³ΠΎ, насколько Π³Π»Π°Π΄ΠΊΠΈΠΌΠΈ ΠΈΠ»ΠΈ ΡˆΠ΅Ρ€ΠΎΡ…ΠΎΠ²Π°Ρ‚Ρ‹ΠΌΠΈ ΡΠ²Π»ΡΡŽΡ‚ΡΡ ΠΊΠΎΠ½Ρ‚Π°ΠΊΡ‚ΠΈΡ€ΡƒΡŽΡ‰ΠΈΠ΅ повСрхности. На микроскопичСском ΡƒΡ€ΠΎΠ²Π½Π΅ Π΄Π°ΠΆΠ΅ самыС Π³Π»Π°Π΄ΠΊΠΈΠ΅ повСрхности ΠΈΠΌΠ΅ΡŽΡ‚ нСровности, ΠΊΠΎΡ‚ΠΎΡ€Ρ‹Π΅ ΡΠΎΠΏΡ€ΠΈΠΊΠ°ΡΠ°ΡŽΡ‚ΡΡ ΠΈ ΡΡ†Π΅ΠΏΠ»ΡΡŽΡ‚ΡΡ Π΄Ρ€ΡƒΠ³ с Π΄Ρ€ΡƒΠ³ΠΎΠΌ. На изобраТСниях Π½ΠΈΠΆΠ΅ Π²Ρ‹ ΠΌΠΎΠΆΠ΅Ρ‚Π΅ ясно ΡƒΠ²ΠΈΠ΄Π΅Ρ‚ΡŒ, ΠΊΠ°ΠΊ микроструктура повСрхности трСния ΠΏΡ€Π΅Π΄ΠΎΡ‚Π²Ρ€Π°Ρ‰Π°Π΅Ρ‚ ΠΏΠ΅Ρ€Π΅ΠΌΠ΅Ρ‰Π΅Π½ΠΈΠ΅ ΠΌΠ΅ΠΆΠ΄Ρƒ двумя ΠΎΠ±ΡŠΠ΅ΠΊΡ‚Π°ΠΌΠΈ (Ρ‚Π²Π΅Ρ€Π΄Ρ‹ΠΌΠΈ Ρ‚Π΅Π»Π°ΠΌΠΈ).

Π˜Π·ΠΎΠ±Ρ€Π°ΠΆΠ΅Π½ΠΈΠ΅: ΠšΠΎΠ½Ρ‚Π°ΠΊΡ‚ ΠΌΠ΅ΠΆΠ΄Ρƒ двумя повСрхностями

Π˜Π·ΠΎΠ±Ρ€Π°ΠΆΠ΅Π½ΠΈΠ΅: ΠœΠΈΠΊΡ€ΠΎΡΡ‚Ρ€ΡƒΠΊΡ‚ΡƒΡ€Π° повСрхности трСния

Π’ зависимости ΠΎΡ‚ состояния ΡΠΊΠΎΠ»ΡŒΠ·ΡΡ‰ΠΈΡ… повСрхностСй Ρ€Π°Π·Π»ΠΈΡ‡Π°ΡŽΡ‚ нСсколько Ρ‚ΠΈΠΏΠΎΠ² трСния:

Π‘ΡƒΡ…ΠΎΠ΅ Ρ‚Ρ€Π΅Π½ΠΈΠ΅ , ΠΊΠ°ΠΊ слСдуСт ΠΈΠ· названия, Π²ΠΎΠ·Π½ΠΈΠΊΠ°Π΅Ρ‚ ΠΌΠ΅ΠΆΠ΄Ρƒ двумя сухими Ρ‚Π²Π΅Ρ€Π΄Ρ‹ΠΌΠΈ Ρ‚Π΅Π»Π°ΠΌΠΈ Π±Π΅Π· ΠΊΠ°ΠΊΠΎΠΉ-Π»ΠΈΠ±ΠΎ смазки (масла, консистСнтной смазки ΠΈ Ρ‚. Π΄.) ΠΌΠ΅ΠΆΠ΄Ρƒ Π½ΠΈΠΌΠΈ. Одним ΠΈΠ· ΠΏΡ€ΠΈΠΌΠ΅Ρ€ΠΎΠ² сухого трСния являСтся тормозная систСма транспортного срСдства. ΠœΠ΅ΠΆΠ΄Ρƒ Ρ‚ΠΎΡ€ΠΌΠΎΠ·Π½Ρ‹ΠΌ диском ΠΈ Ρ‚ΠΎΡ€ΠΌΠΎΠ·Π½ΠΎΠΉ ΠΊΠΎΠ»ΠΎΠ΄ΠΊΠΎΠΉ имССтся прямой ΠΊΠΎΠ½Ρ‚Π°ΠΊΡ‚ Π±Π΅Π· ΠΊΠ°ΠΊΠΎΠΉ-Π»ΠΈΠ±ΠΎ смазки.Π‘ΡƒΡ…ΠΎΠ΅ Ρ‚Ρ€Π΅Π½ΠΈΠ΅ Ρ‚Π°ΠΊΠΆΠ΅ извСстно ΠΊΠ°ΠΊ ΠšΡƒΠ»ΠΎΠ½ΠΎΠ²ΡΠΊΠΎΠ΅ Ρ‚Ρ€Π΅Π½ΠΈΠ΅ , Π² Ρ‡Π΅ΡΡ‚ΡŒ французского Ρ„ΠΈΠ·ΠΈΠΊΠ° Шарля-ΠžΠ³ΡŽΡΡ‚Π΅Π½Π° Π΄Π΅ ΠšΡƒΠ»ΠΎΠ½Π°, ΠΊΠΎΡ‚ΠΎΡ€Ρ‹ΠΉ Π³Π»ΡƒΠ±ΠΎΠΊΠΎ ΠΈΠ·ΡƒΡ‡Π°Π» сухоС Ρ‚Ρ€Π΅Π½ΠΈΠ΅.

Π’Ρ€Π΅Π½ΠΈΠ΅ Π²ΠΎ Π²Π»Π°ΠΆΠ½ΠΎΠΌ состоянии , Π²ΠΎΠ·Π½ΠΈΠΊΠ°Π΅Ρ‚, ΠΊΠΎΠ³Π΄Π° ΠΌΠ΅ΠΆΠ΄Ρƒ повСрхностями скольТСния находится смазка. ΠŸΡ€ΠΈΠΌΠ΅Ρ€Ρ‹ ΠΌΠΎΠΊΡ€ΠΎΠ³ΠΎ трСния: ΠΏΠΎΡ€ΡˆΠ½Π΅Π²Ρ‹Π΅ ΠΊΠΎΠ»ΡŒΡ†Π° Π²Π½ΡƒΡ‚Ρ€ΠΈ Ρ†ΠΈΠ»ΠΈΠ½Π΄Ρ€Π°, насосныС элСмСнты Π²Π½ΡƒΡ‚Ρ€ΠΈ Ρ‚ΠΎΠΏΠ»ΠΈΠ²Π½ΠΎΠ³ΠΎ насоса, ΠΌΡƒΡ„Ρ‚Π° Π±Π»ΠΎΠΊΠΈΡ€ΠΎΠ²ΠΊΠΈ гидротрансформатора ΠΈ Ρ‚. Π”.

Π’ этой ΡΡ‚Π°Ρ‚ΡŒΠ΅ ΠΌΡ‹ сосрСдоточимся Π½Π° характСристиках сухого трСния ΠΈ способах расчСта сила трСния.

Если Ρƒ нас Π΅ΡΡ‚ΡŒ Ρ‚Π΅Π»ΠΎ Π½Π° Ρ‚Π²Π΅Ρ€Π΄ΠΎΠΉ повСрхности, ΠΈ ΠΌΡ‹ ΠΏΡ€ΠΈΠ»ΠΎΠΆΠΈΠΌ ΠΊ Π½Π΅ΠΌΡƒ Ρ‚ΠΎΠ»ΠΊΠ°ΡŽΡ‰ΡƒΡŽ силу, Ρ‚Π΅Π»ΠΎ Π½Π΅ Π±ΡƒΠ΄Π΅Ρ‚ Π΄Π²ΠΈΠ³Π°Ρ‚ΡŒΡΡ. Π­Ρ‚ΠΎ происходит ΠΏΠΎΡ‚ΠΎΠΌΡƒ, Ρ‡Ρ‚ΠΎ сила трСния ΠΌΠ΅ΠΆΠ΄Ρƒ Ρ‚Π΅Π»ΠΎΠΌ ΠΈ ΠΏΠΎΠ²Π΅Ρ€Ρ…Π½ΠΎΡΡ‚ΡŒΡŽ ΠΏΡ€ΠΎΡ‚ΠΈΠ²ΠΎΠΏΠΎΠ»ΠΎΠΆΠ½Π° Ρ‚ΠΎΠ»ΠΊΠ°ΡŽΡ‰Π΅ΠΉ силС ΠΈ ΡƒΠ΄Π΅Ρ€ΠΆΠΈΠ²Π°Π΅Ρ‚ Ρ‚Π΅Π»ΠΎ Π½Π° мСстС.

Π˜Π·ΠΎΠ±Ρ€Π°ΠΆΠ΅Π½ΠΈΠ΅: РавновСсиС Ρ‚Π΅Π»Π° с Ρ‚Ρ€Π΅Π½ΠΈΠ΅ΠΌ

Π³Π΄Π΅:
F p [N] — Ρ‚ΠΎΠ»ΠΊΠ°ΡŽΡ‰Π°Ρ сила
F f [N] — сила трСния
G [N] — вСс Ρ‚Π΅Π»Π°
N [-] — Π½ΠΎΡ€ΠΌΠ°Π»ΡŒΠ½Π°Ρ рСакция

Из Π΄ΠΈΠ°Π³Ρ€Π°ΠΌΠΌΡ‹ свободного Ρ‚Π΅Π»Π°, ΠΏΠΎΠΊΠ° Ρ‚Π΅Π»ΠΎ находится Π² равновСсии (Π½Π΅ двиТСтся), ΠΌΡ‹ ΠΌΠΎΠΆΠ΅ΠΌ Π·Π°ΠΏΠΈΡΠ°Ρ‚ΡŒ уравнСния равновСсия сил для ΠΎΠ±Π΅ΠΈΡ… осСй (x ΠΈ y).

Π“ΠΎΡ€ΠΈΠ·ΠΎΠ½Ρ‚Π°Π»ΡŒΠ½ΠΎΠ΅ равновСсиС:

\ [\ sum F_ {x} = 0 \]
\ [F_ {f} -F_ {p} = 0 \ tag {1} \]
\ [F_ {f} = F_ { p} \ tag {2} \]

Π’Π΅Ρ€Ρ‚ΠΈΠΊΠ°Π»ΡŒΠ½ΠΎΠ΅ равновСсиС:

\ [\ sum F_ {y} = 0 \]
\ [NG = 0 \ tag {3} \]
\ [N = G \ tag { 4} \]

Π£Ρ€Π°Π²Π½Π΅Π½ΠΈΠ΅ (1) Π²Π΅Ρ€Π½ΠΎ Π΄ΠΎ Ρ‚Π΅Ρ… ΠΏΠΎΡ€, ΠΏΠΎΠΊΠ° Ρ‚Π΅Π»ΠΎ Π½Π΅ двиТСтся. Вопрос Π² Ρ‚ΠΎΠΌ, , с ΠΊΠ°ΠΊΠΎΠΉ силой Π½ΡƒΠΆΠ½ΠΎ Ρ‚ΠΎΠ»ΠΊΠ½ΡƒΡ‚ΡŒ Ρ‚Π΅Π»ΠΎ, Ρ‡Ρ‚ΠΎΠ±Ρ‹ ΠΎΠ½ΠΎ Π½Π°Ρ‡Π°Π»ΠΎ Π΄Π²ΠΈΠ³Π°Ρ‚ΡŒΡΡ?

Π‘ΠΈΠ»Π° трСния являСтся Ρ€Π΅Π°ΠΊΡ†ΠΈΠ΅ΠΉ Π½Π° Ρ‚ΠΎΠ»ΠΊΠ°ΡŽΡ‰ΡƒΡŽ силу.Если Π½Π΅Ρ‚ Ρ‚ΠΎΠ»ΠΊΠ°ΡŽΡ‰Π΅ΠΉ силы, Π½Π΅Ρ‚ ΠΈ силы трСния. Если ΠΌΡ‹ Π±ΡƒΠ΄Π΅ΠΌ ΠΏΡ€ΠΎΠ΄ΠΎΠ»ΠΆΠ°Ρ‚ΡŒ сильнСС Ρ‚ΠΎΠ»ΠΊΠ°Ρ‚ΡŒ Ρ‚Π΅Π»ΠΎ, ΠΎΠ½ΠΎ, Π² ΠΊΠΎΠ½Ρ†Π΅ ΠΊΠΎΠ½Ρ†ΠΎΠ², Π½Π°Ρ‡Π½Π΅Ρ‚ Π΄Π²ΠΈΠ³Π°Ρ‚ΡŒΡΡ. Π­Ρ‚ΠΎ происходит ΠΏΠΎΡ‚ΠΎΠΌΡƒ, Ρ‡Ρ‚ΠΎ наша Ρ‚ΠΎΠ»ΠΊΠ°ΡŽΡ‰Π°Ρ сила стала Π²Ρ‹ΡˆΠ΅ максимальной силы трСния, ΠΈ Ρ‚Π΅Π»ΠΎ большС Π½Π΅ находится Π² равновСсии.

МаксимальноС Π·Π½Π°Ρ‡Π΅Π½ΠΈΠ΅ силы трСния, Ρ‚Π°ΠΊΠΆΠ΅ Π½Π°Π·Ρ‹Π²Π°Π΅ΠΌΠΎΠΉ силой трСния покоя, зависит ΠΎΡ‚ Π½ΠΎΡ€ΠΌΠ°Π»ΡŒΠ½ΠΎΠΉ силы. ΠΠΎΡ€ΠΌΠ°Π»ΡŒΠ½Π°Ρ сила опрСдСляСтся ΠΊΠ°ΠΊ сила Ρ€Π΅Π°ΠΊΡ†ΠΈΠΈ стоячСй повСрхности Π½Π° Ρ‚Π΅Π»Π΅, вызванная силой вСса Ρ‚Π΅Π»Π°. Если Π±Ρ‹ Π½Π΅ Π±Ρ‹Π»ΠΎ Π½ΠΎΡ€ΠΌΠ°Π»ΡŒΠ½ΠΎΠΉ силы Ρ€Π΅Π°ΠΊΡ†ΠΈΠΈ для ΡƒΡ€Π°Π²Π½ΠΎΠ²Π΅ΡˆΠΈΠ²Π°Π½ΠΈΡ силы вСса Ρ‚Π΅Π»Π°, Ρ‚Π΅Π»ΠΎ ΠΏΠΎΠ³Ρ€ΡƒΠ·ΠΈΠ»ΠΎΡΡŒ Π±Ρ‹ Π² ΡΡ‚ΠΎΡΡ‡ΡƒΡŽ ΠΏΠΎΠ²Π΅Ρ€Ρ…Π½ΠΎΡΡ‚ΡŒ.

Π­ΠΊΡΠΏΠ΅Ρ€ΠΈΠΌΠ΅Π½Ρ‚Π°Π»ΡŒΠ½ΠΎ Π±Ρ‹Π»ΠΎ ΠΎΠΏΡ€Π΅Π΄Π΅Π»Π΅Π½ΠΎ, Ρ‡Ρ‚ΠΎ эта ΠΏΡ€Π΅Π΄Π΅Π»ΡŒΠ½Π°Ρ сила трСния покоя прямо ΠΏΡ€ΠΎΠΏΠΎΡ€Ρ†ΠΈΠΎΠ½Π°Π»ΡŒΠ½Π° Ρ€Π΅Π·ΡƒΠ»ΡŒΡ‚ΠΈΡ€ΡƒΡŽΡ‰Π΅ΠΉ Π½ΠΎΡ€ΠΌΠ°Π»ΡŒΠ½ΠΎΠΉ силС ΠΈ рассчитываСтся ΠΊΠ°ΠΊ:

\ [\ bbox [# FFFF9D] {F_ {f} = \ mu \ cdot N } \ tag {5} \]

Π³Π΄Π΅:
ΞΌ [-] — коэффициСнт трСния

Π§Ρ‚ΠΎΠ±Ρ‹ ΠΎΡ‚Π²Π΅Ρ‚ΠΈΡ‚ΡŒ Π½Π° поставлСнный Π²Ρ‹ΡˆΠ΅ вопрос, Ρ‚Π΅Π»ΠΎ Π½Π°Ρ‡Π½Π΅Ρ‚ Π΄Π²ΠΈΠ³Π°Ρ‚ΡŒΡΡ, ΠΊΠΎΠ³Π΄Π° Ρ‚ΠΎΠ»ΠΊΠ°ΡŽΡ‰Π°Ρ сила большС, Ρ‡Π΅ΠΌ сила трСния , которая, начиная с уравнСния (4) ΠΈ (5), ΠΎΠ·Π½Π°Ρ‡Π°Π΅Ρ‚:

\ [F_ {p} \ ge \ mu \ cdot G \ tag {6} \]

ΠœΡ‹ Π·Π½Π°Π΅ΠΌ, Ρ‡Ρ‚ΠΎ вСс Ρ‚Π΅Π»Π° рассчитываСтся ΠΊΠ°ΠΊ:

\ [G = m \ cdot g \ tag {7} \]

Π³Π΄Π΅:
ΠΌ [ΠΊΠ³] — масса транспортного срСдства
Π³ [ΠΌ / с 2 ] — ускорСниС свободного падСния

ОбъСдинСниС ΡƒΡ€Π°Π²Π½Π΅Π½ΠΈΠΉ (6) ΠΈ (7) Π΄Π°Π΅Ρ‚ Π·Π½Π°Ρ‡Π΅Π½ΠΈΠ΅ функция Ρ‚ΠΎΠ»ΠΊΠ°ΡŽΡ‰Π΅ΠΉ силы ΠΎΡ‚ массы Ρ‚Π΅Π»Π° ΠΈ коэффициСнта трСния:

\ [F_ {p} \ ge 9.81 \ cdot \ mu \ cdot m \ tag {8} \]

ΠšΠΎΡΡ„Ρ„ΠΈΡ†ΠΈΠ΅Π½Ρ‚ трСния зависит ΠΎΡ‚ ΠΎΡ‚Π½ΠΎΡΠΈΡ‚Π΅Π»ΡŒΠ½ΠΎΠ³ΠΎ двиТСния ΠΌΠ΅ΠΆΠ΄Ρƒ повСрхностями трСния. Если Π½Π΅Ρ‚ двиТСния, коэффициСнт трСния называСтся , коэффициСнт трСния покоя . ΠŸΡ€ΠΈ Π½Π°Π»ΠΈΡ‡ΠΈΠΈ двиТСния (скорости) ΠΌΠ΅ΠΆΠ΄Ρƒ повСрхностями трСния (Ρ‚Π΅Π»Π°ΠΌΠΈ) коэффициСнт трСния называСтся , коэффициСнт кинСтичСского трСния .

Π˜Π·ΠΎΠ±Ρ€Π°ΠΆΠ΅Π½ΠΈΠ΅: ΠšΠΎΡΡ„Ρ„ΠΈΡ†ΠΈΠ΅Π½Ρ‚ трСния, функция скорости

Π³Π΄Π΅:
v [ΠΌ / с] — ΠΎΡ‚Π½ΠΎΡΠΈΡ‚Π΅Π»ΡŒΠ½Π°Ρ ΡΠΊΠΎΡ€ΠΎΡΡ‚ΡŒ ΠΌΠ΅ΠΆΠ΄Ρƒ двумя ΠΊΠΎΠ½Ρ‚Π°ΠΊΡ‚ΠΈΡ€ΡƒΡŽΡ‰ΠΈΠΌΠΈ Ρ‚Π΅Π»Π°ΠΌΠΈ
ΞΌ с [-] — коэффициСнт статичСского трСния
ΞΌ k [- ] — коэффициСнт кинСтичСского трСния

Для Π΄Π²ΡƒΡ… Π΄Π°Π½Π½Ρ‹Ρ… Ρ‚Π΅Π» коэффициСнт трСния Π½Π΅ фиксирован, ΠΎΠ½ зависит ΠΎΡ‚ скорости ΠΎΡ‚Π½ΠΎΡΠΈΡ‚Π΅Π»ΡŒΠ½ΠΎΠ³ΠΎ двиТСния Ρ‚Π΅Π» Π² ΠΊΠΎΠ½Ρ‚Π°ΠΊΡ‚Π΅.МаксимальноС Π·Π½Π°Ρ‡Π΅Π½ΠΈΠ΅ коэффициСнта достигаСтся ΠΏΡ€ΠΈ отсутствии двиТСния (нулСвая ΡΠΊΠΎΡ€ΠΎΡΡ‚ΡŒ) ΠΌΠ΅ΠΆΠ΄Ρƒ повСрхностями скольТСния. Π’ этом случаС коэффициСнт кинСтичСского трСния становится коэффициСнтом трСния покоя.

ΠœΠ°Ρ‚Π΅ΠΌΠ°Ρ‚ΠΈΡ‡Π΅ΡΠΊΠΈ ΠΌΡ‹ ΠΌΠΎΠΆΠ΅ΠΌ ΠΎΠΏΡ€Π΅Π΄Π΅Π»ΠΈΡ‚ΡŒ коэффициСнт трСния ΠΊΠ°ΠΊ:

\ [\ begin {split}
\ mu (v) = \ left \ {\ begin {matrix}
\ mu_ {s} \ text {, if} v = 0 \\
\ mu_ {k} (v) \ text {, if} v> 0
\ end {matrix} \ right.
\ end {split} \]

На самом Π΄Π΅Π»Π΅ Π·Π½Π°Ρ‡Π΅Π½ΠΈΠ΅ кинСтичСского коэффициСнта трСния зависит ΠΎΡ‚ значСния ΠΎΡ‚Π½ΠΎΡΠΈΡ‚Π΅Π»ΡŒΠ½ΠΎΠΉ скорости повСрхностСй трСния.Π­Ρ‚ΠΎ происходит ΠΏΠΎΡ‚ΠΎΠΌΡƒ, Ρ‡Ρ‚ΠΎ Ρ‡Π΅ΠΌ Π²Ρ‹ΡˆΠ΅ ΠΎΡ‚Π½ΠΎΡΠΈΡ‚Π΅Π»ΡŒΠ½Π°Ρ ΡΠΊΠΎΡ€ΠΎΡΡ‚ΡŒ трСния, Ρ‚Π΅ΠΌ Π²Ρ‹ΡˆΠ΅ выдСляСмоС Ρ‚Π΅ΠΏΠ»ΠΎ, Ρ‚Π΅ΠΌ сильнСС ΠΈΠ·ΠΌΠ΅Π½ΡΡŽΡ‚ΡΡ свойства ΠΌΠ°Ρ‚Π΅Ρ€ΠΈΠ°Π»Π°. Для простоты Π² этой ΡΡ‚Π°Ρ‚ΡŒΠ΅ ΠΌΡ‹ Π±ΡƒΠ΄Π΅ΠΌ ΡƒΡ‡ΠΈΡ‚Ρ‹Π²Π°Ρ‚ΡŒ, Ρ‡Ρ‚ΠΎ кинСтичСский коэффициСнт трСния зависит ΠΎΡ‚ скорости.

ΠŸΠΎΡΠΊΠΎΠ»ΡŒΠΊΡƒ коэффициСнт трСния зависит ΠΎΡ‚ ΠΎΡ‚Π½ΠΎΡΠΈΡ‚Π΅Π»ΡŒΠ½ΠΎΠΉ скорости ΡΠΊΠΎΠ»ΡŒΠ·ΡΡ‰ΠΈΡ… повСрхностСй, сила трСния Π±ΡƒΠ΄Π΅Ρ‚ ΠΈΠΌΠ΅Ρ‚ΡŒ Ρ‚Π°ΠΊΡƒΡŽ ​​ТС Π·Π°Π²ΠΈΡΠΈΠΌΠΎΡΡ‚ΡŒ. Максимальная сила трСния достигаСтся, ΠΊΠΎΠ³Π΄Π° Ρ‚Π΅Π»ΠΎ статично, ΠΊΠΎΠ³Π΄Π° ΠΎΠ½ΠΎ Π½Π°Ρ‡ΠΈΠ½Π°Π΅Ρ‚ Π΄Π²ΠΈΠ³Π°Ρ‚ΡŒΡΡ, сила трСния ΠΏΠ°Π΄Π°Π΅Ρ‚ вмСстС с коэффициСнтом трСния.

Π˜Π·ΠΎΠ±Ρ€Π°ΠΆΠ΅Π½ΠΈΠ΅: функция силы трСния ΠΎΡ‚ силы Ρ‚ΠΎΠ»Ρ‡ΠΊΠ°

Π³Π΄Π΅:
F fmax [N] — максимальная сила трСния
F fs [N] — сила трСния покоя
F fk [N] — кинСтичСская сила трСния
F p [Н] — Ρ‚ΠΎΠ»ΠΊΠ°ΡŽΡ‰Π°Ρ сила
x [ΠΌ] — смСщСниС Ρ‚Π΅Π»Π° (ΠΎΠ±ΡŠΠ΅ΠΊΡ‚Π°)

Из изобраТСния Π²Ρ‹ΡˆΠ΅ ΠΌΡ‹ ΠΌΠΎΠΆΠ΅ΠΌ ΡΠ΄Π΅Π»Π°Ρ‚ΡŒ ΡΠ»Π΅Π΄ΡƒΡŽΡ‰ΠΈΠ΅ Π²Ρ‹Π²ΠΎΠ΄Ρ‹:

  • Π² статичСской области сила трСния прямо ΠΏΡ€ΠΎΠΏΠΎΡ€Ρ†ΠΈΠΎΠ½Π°Π»ΡŒΠ½Π° Ρ‚ΠΎΠ»ΠΊΠ°ΡŽΡ‰Π°Ρ сила
  • максимальная сила трСния достигаСтся Π² статичСской области
  • , ΠΊΠΎΠ³Π΄Π° Ρ‚ΠΎΠ»ΠΊΠ°ΡŽΡ‰Π°Ρ сила ΠΏΡ€Π΅Π²Ρ‹ΡˆΠ°Π΅Ρ‚ ΠΌΠ°ΠΊΡΠΈΠΌΠ°Π»ΡŒΠ½ΡƒΡŽ силу трСния покоя, Ρ‚Π΅Π»ΠΎ Π½Π°Ρ‡ΠΈΠ½Π°Π΅Ρ‚ Π΄Π²ΠΈΠ³Π°Ρ‚ΡŒΡΡ ΠΈ сила трСния ΠΏΠ°Π΄Π°Π΅Ρ‚

Максимальная (статичСская) сила трСния рассчитываСтся ΠΊΠ°ΠΊ:

\ [F_ {fmax} = \ mu_ {s} \ cdot N \ tag {9} \]

Максимальная кинСтичСская сила трСния рассчитываСтся ΠΊΠ°ΠΊ:

\ [F_ {fk} = \ mu_ { k} \ cdot N \ tag {10} \]

ΠšΠΎΡΡ„Ρ„ΠΈΡ†ΠΈΠ΅Π½Ρ‚ трСния Ρ€Π°Π²Π΅Π½ de ΠΏΡ€Π΅ΠΊΡ€Π°Ρ‰Π΅Π½ΠΎ ΡΠΊΡΠΏΠ΅Ρ€ΠΈΠΌΠ΅Π½Ρ‚Π°Π»ΡŒΠ½ΠΎ, Π½ΠΎ Π΅ΡΡ‚ΡŒ Π½Π΅ΠΊΠΎΡ‚ΠΎΡ€Ρ‹Π΅ значСния, доступныС Π² ΠΊΠ½ΠΈΠ³Π°Ρ… ΠΏΠΎ Ρ„ΠΈΠ·ΠΈΠΊΠ΅.Π’ Ρ‚Π°Π±Π»ΠΈΡ†Π΅ Π½ΠΈΠΆΠ΅ ΠΏΡ€ΠΈΠ²Π΅Π΄Π΅Π½Ρ‹ значСния коэффициСнта трСния, взятыС ΠΈΠ· справочных ΠΌΠ°Ρ‚Π΅Ρ€ΠΈΠ°Π»ΠΎΠ² [1], [2] ΠΈ [3].

0,5 9048 с Π±Ρ€ΠΎΠ½Π·ΠΎΠ²Ρ‹ΠΌ ΠΏΠΎΠΊΡ€Ρ‹Ρ‚ΠΈΠ΅ΠΌ
ΠšΠΎΡΡ„Ρ„ΠΈΡ†ΠΈΠ΅Π½Ρ‚Ρ‹ трСния
ΠœΠ°Ρ‚Π΅Ρ€ΠΈΠ°Π» ΞΌ s [-] ΞΌ k [-]
Π‘Ρ‚Π°Π»ΡŒ Π½Π° стали
Π‘Ρ‚Π°Π»ΡŒ Π½Π° Π»ΡŒΠ΄Ρƒ 0,027 0,014
ΠœΠ΅Ρ‚Π°Π»Π» Π½Π° Π»ΡŒΠ΄Ρƒ 0.03 — 0,05
Алюминий Π½Π° стали 0,61 0,47
МСдь Π½Π° стали 0,53 0,36
МСдь Π½Π° ΠΌΠ΅Π΄ΠΈ 9048 Π Π΅Π·ΠΈΠ½Π° ΠΏΠΎ Π±Π΅Ρ‚ΠΎΠ½Ρƒ 1,00 0,80
Π”Π΅Ρ€Π΅Π²ΠΎ ΠΏΠΎ Π΄Π΅Ρ€Π΅Π²Ρƒ 0,25 — 0,50 0,2
Π”ΡƒΠ± Π½Π° Π΄ΡƒΠ±Π΅ (сухой, вдоль Π²ΠΎΠ»ΠΎΠΊΠΎΠ½) 0.62 0,48
Π”ΡƒΠ± Π½Π° Π΄ΡƒΠ±Π΅ (сухой, пСрпСндикулярно Π²ΠΎΠ»ΠΎΠΊΠ½Π°ΠΌ) 0,54 0,34
Π‘Ρ‚Π΅ΠΊΠ»ΠΎ Π½Π° стСклС 0,94 0,4
ΠΌΠΎΠΊΡ€ΠΎΠ΅ Π΄Π΅Ρ€Π΅Π²ΠΎ Π½Π° снСгу 0,14 0,1
ВощСная дрСвСсина Π½Π° сухом снСгу 0,04
ΠœΠ΅Ρ‚Π°Π»Π» ΠΏΠΎ ΠΌΠ΅Ρ‚Π°Π»Π»Ρƒ (со смазкой) 0,15 0,06
Π›Π΅Π΄ Π½Π° Π»ΡŒΠ΄Ρƒ 0.1 0,03
Π’Π΅Ρ„Π»ΠΎΠ½ Π½Π° Ρ‚Π΅Ρ„Π»ΠΎΠ½Π΅ 0,04 0,04
Π‘ΠΈΠ½ΠΎΠ²ΠΈΠ΄Π½Ρ‹Π΅ суставы Π½Π° Π»ΡŽΠ΄ΡΡ… 0,01 0,003
9048
Π§ΡƒΠ³ΡƒΠ½ Π½Π° Ρ‡ΡƒΠ³ΡƒΠ½Π΅ (слСгка смазанный) 0,15
Π§ΡƒΠ³ΡƒΠ½ Π½Π° Π±Ρ€ΠΎΠ½Π·Π΅ (слСгка смазанный) 0,15
ΠšΠΎΠΆΠ°Π½Ρ‹ΠΉ Ρ€Π΅ΠΌΠ΅Π½ΡŒ Π½Π° Ρ‡ΡƒΠ³ΡƒΠ½Π΅ (слСгка смазанный) 0.28
КоТа ΠΏΠΎ Π΄Π΅Ρ€Π΅Π²Ρƒ 0,20 — 0,50
КоТа ΠΏΠΎ ΠΌΠ΅Ρ‚Π°Π»Π»Ρƒ 0,03 — 0,60

Для Π»ΡƒΡ‡ΡˆΠ΅Π³ΠΎ понимания силы трСния , Π΄Π°Π²Π°ΠΉΡ‚Π΅ рассмотрим нСсколько практичСских ΠΏΡ€ΠΈΠΌΠ΅Ρ€ΠΎΠ².

ΠŸΡ€ΠΈΠΌΠ΅Ρ€ 1. Π§Π΅Π»ΠΎΠ²Π΅ΠΊ Ρ‚ΠΎΠ»ΠΊΠ°Π΅Ρ‚ Π°Π»ΡŽΠΌΠΈΠ½ΠΈΠ΅Π²Ρ‹ΠΉ ящик вСсом 50 ΠΊΠ³ ΠΏΠΎ ΡΡ‚Π°Π»ΡŒΠ½ΠΎΠΌΡƒ ΠΏΠΎΠ»Ρƒ. ΠŸΠΎΠ΄ΡΡ‡ΠΈΡ‚Π°ΠΉΡ‚Π΅, с ΠΊΠ°ΠΊΠΎΠΉ силой Π½ΡƒΠΆΠ½ΠΎ Ρ‚ΠΎΠ»ΠΊΠ½ΡƒΡ‚ΡŒ Ρ‡Π΅Π»ΠΎΠ²Π΅ΠΊΠ°, Ρ‡Ρ‚ΠΎΠ±Ρ‹ ящик Π½Π°Ρ‡Π°Π» Π΄Π²ΠΈΠ³Π°Ρ‚ΡŒΡΡ.КакоС Ρ‚ΠΎΠ»ΠΊΠ°ΡŽΡ‰Π΅Π΅ усилиС трСбуСтся Π² Π΄Π²ΠΈΠΆΠ΅Π½ΠΈΠΈ, Ρ‡Ρ‚ΠΎΠ±Ρ‹ ящик ΠΏΡ€ΠΎΠ΄ΠΎΠ»ΠΆΠ°Π» ΡΠΊΠΎΠ»ΡŒΠ·ΠΈΡ‚ΡŒ? Каково ΡƒΠΌΠ΅Π½ΡŒΡˆΠ΅Π½ΠΈΠ΅ силы Ρ‚ΠΎΠ»Ρ‡ΠΊΠ°, ΠΊΠΎΠ³Π΄Π° ящик Π½Π°Ρ‡ΠΈΠ½Π°Π΅Ρ‚ Π΄Π²ΠΈΠ³Π°Ρ‚ΡŒΡΡ?

Π˜Π·ΠΎΠ±Ρ€Π°ΠΆΠ΅Π½ΠΈΠ΅: Π’ΠΎΠ»ΠΊΠ°ΡŽΡ‰ΠΈΠΉ ящик — сила трСния

Π˜ΡΡ…ΠΎΠ΄Ρ ΠΈΠ· опрСдСлСния ΠΏΡ€ΠΎΠ±Π»Π΅ΠΌΡ‹, ΠΌΡ‹ ΠΌΠΎΠΆΠ΅ΠΌ ΠΈΠ·Π²Π»Π΅Ρ‡ΡŒ Π²Ρ…ΠΎΠ΄Π½Ρ‹Π΅ Π΄Π°Π½Π½Ρ‹Π΅ ΠΊΠ°ΠΊ:

\ [\ begin {split}
m & = 50 \ text {kg} \\
\ mu_ {s} & = 0,61 \\
\ mu_ {k} & = 0,47 \\
\ end {split} \]

Π¨Π°Π³ 1 . РассчитайтС Π½ΠΎΡ€ΠΌΠ°Π»ΡŒΠ½ΡƒΡŽ силу Ρ€Π΅Π°ΠΊΡ†ΠΈΠΈ Π½Π° ΠΎΠ±Ρ€Π΅ΡˆΠ΅Ρ‚ΠΊΡƒ.

\ [\ begin {split}
\ sum F_ {y} & = 0 \\
N — G & = 0 \\
N & = G
\ end {split} \]

Π¨Π°Π³ 2 .РассчитайтС силу трСния покоя, ΠΈΡΠΏΠΎΠ»ΡŒΠ·ΡƒΡ ΡƒΡ€Π°Π²Π½Π΅Π½ΠΈΠ΅ (5).

\ [F_ {fs} = \ mu_ {s} \ cdot N = \ mu_ {s} \ cdot G = \ mu_ {s} \ cdot m \ cdot g = 0,61 \ cdot 50 \ cdot 9,81 = 298,9 \ text { N} \]

Π¨Π°Π³ 3 . РассчитайтС Ρ‚ΠΎΠ»ΠΊΠ°ΡŽΡ‰ΡƒΡŽ силу ΠΈΠ· условия равновСсия.

\ [\ begin {split}
\ sum F_ {x} & = 0 \\
F_ {p} — F_ {f} & = 0 \\
F_ {p} & = F_ {f}
\ end { split} \]

Π§Ρ‚ΠΎΠ±Ρ‹ ящик Π½Π°Ρ‡Π°Π» Π΄Π²ΠΈΠ³Π°Ρ‚ΡŒΡΡ, сила Ρ‚ΠΎΠ»Ρ‡ΠΊΠ° Π΄ΠΎΠ»ΠΆΠ½Π° Π±Ρ‹Ρ‚ΡŒ Π²Ρ‹ΡˆΠ΅ 298,9 Н.

Step 4 .РассчитайтС ΠΊΠΈΠ½Π΅Ρ‚ΠΈΡ‡Π΅ΡΠΊΡƒΡŽ силу трСния.

\ [F_ {fk} = \ mu_ {k} \ cdot N = \ mu_ {k} \ cdot G = \ mu_ {k} \ cdot m \ cdot g = 0,47 \ cdot 50 \ cdot 9,81 = 230,535 \ text { N} \]

Π§Ρ‚ΠΎΠ±Ρ‹ ящик ΠΏΡ€ΠΎΠ΄ΠΎΠ»ΠΆΠ°Π» Π΄Π²ΠΈΠ³Π°Ρ‚ΡŒΡΡ, сила Ρ‚ΠΎΠ»Ρ‡ΠΊΠ° Π΄ΠΎΠ»ΠΆΠ½Π° Π±Ρ‹Ρ‚ΡŒ Π²Ρ‹ΡˆΠ΅ 230,535 Н.

Π¨Π°Π³ 5 . Π‘Π½ΠΈΠΆΠ΅Π½ΠΈΠ΅ силы Ρ‚ΠΎΠ»Ρ‡ΠΊΠ° рассчитываСтся ΠΊΠ°ΠΊ:

\ [\ Delta F_ {f} \ text {[%]} = \ frac {\ left | F_ {fk} — F_ {fs} \ right |} {F_ {fs}} \ cdot 100 = 22.87 \ text {%} \]

Как Π²ΠΈΠ΄ΠΈΡ‚Π΅, сила Ρ‚ΠΎΠ»Ρ‡ΠΊΠ° Π΄ΠΎΠ»ΠΆΠ½Π° Π±Ρ‹Ρ‚ΡŒ Π²Ρ‹ΡˆΠ΅, Ρ‡Ρ‚ΠΎΠ±Ρ‹ Π½Π°Ρ‡Π°Ρ‚ΡŒ ΠΏΠ΅Ρ€Π΅ΠΌΠ΅Ρ‰Π΅Π½ΠΈΠ΅ ящика ΠΈ Π½Π΅ΠΌΠ½ΠΎΠ³ΠΎ Π½ΠΈΠΆΠ΅, Ρ‡Ρ‚ΠΎΠ±Ρ‹ ящик ΠΏΡ€ΠΎΠ΄ΠΎΠ»ΠΆΠ°Π» Π΄Π²ΠΈΠ³Π°Ρ‚ΡŒΡΡ.

ΠŸΡ€ΠΈΠΌΠ΅Ρ€ 2. Π§Π΅Π»ΠΎΠ²Π΅ΠΊ тянСт Π°Π»ΡŽΠΌΠΈΠ½ΠΈΠ΅Π²Ρ‹ΠΉ ящик вСсом 50 ΠΊΠ³ ΠΏΠΎ ΡΡ‚Π°Π»ΡŒΠ½ΠΎΠΌΡƒ ΠΏΠΎΠ»Ρƒ, ΠΈΡΠΏΠΎΠ»ΡŒΠ·ΡƒΡ Π²Π΅Ρ€Π΅Π²ΠΊΡƒ ΠΏΠΎΠ΄ ΡƒΠ³Π»ΠΎΠΌ 30 Β° ΠΊ Π³ΠΎΡ€ΠΈΠ·ΠΎΠ½Ρ‚Π°Π»ΠΈ. ΠŸΠΎΠ΄ΡΡ‡ΠΈΡ‚Π°ΠΉΡ‚Π΅, с ΠΊΠ°ΠΊΠΎΠΉ силой Π½ΡƒΠΆΠ½ΠΎ Ρ‚ΡΠ½ΡƒΡ‚ΡŒ Ρ‡Π΅Π»ΠΎΠ²Π΅ΠΊΠ°, Ρ‡Ρ‚ΠΎΠ±Ρ‹ ящик Π½Π°Ρ‡Π°Π» Π΄Π²ΠΈΠ³Π°Ρ‚ΡŒΡΡ. КакоС тяговоС усилиС трСбуСтся, Ρ‡Ρ‚ΠΎΠ±Ρ‹ ящик ΠΏΡ€ΠΎΠ΄ΠΎΠ»ΠΆΠ°Π» Π΄Π²ΠΈΠ³Π°Ρ‚ΡŒΡΡ Π² Π΄Π²ΠΈΠΆΠ΅Π½ΠΈΠΈ? Каково ΡƒΠΌΠ΅Π½ΡŒΡˆΠ΅Π½ΠΈΠ΅ тягового усилия, ΠΊΠΎΠ³Π΄Π° ящик Π½Π°Ρ‡ΠΈΠ½Π°Π΅Ρ‚ Π΄Π²ΠΈΠ³Π°Ρ‚ΡŒΡΡ? По ΡΡ€Π°Π²Π½Π΅Π½ΠΈΡŽ с ΠΏΡ€Π΅Π΄Ρ‹Π΄ΡƒΡ‰ΠΈΠΌ ΠΏΡ€ΠΈΠΌΠ΅Ρ€ΠΎΠΌ Π»Π΅Π³Ρ‡Π΅ Π»ΠΈ Ρ‚ΡΠ½ΡƒΡ‚ΡŒ ΠΈΠ»ΠΈ Ρ‚ΠΎΠ»ΠΊΠ°Ρ‚ΡŒ корпус (ΠΎΠ±ΡŠΠ΅ΠΊΡ‚) сайдинга?

Π˜Π·ΠΎΠ±Ρ€Π°ΠΆΠ΅Π½ΠΈΠ΅: ВытягиваниС ящика — сила трСния

Π˜ΡΡ…ΠΎΠ΄Π½Ρ‹Π΅ Π΄Π°Π½Π½Ρ‹Π΅ ΠΈΠ· опрСдСлСния ΠΏΡ€ΠΎΠ±Π»Π΅ΠΌΡ‹ ΠΌΡ‹ ΠΌΠΎΠΆΠ΅ΠΌ ΠΈΠ·Π²Π»Π΅Ρ‡ΡŒ ΠΊΠ°ΠΊ:

\ [\ begin {split}
m & = 50 \ text {kg} \\
\ mu_ {s} & = 0.{\ circ}
\ end {split} \]

Π¨Π°Π³ 1 . РассчитайтС Π½ΠΎΡ€ΠΌΠ°Π»ΡŒΠ½ΡƒΡŽ силу Ρ€Π΅Π°ΠΊΡ†ΠΈΠΈ Π½Π° ΠΎΠ±Ρ€Π΅ΡˆΠ΅Ρ‚ΠΊΡƒ.

\ [\ begin {split}
\ sum F_ {y} & = 0 \\
N + F_ {py} — G & = 0 \\
N + F_ {p} \ cdot \ sin (\ alpha) — G & = 0 \\
N & = G — F_ {p} \ cdot \ sin (\ alpha)
\ end {split} \]

Π¨Π°Π³ 2 . РассчитайтС силу трСния покоя, ΠΈΡΠΏΠΎΠ»ΡŒΠ·ΡƒΡ ΡƒΡ€Π°Π²Π½Π΅Π½ΠΈΠ΅ (5).

\ [F_ {fs} = \ mu_ {s} \ cdot N = \ mu_ {s} \ left (G — F_ {p} \ cdot \ sin (\ alpha) \ right) \]

Π¨Π°Π³ 3 .РассчитайтС силу трСния покоя ΠΈΠ· равновСсия Π³ΠΎΡ€ΠΈΠ·ΠΎΠ½Ρ‚Π°Π»ΡŒΠ½Ρ‹Ρ… сил.

\ [\ begin {split}
\ sum F_ {x} & = 0 \\
F_ {px} — F_ {fs} & = 0 \\
F_ {p} \ cdot \ cos (\ alpha) & = F_ {fs}
\ end {split} \]

Π¨Π°Π³ 4 . На ΡˆΠ°Π³Π°Ρ… 2 ΠΈ 3 ΠΌΡ‹ вычисляСм Π·Π½Π°Ρ‡Π΅Π½ΠΈΠ΅ (статичСской) тягового усилия для Π½Π°Ρ‡Π°Π»Π° пСрСмСщСния ящика.

\ [\ begin {split}
F_ {ps} \ cdot \ cos (\ alpha) & = \ mu_ {s} \ cdot \ left (G — F_ {p} \ sin (\ alpha) \ right) \\
F_ {ps} & = \ frac {\ mu_ {s} \ cdot m \ cdot g} {\ cos (\ alpha) + \ mu_ {s} \ cdot \ sin (\ alpha)} \\
F_ {ps } & = 255.51 \ text {N}
\ end {split} \]

Π§Ρ‚ΠΎΠ±Ρ‹ ящик Π½Π°Ρ‡Π°Π» Π΄Π²ΠΈΠ³Π°Ρ‚ΡŒΡΡ, тяговоС усилиС Π΄ΠΎΠ»ΠΆΠ½ΠΎ Π±Ρ‹Ρ‚ΡŒ Π²Ρ‹ΡˆΠ΅ 255,51 Н.

ΠœΡ‹ Ρ‚Π°ΠΊΠΆΠ΅ ΠΌΠΎΠΆΠ΅ΠΌ Ρ€Π°ΡΡΡ‡ΠΈΡ‚Π°Ρ‚ΡŒ силу статичСского трСния ΠΊΠ°ΠΊ:

\ [F_ { fs} = F_ {ps} \ cdot \ cos (\ alpha) = 221.28 \ text {N} \]

Π¨Π°Π³ 5 . РассчитайтС ΠΊΠΈΠ½Π΅Ρ‚ΠΈΡ‡Π΅ΡΠΊΡƒΡŽ Ρ‚ΡΠ³ΠΎΠ²ΡƒΡŽ силу.

\ [F_ {pk} = \ frac {\ mu_ {k} \ cdot m \ cdot g} {\ cos (\ alpha) + \ mu_ {k} \ cdot \ sin (\ alpha)} = 209,38 \ text { N} \]

Π§Ρ‚ΠΎΠ±Ρ‹ ящик ΠΏΡ€ΠΎΠ΄ΠΎΠ»ΠΆΠ°Π» Π΄Π²ΠΈΠ³Π°Ρ‚ΡŒΡΡ, сила Ρ‚ΠΎΠ»Ρ‡ΠΊΠ° Π΄ΠΎΠ»ΠΆΠ½Π° Π±Ρ‹Ρ‚ΡŒ Π²Ρ‹ΡˆΠ΅ 209.38 Н.

ΠœΡ‹ Ρ‚Π°ΠΊΠΆΠ΅ ΠΌΠΎΠΆΠ΅ΠΌ Ρ€Π°ΡΡΡ‡ΠΈΡ‚Π°Ρ‚ΡŒ ΠΊΠΈΠ½Π΅Ρ‚ΠΈΡ‡Π΅ΡΠΊΡƒΡŽ силу трСния ΠΊΠ°ΠΊ:

\ [F_ {fk} = F_ {pk} \ cdot \ cos (\ alpha) = 181.33 \ text {N} \]

Step 6 . Π‘Π½ΠΈΠΆΠ΅Π½ΠΈΠ΅ тягового усилия рассчитываСтся ΠΊΠ°ΠΊ:

\ [\ Delta F_ {p} \ text {[%]} = \ frac {\ left | F_ {pk} — F_ {ps} \ right |} {F_ {ps}} \ cdot 100 = 18.05 \ text {%} \]

Как Π²ΠΈΠ΄ΠΈΡ‚Π΅, тяговоС усилиС Π΄ΠΎΠ»ΠΆΠ½ΠΎ Π±Ρ‹Ρ‚ΡŒ Π²Ρ‹ΡˆΠ΅, Ρ‡Ρ‚ΠΎΠ±Ρ‹ ящик Π½Π°Ρ‡Π°Π» ΠΏΠ΅Ρ€Π΅ΠΌΠ΅Ρ‰Π°Ρ‚ΡŒΡΡ ΠΈ Π½Π΅ΠΌΠ½ΠΎΠ³ΠΎ Π½ΠΈΠΆΠ΅, Ρ‡Ρ‚ΠΎΠ±Ρ‹ ящик двигался.

Π¨Π°Π³ 7 . По ΡΡ€Π°Π²Π½Π΅Π½ΠΈΡŽ с силой Ρ‚ΠΎΠ»Ρ‡ΠΊΠ°, сила тяги Π΄ΠΎΠ»ΠΆΠ½Π° Π±Ρ‹Ρ‚ΡŒ мСньшС, Ρ‡Ρ‚ΠΎΠ±Ρ‹ ящик сдвинулся с мСста.Π Π°Π·Π½ΠΈΡ†Π° Π² Ρ‚ΠΎΠ»ΠΊΠ°ΡŽΡ‰Π΅ΠΉ ΠΈ тянущСй силах для статичСской ΠΈ кинСматичСской областСй составляСт:

\ [\ begin {split}
\ Delta F_ {ps} \ text {[%]} & = \ frac {\ left | 255,507 — 298,9 \ right |} {298,9} \ cdot 100 = 14,52 \ text {%} \\
\ Delta F_ {pk} \ text {[%]} & = \ frac {\ left | 209.38 — 230.535 \ right |} {230.535} \ cdot 100 = 9.18 \ text {%}
\ end {split} \]

Π­Ρ‚ΠΎ связано с эффСктом ΡƒΠ³Π»Π° Π½Π°ΠΊΠ»ΠΎΠ½Π° Π²Π΅Ρ€Π΅Π²ΠΊΠΈ, ΠΊΠΎΡ‚ΠΎΡ€Ρ‹ΠΉ сниТаСт Π½ΠΎΡ€ΠΌΠ°Π»ΡŒΠ½ΡƒΡŽ Ρ€Π΅Π°ΠΊΡ†ΠΈΡŽ Π½Π° ящик , ΠΎΡ‚ΡΡŽΠ΄Π° ΠΈ сила трСния. Однако это ΡƒΠΌΠ΅Π½ΡŒΡˆΠ΅Π½ΠΈΠ΅ Π½ΠΎΡ€ΠΌΠ°Π»ΡŒΠ½ΠΎΠΉ силы, Π΄Π΅ΠΉΡΡ‚Π²ΡƒΡŽΡ‰Π΅ΠΉ Π½Π° ящик, прСобразуСтся Π² Π²Π΅Ρ€Ρ‚ΠΈΠΊΠ°Π»ΡŒΠ½ΡƒΡŽ силу, Π²ΠΎΠ·Π΄Π΅ΠΉΡΡ‚Π²ΡƒΡŽΡ‰ΡƒΡŽ Π½Π° ΠΏΠ»Π΅Ρ‡ΠΎ Ρ‡Π΅Π»ΠΎΠ²Π΅ΠΊΠ°, которая ΠΎΠΊΠ°Π·Ρ‹Π²Π°Π΅Ρ‚ Π΄Π°Π²Π»Π΅Π½ΠΈΠ΅.Π­Ρ‚Π° Π²Π΅Ρ€Ρ‚ΠΈΠΊΠ°Π»ΡŒΠ½Π°Ρ сила Ρ€Π°Π²Π½Π°:

\ [\ begin {split}
\ text {static:} F_ {ps} \ cdot \ sin (\ alpha) & = 127.75 \ text {N} \\
\ text {kinetic: } F_ {pk} \ cdot \ sin (\ alpha) & = 104.69 \ text {N}
\ end {split} \]

ΠŸΡ€ΠΈΠΌΠ΅Ρ€ 3. Для Ρ‚Π΅Π» Π½Π° рисункС Π½ΠΈΠΆΠ΅ Π½Π°ΠΉΠ΄ΠΈΡ‚Π΅ Π·Π½Π°Ρ‡Π΅Π½ΠΈΠ΅ массы Ρ‚Π΅Π»Π° m 2 , для ΠΊΠΎΡ‚ΠΎΡ€ΠΎΠ³ΠΎ масса Ρ‚Π΅Π»Π° m 1 = 30 ΠΊΠ³ Π½Π°Ρ‡ΠΈΠ½Π°Π΅Ρ‚ Π΄Π²ΠΈΠΆΠ΅Π½ΠΈΠ΅ Π²Π²Π΅Ρ€Ρ…. ΠšΠΎΡ€ΠΏΡƒΡ 1 ΠΈ Π°ΠΏΠΏΠ°Ρ€Π΅Π»ΡŒ Π²Ρ‹ΠΏΠΎΠ»Π½Π΅Π½Ρ‹ ΠΈΠ· ΠΌΠ΅Ρ‚Π°Π»Π»Π° со смазкой повСрхностСй скольТСния. Π£Π³ΠΎΠ» ската 45 Β°.{\ circ}
\ end {split} \]

Π¨Π°Π³ 1 . РассчитайтС Π·Π½Π°Ρ‡Π΅Π½ΠΈΠ΅ тягового усилия F p1 [Н] .

Π“ΠΎΡ€ΠΈΠ·ΠΎΠ½Ρ‚Π°Π»ΡŒΠ½ΠΎΠ΅ равновСсиС:

\ [\ begin {split}
\ sum F_ {x} & = 0 \\
F_ {p1} — F_ {f1} — G_ {1} \ cdot \ sin (\ alpha) & = 0 \\
F_ {p1} & = F_ {f1} + G_ {1} \ cdot \ sin (\ alpha)
\ end {split} \]

Π’Π΅Ρ€Ρ‚ΠΈΠΊΠ°Π»ΡŒΠ½ΠΎΠ΅ равновСсиС:

\ [\ begin {split}
\ sum F_ {y} & = 0 \\
N — G_ {1} \ cdot \ cos (\ alpha) & = 0 \\
N & = G_ {1} \ cdot \ cos (\ alpha)
\ end {split } \]

Π‘ΠΈΠ»Π° статичСского трСния F f1 [Н] рассчитываСтся ΠΊΠ°ΠΊ:

\ [F_ {f1} = \ mu_ {s} \ cdot N = \ mu_ {s} \ cdot G_ {1} \ cdot \ cos (\ alpha) \]

Π’Π΅ΠΏΠ΅Ρ€ΡŒ ΠΌΡ‹ ΠΌΠΎΠΆΠ΅ΠΌ ΠΏΠΎΠ»ΡƒΡ‡ΠΈΡ‚ΡŒ Π²Ρ‹Ρ€Π°ΠΆΠ΅Π½ΠΈΠ΅ Ρ„ΡƒΠ½ΠΊΡ†ΠΈΠΈ тягового усилия извСстных ΠΏΠ°Ρ€Π°ΠΌΠ΅Ρ‚Ρ€ΠΎΠ²:

\ [\ begin {split}
F_ {p1} & = G_ {1} \ cdot \ left ( \ mu_ {s} \ cdot \ cos (\ alpha) + \ sin (\ alpha) \ right) \\
& = m_ {1} \ cdot g \ cdot \ left (\ mu_ {s} \ cdot \ cos ( \ alpha) + \ sin (\ alpha) \ right) \\
& = 185.38 \ text {N}
\ end {split} \]

Π¨Π°Π³ 2 . РассчитайтС Π·Π½Π°Ρ‡Π΅Π½ΠΈΠ΅ тягового усилия F p2 [Н] .

Π’Π΅Ρ€Ρ‚ΠΈΠΊΠ°Π»ΡŒΠ½ΠΎΠ΅ равновСсиС:

\ [\ begin {split}
\ sum F_ {y} & = 0 \\
F_ {p2} — G_ {2} & = 0 \\
F_ {p2} & = G_ {2 } \\
F_ {p2} & = m_ {2} \ cdot g
\ end {split} \]

Π¨Π°Π³ 3 . Π’Ρ‹Ρ‡ΠΈΡΠ»ΠΈΡ‚ΡŒ ΠΌ 2 [ΠΊΠ³] .

НатяТСниС троса (ΠΏΡ€ΠΎΠ²ΠΎΠ»ΠΎΠΊΠΈ), ΡΠΎΠ΅Π΄ΠΈΠ½ΡΡŽΡ‰Π΅Π³ΠΎ Π΄Π²Π° корпуса, ΠΎΠ΄ΠΈΠ½Π°ΠΊΠΎΠ²ΠΎ с ΠΎΠ±Π΅ΠΈΡ… сторон.Из этого условия ΠΌΡ‹ ΠΌΠΎΠΆΠ΅ΠΌ Π²Ρ‹Ρ‡ΠΈΡΠ»ΠΈΡ‚ΡŒ m 2 ΠΊΠ°ΠΊ:

\ [\ begin {split}
F_ {p2} & = F_ {p1} \\
m_ {2} \ cdot 9.81 & = 185.38 \ text {} \\
m_ {2} & = 18.9 \ text {kg}
\ end {split} \]

Если масса Ρ‚Π΅Π»Π° m 2 тяТСлСС 18,9 ΠΊΠ³, масса Ρ‚Π΅Π»Π° m 1 Π½Π°Ρ‡Π½Π΅Ρ‚ скольТСниС Π²Π²Π΅Ρ€Ρ… Π½Π° Ρ€Π°ΠΌΠΏΡƒ.

ΠšΠ°Π»ΡŒΠΊΡƒΠ»ΡΡ‚ΠΎΡ€ силы трСния

ΠšΠ°Π»ΡŒΠΊΡƒΠ»ΡΡ‚ΠΎΡ€ Π½ΠΈΠΆΠ΅ Π΄Π°Π΅Ρ‚ значСния тяговых сил (статичСских ΠΈ кинСматичСских) ΠΈ сил трСния (статичСских ΠΈ кинСматичСских) для Ρ‚Π²Π΅Ρ€Π΄ΠΎΠ³ΠΎ Ρ‚Π΅Π»Π° массой ΠΌ [ΠΊΠ³] , натянутого Π½Π° ΠΏΠΎΠ²Π΅Ρ€Ρ…Π½ΠΎΡΡ‚ΡŒ с ΠΌΠΊΠΌ. s [-] ΠΈ ΞΌ k [-] , с ΡƒΠ³Π»ΠΎΠΌ тягового усилия Ξ± [Β°] .

Π˜Π·ΠΎΠ±Ρ€Π°ΠΆΠ΅Π½ΠΈΠ΅: Π΄ΠΈΠ°Π³Ρ€Π°ΠΌΠΌΠ° силы трСния для ΠΊΠ°Π»ΡŒΠΊΡƒΠ»ΡΡ‚ΠΎΡ€Π°

УравнСния, ΠΈΡΠΏΠΎΠ»ΡŒΠ·ΡƒΠ΅ΠΌΡ‹Π΅ для расчСта сил, основаны Π½Π° выраТСниях ΠΈΠ· ΠŸΡ€ΠΈΠΌΠ΅Ρ€ 2 . Π§Ρ‚ΠΎΠ±Ρ‹ уравнСния ΠΈΠΌΠ΅Π»ΠΈ смысл, Π·Π½Π°Ρ‡Π΅Π½ΠΈΠ΅ массы ΠΎΠ³Ρ€Π°Π½ΠΈΡ‡Π΅Π½ΠΎ Ρ‚ΠΎΠ»ΡŒΠΊΠΎ ΠΏΠΎΠ»ΠΎΠΆΠΈΡ‚Π΅Π»ΡŒΠ½Ρ‹ΠΌΠΈ значСниями, коэффициСнт трСния ΠΌΠ΅ΠΆΠ΄Ρƒ [0, 2] ΠΈ ΡƒΠ³ΠΎΠ» тягового усилия ΠΌΠ΅ΠΆΠ΄Ρƒ [0 Β°, 90 Β°].

Π Π΅Π·ΡƒΠ»ΡŒΡ‚Π°Ρ‚Ρ‹ довольно интСрСсныС. Они ΠΏΠΎΠΊΠ°Π·Ρ‹Π²Π°ΡŽΡ‚, Ρ‡Ρ‚ΠΎ ΠΎΠΏΡ‚ΠΈΠΌΠ°Π»ΡŒΠ½Ρ‹ΠΉ ΡƒΠ³ΠΎΠ» для достиТСния минимального тягового усилия составляСт 31 Β° (статичСский) ΠΈ 25 Β° (кинСтичСский).ΠšΡ€ΠΎΠΌΠ΅ Ρ‚ΠΎΠ³ΠΎ, ΠΊΠ°ΠΊ ΠΈ оТидалось, силы трСния ΠΌΠ°ΠΊΡΠΈΠΌΠ°Π»ΡŒΠ½Ρ‹ ΠΏΡ€ΠΈ ΡƒΠ³Π»Π΅ 0 Β° ΠΈ Π½ΡƒΠ»Π΅Π²Ρ‹Π΅ ΠΏΡ€ΠΈ ΡƒΠ³Π»Π΅ 90 Β°.

ΠšΠ°Ρ‚Π°Π»ΠΎΠΆΠ½Ρ‹ΠΉ Π½ΠΎΠΌΠ΅Ρ€:

[1] R.C. Hibbeler, Engineering Mechanics — Statics, 14th Edition, Pearson, 2017.
[2] M. Radoi, E. Deciu, Mecanica, Editia a II-a revizuita, Editura Didactica si Pedagogica, Bucuresti, 1981.
[3] Physics: Для ΡƒΡ‡Π΅Π½Ρ‹Ρ… ΠΈ ΠΈΠ½ΠΆΠ΅Π½Π΅Ρ€ΠΎΠ² 6-Π΅ Π˜Π—Π”ΠΠΠ˜Π• Π Π°ΠΉΠΌΠΎΠ½Π΄Π° А. БСруэя ΠΈ Π”ΠΆΠΎΠ½Π° Π’. Π”ΠΆΡƒΠ΅Ρ‚Ρ‚Π°. Брукс / ΠšΠΎΡƒΠ» Паблишинг Ко., 2004.

ΠžΠΏΡ€Π΅Π΄Π΅Π»Π΅Π½ΠΈΠ΅ ΠΈΠ½Π΄ΠΈΠ²ΠΈΠ΄ΡƒΠ°Π»ΡŒΠ½Ρ‹Ρ… Π·Π½Π°Ρ‡Π΅Π½ΠΈΠΉ силы

Как Π±Ρ‹Π»ΠΎ сказано Ρ€Π°Π½Π΅Π΅ Π² Π£Ρ€ΠΎΠΊΠ΅ 3 (Π° Ρ‚Π°ΠΊΠΆΠ΅ Π² Π£Ρ€ΠΎΠΊΠ΅ 2), Ρ€Π΅Π·ΡƒΠ»ΡŒΡ‚ΠΈΡ€ΡƒΡŽΡ‰Π°Ρ сила — это вСкторная сумма всСх ΠΈΠ½Π΄ΠΈΠ²ΠΈΠ΄ΡƒΠ°Π»ΡŒΠ½Ρ‹Ρ… сил.Π’ Π£Ρ€ΠΎΠΊΠ΅ 2 ΠΌΡ‹ ΡƒΠ·Π½Π°Π»ΠΈ, ΠΊΠ°ΠΊ ΠΎΠΏΡ€Π΅Π΄Π΅Π»ΠΈΡ‚ΡŒ Ρ‡ΠΈΡΡ‚ΡƒΡŽ силу, Ссли извСстны Π²Π΅Π»ΠΈΡ‡ΠΈΠ½Ρ‹ всСх ΠΎΡ‚Π΄Π΅Π»ΡŒΠ½Ρ‹Ρ… сил. Π’ этом ΡƒΡ€ΠΎΠΊΠ΅ ΠΌΡ‹ ΡƒΠ·Π½Π°Π΅ΠΌ, ΠΊΠ°ΠΊ ΠΎΠΏΡ€Π΅Π΄Π΅Π»ΡΡ‚ΡŒ Π²Π΅Π»ΠΈΡ‡ΠΈΠ½Ρ‹ всСх ΠΎΡ‚Π΄Π΅Π»ΡŒΠ½Ρ‹Ρ… сил, Ссли извСстны масса ΠΈ ускорСниС ΠΎΠ±ΡŠΠ΅ΠΊΡ‚Π°. Π’Ρ€ΠΈ основных уравнСния, ΠΊΠΎΡ‚ΠΎΡ€Ρ‹Π΅ Π±ΡƒΠ΄ΡƒΡ‚ ΠΏΠΎΠ»Π΅Π·Π½Ρ‹: ΡƒΡ€Π°Π²Π½Π΅Π½ΠΈΠ΅ для чистой силы (F net = m β€’ a), ΡƒΡ€Π°Π²Π½Π΅Π½ΠΈΠ΅ для Π³Ρ€Π°Π²ΠΈΡ‚Π°Ρ†ΠΈΠΎΠ½Π½ΠΎΠΉ силы (F grav = m β€’ g) ΠΈ ΡƒΡ€Π°Π²Π½Π΅Π½ΠΈΠ΅ для силы трСния (F frict = ΞΌ β€’ F Π½ΠΎΡ€ΠΌΠ° ).

ΠŸΡ€ΠΎΡ†Π΅ΡΡ опрСдСлСния значСния ΠΎΡ‚Π΄Π΅Π»ΡŒΠ½Ρ‹Ρ… сил, Π΄Π΅ΠΉΡΡ‚Π²ΡƒΡŽΡ‰ΠΈΡ… Π½Π° ΠΎΠ±ΡŠΠ΅ΠΊΡ‚, Π²ΠΊΠ»ΡŽΡ‡Π°Π΅Ρ‚ ΠΏΡ€ΠΈΠΌΠ΅Π½Π΅Π½ΠΈΠ΅ Π²Ρ‚ΠΎΡ€ΠΎΠ³ΠΎ Π·Π°ΠΊΠΎΠ½Π° ΠΡŒΡŽΡ‚ΠΎΠ½Π° (F net = m β€’ a) ΠΈ ΠΏΡ€ΠΈΠΌΠ΅Π½Π΅Π½ΠΈΠ΅ значСния чистой силы. Если масса (ΠΌ) ΠΈ ускорСниС (Π°) извСстны, Ρ‚ΠΎ чистая сила (F net ) ΠΌΠΎΠΆΠ΅Ρ‚ Π±Ρ‹Ρ‚ΡŒ ΠΎΠΏΡ€Π΅Π΄Π΅Π»Π΅Π½Π° с использованиСм уравнСния.

F net = m β€’ a

Если числовоС Π·Π½Π°Ρ‡Π΅Π½ΠΈΠ΅ чистой силы ΠΈ Π½Π°ΠΏΡ€Π°Π²Π»Π΅Π½ΠΈΠ΅ чистой силы извСстны, Ρ‚ΠΎ ΠΌΠΎΠΆΠ½ΠΎ ΠΎΠΏΡ€Π΅Π΄Π΅Π»ΠΈΡ‚ΡŒ Π·Π½Π°Ρ‡Π΅Π½ΠΈΠ΅ всСх ΠΈΠ½Π΄ΠΈΠ²ΠΈΠ΄ΡƒΠ°Π»ΡŒΠ½Ρ‹Ρ… сил.Π’Π°ΠΊΠΈΠΌ ΠΎΠ±Ρ€Π°Π·ΠΎΠΌ, Π·Π°Π΄Π°Ρ‡Π° Π²ΠΊΠ»ΡŽΡ‡Π°Π΅Ρ‚ использованиС ΠΏΡ€ΠΈΠ²Π΅Π΄Π΅Π½Π½Ρ‹Ρ… Π²Ρ‹ΡˆΠ΅ ΡƒΡ€Π°Π²Π½Π΅Π½ΠΈΠΉ, Π΄Π°Π½Π½ΠΎΠΉ ΠΈΠ½Ρ„ΠΎΡ€ΠΌΠ°Ρ†ΠΈΠΈ ΠΈ вашСго понимания чистой силы для опрСдСлСния значСния ΠΎΡ‚Π΄Π΅Π»ΡŒΠ½Ρ‹Ρ… сил.

Π’Π°ΡˆΠ° ΠΎΡ‡Π΅Ρ€Π΅Π΄ΡŒ ΠΏΡ€Π°ΠΊΡ‚ΠΈΠΊΠΎΠ²Π°Ρ‚ΡŒΡΡ

Π§Ρ‚ΠΎΠ±Ρ‹ ΠΏΠΎΠ½ΡΡ‚ΡŒ, ΠΊΠ°ΠΊ примСняСтся этот ΠΌΠ΅Ρ‚ΠΎΠ΄, ΠΏΠΎΠΏΡ€ΠΎΠ±ΡƒΠΉΡ‚Π΅ Π²Ρ‹ΠΏΠΎΠ»Π½ΠΈΡ‚ΡŒ ΡΠ»Π΅Π΄ΡƒΡŽΡ‰ΠΈΠ΅ практичСскиС Π·Π°Π΄Π°Ρ‡ΠΈ. ΠŸΡ€ΠΎΠ±Π»Π΅ΠΌΡ‹ ΠΏΡ€ΠΎΠ³Ρ€Π΅ΡΡΠΈΡ€ΡƒΡŽΡ‚ ΠΎΡ‚ простых ΠΊ Π±ΠΎΠ»Π΅Π΅ слоТным. РСшив ΠΏΡ€ΠΎΠ±Π»Π΅ΠΌΡƒ, Π½Π°ΠΆΠΌΠΈΡ‚Π΅ ΠΊΠ½ΠΎΠΏΠΊΡƒ, Ρ‡Ρ‚ΠΎΠ±Ρ‹ ΠΏΡ€ΠΎΠ²Π΅Ρ€ΠΈΡ‚ΡŒ свои ΠΎΡ‚Π²Π΅Ρ‚Ρ‹.

ΠŸΡ€Π°ΠΊΡ‚ΠΈΠΊΠ° β„–1

Π”ΠΈΠ°Π³Ρ€Π°ΠΌΠΌΡ‹ свободного Ρ‚Π΅Π»Π° для Ρ‡Π΅Ρ‚Ρ‹Ρ€Π΅Ρ… ситуаций ΠΏΠΎΠΊΠ°Π·Π°Π½Ρ‹ Π½ΠΈΠΆΠ΅.Чистая сила извСстна для ΠΊΠ°ΠΆΠ΄ΠΎΠΉ ситуации. Однако Π²Π΅Π»ΠΈΡ‡ΠΈΠ½Π° Π½Π΅ΠΊΠΎΡ‚ΠΎΡ€Ρ‹Ρ… ΠΎΡ‚Π΄Π΅Π»ΡŒΠ½Ρ‹Ρ… сил нСизвСстна. ΠŸΡ€ΠΎΠ°Π½Π°Π»ΠΈΠ·ΠΈΡ€ΡƒΠΉΡ‚Π΅ ΠΊΠ°ΠΆΠ΄ΡƒΡŽ ΡΠΈΡ‚ΡƒΠ°Ρ†ΠΈΡŽ ΠΈΠ½Π΄ΠΈΠ²ΠΈΠ΄ΡƒΠ°Π»ΡŒΠ½ΠΎ ΠΈ ΠΎΠΏΡ€Π΅Π΄Π΅Π»ΠΈΡ‚Π΅ Π²Π΅Π»ΠΈΡ‡ΠΈΠ½Ρƒ нСизвСстных сил.

ΠŸΡ€Π°ΠΊΡ‚ΠΈΠΊΠ° β„–2

К ΠΎΠ±ΡŠΠ΅ΠΊΡ‚Ρƒ вСсом 6 ΠΊΠ³ ΠΏΡ€ΠΈΠΊΠ»Π°Π΄Ρ‹Π²Π°ΡŽΡ‚ Π½Π°ΠΏΡ€Π°Π²Π»Π΅Π½Π½ΡƒΡŽ Π²ΠΏΡ€Π°Π²ΠΎ силу, Ρ‡Ρ‚ΠΎΠ±Ρ‹ ΠΏΠ΅Ρ€Π΅ΠΌΠ΅Ρ‰Π°Ρ‚ΡŒ Π΅Π³ΠΎ ΠΏΠΎ ΡˆΠ΅Ρ€ΠΎΡ…ΠΎΠ²Π°Ρ‚ΠΎΠΉ повСрхности с постоянной ΡΠΊΠΎΡ€ΠΎΡΡ‚ΡŒΡŽ.На ΠΎΠ±ΡŠΠ΅ΠΊΡ‚ дСйствуСт сила трСния 15 Н. Π˜ΡΠΏΠΎΠ»ΡŒΠ·ΡƒΠΉΡ‚Π΅ Π΄ΠΈΠ°Π³Ρ€Π°ΠΌΠΌΡƒ, Ρ‡Ρ‚ΠΎΠ±Ρ‹ ΠΎΠΏΡ€Π΅Π΄Π΅Π»ΠΈΡ‚ΡŒ Π³Ρ€Π°Π²ΠΈΡ‚Π°Ρ†ΠΈΠΎΠ½Π½ΡƒΡŽ силу, Π½ΠΎΡ€ΠΌΠ°Π»ΡŒΠ½ΡƒΡŽ силу, Ρ‡ΠΈΡΡ‚ΡƒΡŽ силу ΠΈ ΠΏΡ€ΠΈΠ»ΠΎΠΆΠ΅Π½Π½ΡƒΡŽ силу. (ΠŸΡ€Π΅Π½Π΅Π±Ρ€Π΅Π³Π°Ρ сопротивлСниСм Π²ΠΎΠ·Π΄ΡƒΡ…Π°.)


ΠŸΡ€Π°ΠΊΡ‚ΠΈΠΊΠ° β„–3

К ΠΎΠ±ΡŠΠ΅ΠΊΡ‚Ρƒ массой 10 ΠΊΠ³ ΠΏΡ€ΠΈΠΊΠ»Π°Π΄Ρ‹Π²Π°ΡŽΡ‚ Π½Π°ΠΏΡ€Π°Π²Π»Π΅Π½Π½ΡƒΡŽ Π²ΠΏΡ€Π°Π²ΠΎ силу, Ρ‡Ρ‚ΠΎΠ±Ρ‹ ΠΏΠ΅Ρ€Π΅ΠΌΠ΅Ρ‰Π°Ρ‚ΡŒ Π΅Π³ΠΎ ΠΏΠΎ ΡˆΠ΅Ρ€ΠΎΡ…ΠΎΠ²Π°Ρ‚ΠΎΠΉ повСрхности с постоянной ΡΠΊΠΎΡ€ΠΎΡΡ‚ΡŒΡŽ.ΠšΠΎΡΡ„Ρ„ΠΈΡ†ΠΈΠ΅Π½Ρ‚ трСния ΠΌΠ΅ΠΆΠ΄Ρƒ ΠΏΡ€Π΅Π΄ΠΌΠ΅Ρ‚ΠΎΠΌ ΠΈ ΠΏΠΎΠ²Π΅Ρ€Ρ…Π½ΠΎΡΡ‚ΡŒΡŽ 0,2. Π˜ΡΠΏΠΎΠ»ΡŒΠ·ΡƒΠΉΡ‚Π΅ Π΄ΠΈΠ°Π³Ρ€Π°ΠΌΠΌΡƒ, Ρ‡Ρ‚ΠΎΠ±Ρ‹ ΠΎΠΏΡ€Π΅Π΄Π΅Π»ΠΈΡ‚ΡŒ Π³Ρ€Π°Π²ΠΈΡ‚Π°Ρ†ΠΈΠΎΠ½Π½ΡƒΡŽ силу, Π½ΠΎΡ€ΠΌΠ°Π»ΡŒΠ½ΡƒΡŽ силу, ΠΏΡ€ΠΈΠ»ΠΎΠΆΠ΅Π½Π½ΡƒΡŽ силу, силу трСния ΠΈ Ρ‡ΠΈΡΡ‚ΡƒΡŽ силу. (ΠŸΡ€Π΅Π½Π΅Π±Ρ€Π΅Π³Π°Ρ сопротивлСниСм Π²ΠΎΠ·Π΄ΡƒΡ…Π°.)


ΠŸΡ€Π°ΠΊΡ‚ΠΈΠΊΠ° β„–4

К ΠΎΠ±ΡŠΠ΅ΠΊΡ‚Ρƒ вСсом 5 ΠΊΠ³ ΠΏΡ€ΠΈΠΊΠ»Π°Π΄Ρ‹Π²Π°ΡŽΡ‚ Π½Π°ΠΏΡ€Π°Π²Π»Π΅Π½Π½ΡƒΡŽ Π²ΠΏΡ€Π°Π²ΠΎ силу для пСрСмСщСния Π΅Π³ΠΎ ΠΏΠΎ ΡˆΠ΅Ρ€ΠΎΡ…ΠΎΠ²Π°Ρ‚ΠΎΠΉ повСрхности с ускорСниСм Π²ΠΏΡ€Π°Π²ΠΎ 2 ΠΌ / с / с.ΠšΠΎΡΡ„Ρ„ΠΈΡ†ΠΈΠ΅Π½Ρ‚ трСния ΠΌΠ΅ΠΆΠ΄Ρƒ ΠΎΠ±ΡŠΠ΅ΠΊΡ‚ΠΎΠΌ ΠΈ ΠΏΠΎΠ²Π΅Ρ€Ρ…Π½ΠΎΡΡ‚ΡŒΡŽ 0,1. Π˜ΡΠΏΠΎΠ»ΡŒΠ·ΡƒΠΉΡ‚Π΅ Π΄ΠΈΠ°Π³Ρ€Π°ΠΌΠΌΡƒ, Ρ‡Ρ‚ΠΎΠ±Ρ‹ ΠΎΠΏΡ€Π΅Π΄Π΅Π»ΠΈΡ‚ΡŒ Π³Ρ€Π°Π²ΠΈΡ‚Π°Ρ†ΠΈΠΎΠ½Π½ΡƒΡŽ силу, Π½ΠΎΡ€ΠΌΠ°Π»ΡŒΠ½ΡƒΡŽ силу, ΠΏΡ€ΠΈΠ»ΠΎΠΆΠ΅Π½Π½ΡƒΡŽ силу, силу трСния ΠΈ Ρ‡ΠΈΡΡ‚ΡƒΡŽ силу. (ΠŸΡ€Π΅Π½Π΅Π±Ρ€Π΅Π³Π°Ρ сопротивлСниСм Π²ΠΎΠ·Π΄ΡƒΡ…Π°.)

ΠŸΡ€Π°ΠΊΡ‚ΠΈΠΊΠ° β„–5

К ΠΎΠ±ΡŠΠ΅ΠΊΡ‚Ρƒ массой 4 ΠΊΠ³ ΠΏΡ€ΠΈΠΊΠ»Π°Π΄Ρ‹Π²Π°ΡŽΡ‚ Π½Π°ΠΏΡ€Π°Π²Π»Π΅Π½Π½ΡƒΡŽ Π²ΠΏΡ€Π°Π²ΠΎ силу 25 Н, Ρ‡Ρ‚ΠΎΠ±Ρ‹ ΠΏΠ΅Ρ€Π΅ΠΌΠ΅ΡΡ‚ΠΈΡ‚ΡŒ Π΅Π³ΠΎ ΠΏΠΎ ΡˆΠ΅Ρ€ΠΎΡ…ΠΎΠ²Π°Ρ‚ΠΎΠΉ повСрхности с ΠΏΡ€Π°Π²Ρ‹ΠΌ ускорСниСм 2.5 ΠΌ / с / с. Π˜ΡΠΏΠΎΠ»ΡŒΠ·ΡƒΠΉΡ‚Π΅ Π΄ΠΈΠ°Π³Ρ€Π°ΠΌΠΌΡƒ, Ρ‡Ρ‚ΠΎΠ±Ρ‹ ΠΎΠΏΡ€Π΅Π΄Π΅Π»ΠΈΡ‚ΡŒ Π³Ρ€Π°Π²ΠΈΡ‚Π°Ρ†ΠΈΠΎΠ½Π½ΡƒΡŽ силу, Π½ΠΎΡ€ΠΌΠ°Π»ΡŒΠ½ΡƒΡŽ силу, силу трСния, Ρ‡ΠΈΡΡ‚ΡƒΡŽ силу ΠΈ коэффициСнт трСния ΠΌΠ΅ΠΆΠ΄Ρƒ ΠΎΠ±ΡŠΠ΅ΠΊΡ‚ΠΎΠΌ ΠΈ ΠΏΠΎΠ²Π΅Ρ€Ρ…Π½ΠΎΡΡ‚ΡŒΡŽ. (ΠŸΡ€Π΅Π½Π΅Π±Ρ€Π΅Π³Π°Ρ сопротивлСниСм Π²ΠΎΠ·Π΄ΡƒΡ…Π°.)

НиТС ΠΏΡ€ΠΈΠ²Π΅Π΄Π΅Π½Ρ‹ Π΅Ρ‰Π΅ нСсколько практичСских Π·Π°Π΄Π°Ρ‡. Π’Ρ‹ Π΄ΠΎΠ»ΠΆΠ½Ρ‹ ΠΏΠΎΡΡ‚Π°Ρ€Π°Ρ‚ΡŒΡΡ Ρ€Π΅ΡˆΠΈΡ‚ΡŒ ΠΊΠ°ΠΊ ΠΌΠΎΠΆΠ½ΠΎ большС ΠΏΡ€ΠΎΠ±Π»Π΅ΠΌ Π±Π΅Π· ΠΏΠΎΠΌΠΎΡ‰ΠΈ Π·Π°ΠΌΠ΅Ρ‚ΠΎΠΊ, Ρ€Π΅ΡˆΠ΅Π½ΠΈΠΉ, ΡƒΡ‡ΠΈΡ‚Π΅Π»Π΅ΠΉ ΠΈ Π΄Ρ€ΡƒΠ³ΠΈΡ… ΡƒΡ‡Π΅Π½ΠΈΠΊΠΎΠ².ΠŸΡ€ΠΈΠΌΠΈΡ‚Π΅ Ρ€Π΅ΡˆΠ΅Π½ΠΈΠ΅ ΠΈΠ½Π΄ΠΈΠ²ΠΈΠ΄ΡƒΠ°Π»ΡŒΠ½ΠΎ Ρ€Π΅ΡˆΠ°Ρ‚ΡŒ ΠΏΡ€ΠΎΠ±Π»Π΅ΠΌΡ‹. А ΠΏΠΎΠΊΠ° стоит ΡƒΠΏΠΎΠΌΡΠ½ΡƒΡ‚ΡŒ Π²Π°ΠΆΠ½ΠΎΠ΅ прСдостСрСТСниС:

Π˜Π·Π±Π΅Π³Π°ΠΉΡ‚Π΅ принуТдСния ΠΏΡ€ΠΎΠ±Π»Π΅ΠΌΡ‹ ΠΊ Ρ„ΠΎΡ€ΠΌΠ΅ Ρ€Π°Π½Π΅Π΅ Ρ€Π΅ΡˆΠ΅Π½Π½ΠΎΠΉ ΠΏΡ€ΠΎΠ±Π»Π΅ΠΌΡ‹. ΠŸΡ€ΠΎΠ±Π»Π΅ΠΌΡ‹ Π² Ρ„ΠΈΠ·ΠΈΠΊΠ΅ Ρ€Π΅Π΄ΠΊΠΎ выглядят ΠΎΠ΄ΠΈΠ½Π°ΠΊΠΎΠ²ΠΎ. ВмСсто Ρ‚ΠΎΠ³ΠΎ Ρ‡Ρ‚ΠΎΠ±Ρ‹ Ρ€Π΅ΡˆΠ°Ρ‚ΡŒ ΠΏΡ€ΠΎΠ±Π»Π΅ΠΌΡ‹ Π½Π°ΠΈΠ·ΡƒΡΡ‚ΡŒ ΠΈΠ»ΠΈ ΠΏΡƒΡ‚Π΅ΠΌ ΠΈΠΌΠΈΡ‚Π°Ρ†ΠΈΠΈ Ρ€Π°Π½Π΅Π΅ Ρ€Π΅ΡˆΠ΅Π½Π½ΠΎΠΉ ΠΏΡ€ΠΎΠ±Π»Π΅ΠΌΡ‹, ΠΈΡΠΏΠΎΠ»ΡŒΠ·ΡƒΠΉΡ‚Π΅ своС ΠΊΠΎΠ½Ρ†Π΅ΠΏΡ‚ΡƒΠ°Π»ΡŒΠ½ΠΎΠ΅ ΠΏΠΎΠ½ΠΈΠΌΠ°Π½ΠΈΠ΅ Π·Π°ΠΊΠΎΠ½ΠΎΠ² ΠΡŒΡŽΡ‚ΠΎΠ½Π° для поиска Ρ€Π΅ΡˆΠ΅Π½ΠΈΠΉ ΠΏΡ€ΠΎΠ±Π»Π΅ΠΌ. Π˜ΡΠΏΠΎΠ»ΡŒΠ·ΡƒΠΉΡ‚Π΅ своС ΠΏΠΎΠ½ΠΈΠΌΠ°Π½ΠΈΠ΅ вСса ΠΈ массы, Ρ‡Ρ‚ΠΎΠ±Ρ‹ Π½Π°ΠΉΡ‚ΠΈ m ΠΈΠ»ΠΈ Fgrav Π² ΠΏΡ€ΠΎΠ±Π»Π΅ΠΌΠ΅.Π˜ΡΠΏΠΎΠ»ΡŒΠ·ΡƒΠΉΡ‚Π΅ своС ΠΊΠΎΠ½Ρ†Π΅ΠΏΡ‚ΡƒΠ°Π»ΡŒΠ½ΠΎΠ΅ ΠΏΠΎΠ½ΠΈΠΌΠ°Π½ΠΈΠ΅ чистой силы (вСкторная сумма всСх сил ), Ρ‡Ρ‚ΠΎΠ±Ρ‹ Π½Π°ΠΉΡ‚ΠΈ Π·Π½Π°Ρ‡Π΅Π½ΠΈΠ΅ Fnet ΠΈΠ»ΠΈ Π·Π½Π°Ρ‡Π΅Π½ΠΈΠ΅ ΠΎΡ‚Π΄Π΅Π»ΡŒΠ½ΠΎΠΉ силы. НС отдСляйтС Ρ€Π΅ΡˆΠ΅Π½ΠΈΠ΅ физичСских Π·Π°Π΄Π°Ρ‡ ΠΎΡ‚ вашСго понимания ΠΊΠΎΠ½Ρ†Π΅ΠΏΡ†ΠΈΠΉ Ρ„ΠΈΠ·ΠΈΠΊΠΈ. Если Π²Ρ‹ Π½Π΅ ΠΌΠΎΠΆΠ΅Ρ‚Π΅ Ρ€Π΅ΡˆΠ°Ρ‚ΡŒ физичСскиС Π·Π°Π΄Π°Ρ‡ΠΈ, ΠΏΠΎΠ΄ΠΎΠ±Π½Ρ‹Π΅ ΠΏΡ€ΠΈΠ²Π΅Π΄Π΅Π½Π½Ρ‹ΠΌ Π²Ρ‹ΡˆΠ΅, это Π½Π΅ ΠΎΠ±ΡΠ·Π°Ρ‚Π΅Π»ΡŒΠ½ΠΎ ΠΎΠ·Π½Π°Ρ‡Π°Π΅Ρ‚, Ρ‡Ρ‚ΠΎ Ρƒ вас Π΅ΡΡ‚ΡŒ матСматичСскиС трудности. Π’ΠΏΠΎΠ»Π½Π΅ вСроятно, Ρ‡Ρ‚ΠΎ Ρƒ вас ΠΏΡ€ΠΎΠ±Π»Π΅ΠΌΡ‹ с физичСскими концСпциями.


Π₯ΠΎΡ‚ΠΈΠΌ ΠΏΡ€Π΅Π΄Π»ΠΎΠΆΠΈΡ‚ΡŒ… Иногда просто ΠΏΡ€ΠΎΡ‡ΠΈΡ‚Π°Ρ‚ΡŒ ΠΎΠ± этом нСдостаточно. Π’Ρ‹ Π΄ΠΎΠ»ΠΆΠ½Ρ‹ с Π½ΠΈΠΌ Π²Π·Π°ΠΈΠΌΠΎΠ΄Π΅ΠΉΡΡ‚Π²ΠΎΠ²Π°Ρ‚ΡŒ! И это ΠΈΠΌΠ΅Π½Π½ΠΎ Ρ‚ΠΎ, Ρ‡Ρ‚ΠΎ Π²Ρ‹ Π΄Π΅Π»Π°Π΅Ρ‚Π΅, ΠΊΠΎΠ³Π΄Π° ΠΈΡΠΏΠΎΠ»ΡŒΠ·ΡƒΠ΅Ρ‚Π΅ ΠΎΠ΄ΠΈΠ½ ΠΈΠ· ΠΈΠ½Ρ‚Π΅Ρ€Π°ΠΊΡ‚ΠΈΠ²Π½Ρ‹Ρ… ΠΌΠ°Ρ‚Π΅Ρ€ΠΈΠ°Π»ΠΎΠ² The Physics Classroom. ΠœΡ‹ Ρ…ΠΎΡ‚Π΅Π»ΠΈ Π±Ρ‹ ΠΏΡ€Π΅Π΄Π»ΠΎΠΆΠΈΡ‚ΡŒ Π²Π°ΠΌ ΡΠΎΠ²ΠΌΠ΅ΡΡ‚ΠΈΡ‚ΡŒ Ρ‡Ρ‚Π΅Π½ΠΈΠ΅ этой страницы с использованиСм нашСго Force Interactive. Π’Ρ‹ ΠΌΠΎΠΆΠ΅Ρ‚Π΅ Π½Π°ΠΉΡ‚ΠΈ Π΅Π³ΠΎ Π² Ρ€Π°Π·Π΄Π΅Π»Π΅ Physics Interactives Π½Π° нашСм сайтС. Force Interactive позволяСт учащСмуся ΠΈΡΡΠ»Π΅Π΄ΠΎΠ²Π°Ρ‚ΡŒ влияниС ΠΈΠ·ΠΌΠ΅Π½Π΅Π½ΠΈΠΉ ΠΏΡ€ΠΈΠ»ΠΎΠΆΠ΅Π½Π½ΠΎΠΉ силы, чистой силы, массы ΠΈ трСния Π½Π° ускорСниС ΠΎΠ±ΡŠΠ΅ΠΊΡ‚Π°.

ΠŸΡ€ΠΎΠ²Π΅Ρ€ΡŒΡ‚Π΅ своС ΠΏΠΎΠ½ΠΈΠΌΠ°Π½ΠΈΠ΅

1. Π›ΠΈ Милон катаСтся Π½Π° санях со своими Π΄Ρ€ΡƒΠ·ΡŒΡΠΌΠΈ, ΠΊΠΎΠ³Π΄Π° Π΅Π³ΠΎ Ρ€Π°Π·Π΄Ρ€Π°ΠΆΠ°Π΅Ρ‚ ΠΎΠ΄ΠΈΠ½ ΠΈΠ· ΠΊΠΎΠΌΠΌΠ΅Π½Ρ‚Π°Ρ€ΠΈΠ΅Π² Π΄Ρ€ΡƒΠ³Π°. Он ΠΏΡ€ΠΈΠ»Π°Π³Π°Π΅Ρ‚ ΠΊ своим 4,68 ΠΊΠ³ сани с силой 9,13 Н, Π½Π°ΠΏΡ€Π°Π²Π»Π΅Π½Π½ΠΎΠΉ Π²ΠΏΡ€Π°Π²ΠΎ, Ρ‡Ρ‚ΠΎΠ±Ρ‹ Ρ€Π°Π·ΠΎΠ³Π½Π°Ρ‚ΡŒ ΠΈΡ… ΠΏΠΎ снСгу. Если ускорСниС санСй составляСт 0,815 ΠΌ / с / с, Ρ‚ΠΎ ΠΊΠ°ΠΊΠΎΠ² коэффициСнт трСния санСй ΠΏΠΎ снСгу?

2.Π’ Π»Π°Π±ΠΎΡ€Π°Ρ‚ΠΎΡ€ΠΈΠΈ Ρ„ΠΈΠ·ΠΈΠΊΠΈ ЭрнСсто ΠΈ Аманда ΠΏΡ€ΠΈΠΊΠ»Π°Π΄Ρ‹Π²Π°ΡŽΡ‚ Π½Π°ΠΏΡ€Π°Π²Π»Π΅Π½Π½ΡƒΡŽ Π²ΠΏΡ€Π°Π²ΠΎ силу 34,5 Н ΠΊ Ρ‚Π΅Π»Π΅ΠΆΠΊΠ΅ массой 4,52 ΠΊΠ³, Ρ‡Ρ‚ΠΎΠ±Ρ‹ Ρ€Π°Π·ΠΎΠ³Π½Π°Ρ‚ΡŒ Π΅Π΅ ΠΏΠΎ Π³ΠΎΡ€ΠΈΠ·ΠΎΠ½Ρ‚Π°Π»ΡŒΠ½ΠΎΠΉ повСрхности со ΡΠΊΠΎΡ€ΠΎΡΡ‚ΡŒΡŽ 1,28 ΠΌ / с / с. ΠžΠΏΡ€Π΅Π΄Π΅Π»ΠΈΡ‚Π΅ силу трСния, Π΄Π΅ΠΉΡΡ‚Π²ΡƒΡŽΡ‰ΡƒΡŽ Π½Π° Ρ‚Π΅Π»Π΅ΠΆΠΊΡƒ.

Π€ΠΎΡ€ΠΌΡƒΠ»Π° статичСского трСния

БтатичСскоС Ρ‚Ρ€Π΅Π½ΠΈΠ΅ — это сила, которая ΡƒΠ΄Π΅Ρ€ΠΆΠΈΠ²Π°Π΅Ρ‚ ΠΎΠ±ΡŠΠ΅ΠΊΡ‚ Π² Π½Π΅ΠΏΠΎΠ΄Π²ΠΈΠΆΠ½ΠΎΠΌ состоянии. Π•Π³ΠΎ Π½Π΅ΠΎΠ±Ρ…ΠΎΠ΄ΠΈΠΌΠΎ ΠΏΡ€Π΅ΠΎΠ΄ΠΎΠ»Π΅Ρ‚ΡŒ, Ρ‡Ρ‚ΠΎΠ±Ρ‹ Π½Π°Ρ‡Π°Ρ‚ΡŒ Π΄Π²ΠΈΠΆΠ΅Π½ΠΈΠ΅ ΠΎΠ±ΡŠΠ΅ΠΊΡ‚Π°. Когда ΠΎΠ±ΡŠΠ΅ΠΊΡ‚ двиТСтся, ΠΎΠ½ испытываСт кинСтичСскоС Ρ‚Ρ€Π΅Π½ΠΈΠ΅.Если ΠΊ ΠΎΠ±ΡŠΠ΅ΠΊΡ‚Ρƒ ΠΏΡ€ΠΈΠ»ΠΎΠΆΠ΅Π½Π° нСбольшая сила, статичСскоС Ρ‚Ρ€Π΅Π½ΠΈΠ΅ ΠΈΠΌΠ΅Π΅Ρ‚ Ρ€Π°Π²Π½ΡƒΡŽ Π²Π΅Π»ΠΈΡ‡ΠΈΠ½Ρƒ Π² ΠΏΡ€ΠΎΡ‚ΠΈΠ²ΠΎΠΏΠΎΠ»ΠΎΠΆΠ½ΠΎΠΌ Π½Π°ΠΏΡ€Π°Π²Π»Π΅Π½ΠΈΠΈ. Если усилиС ΡƒΠ²Π΅Π»ΠΈΡ‡ΠΈΡ‚ΡŒ, Π² ΠΊΠ°ΠΊΠΎΠΉ-Ρ‚ΠΎ ΠΌΠΎΠΌΠ΅Π½Ρ‚ Π±ΡƒΠ΄Π΅Ρ‚ достигнуто Π·Π½Π°Ρ‡Π΅Π½ΠΈΠ΅ максимального статичСского трСния, ΠΈ ΠΎΠ±ΡŠΠ΅ΠΊΡ‚ сдвинСтся. ΠšΠΎΡΡ„Ρ„ΠΈΡ†ΠΈΠ΅Π½Ρ‚ статичСского трСния обозначаСтся грСчСской Π±ΡƒΠΊΠ²ΠΎΠΉ «мю» (ΞΌ) с Π½ΠΈΠΆΠ½ΠΈΠΌ индСксом Β«sΒ». Максимальная сила статичСского трСния составляСт с Π² Ρ€Π°Π· большС Π½ΠΎΡ€ΠΌΠ°Π»ΡŒΠ½ΠΎΠΉ силы, Π΄Π΅ΠΉΡΡ‚Π²ΡƒΡŽΡ‰Π΅ΠΉ Π½Π° ΠΎΠ±ΡŠΠ΅ΠΊΡ‚.

сила трСния покоя ≀ (коэффициСнт трСния покоя) (Π½ΠΎΡ€ΠΌΠ°Π»ΡŒΠ½Π°Ρ сила) максимальная сила трСния покоя = (коэффициСнт трСния покоя) (Π½ΠΎΡ€ΠΌΠ°Π»ΡŒΠ½Π°Ρ сила)

F с ≀ ΞΌ с Ξ· ΠΈ F с макс = ΞΌ с Ξ·

F с = сила трСния покоя

ΞΌ с = коэффициСнт трСния покоя

Ξ· = Π½ΠΎΡ€ΠΌΠ°Π»ΡŒΠ½Π°Ρ сила (грСчСская Π±ΡƒΠΊΠ²Π° «эта»)

≀ ΠΎΠ·Π½Π°Ρ‡Π°Π΅Ρ‚ «мСньшС ΠΈΠ»ΠΈ Ρ€Π°Π²Π½ΠΎΒ»

F с макс = максимальная сила статичСского трСния

Π€ΠΎΡ€ΠΌΡƒΠ»Π° статичСского трСния Вопросы:

1) К сани, Π½Π°Π±ΠΈΡ‚ΠΎΠΉ Π΄Ρ€ΠΎΠ²Π°ΠΌΠΈ, Π² заснСТСнном лСсу прилагаСтся усилиС 5500 Н.Π›Ρ‹ΠΆΠΈ санок ΠΈΠΌΠ΅ΡŽΡ‚ коэффициСнт трСния покоя ΞΌ s = 0,75 со снСгом. Если ΠΏΠΎΠ»Π½ΠΎΡΡ‚ΡŒΡŽ Π·Π°Π³Ρ€ΡƒΠΆΠ΅Π½Π½Ρ‹Π΅ салазки ΠΈΠΌΠ΅ΡŽΡ‚ массу 700 ΠΊΠ³, ΠΊΠ°ΠΊΠΎΠ²Π° максимальная сила статичСского трСния ΠΈ достаточно Π»ΠΈ ΠΏΡ€ΠΈΠ»ΠΎΠΆΠ΅Π½Π½ΠΎΠΉ силы для Π΅Π΅ прСодолСния?

ΠžΡ‚Π²Π΅Ρ‚: На плоской повСрхности Π½ΠΎΡ€ΠΌΠ°Π»ΡŒΠ½Π°Ρ сила, Π΄Π΅ΠΉΡΡ‚Π²ΡƒΡŽΡ‰Π°Ρ Π½Π° ΠΎΠ±ΡŠΠ΅ΠΊΡ‚, составляСт Ξ· = mg . Π˜ΡΠΏΠΎΠ»ΡŒΠ·ΡƒΡ это, ΠΌΠΎΠΆΠ½ΠΎ Π½Π°ΠΉΡ‚ΠΈ ΠΌΠ°ΠΊΡΠΈΠΌΠ°Π»ΡŒΠ½ΡƒΡŽ силу статичСского трСния:

F с макс = μ с η

F с макс = μ с мг

F с макс = (0.75) (700 кг) (9,8 м / (с 2 ))

F с макс = 5145 ΠΊΠ³ βˆ™ ΠΌ / с 2

F с макс = 5145 Н

Максимальная сила трСния покоя составляСт 5145 Н, поэтому ΠΏΡ€ΠΈΠ»ΠΎΠΆΠ΅Π½Π½ΠΎΠΉ силы 5500 Н достаточно, Ρ‡Ρ‚ΠΎΠ±Ρ‹ ΠΏΡ€Π΅ΠΎΠ΄ΠΎΠ»Π΅Ρ‚ΡŒ Π΅Π΅ ΠΈ Π½Π°Ρ‡Π°Ρ‚ΡŒ Π΄Π²ΠΈΠΆΠ΅Π½ΠΈΠ΅ салазок.

2) Π§Π΅Π»ΠΎΠ²Π΅ΠΊ, строящий ΠΌΠ°ΡˆΠΈΠ½Ρƒ для производства ΠΊΠΈΡ€ΠΏΠΈΡ‡Π°, Ρ…ΠΎΡ‡Π΅Ρ‚ ΠΈΠ·ΠΌΠ΅Ρ€ΠΈΡ‚ΡŒ коэффициСнт статичСского трСния ΠΌΠ΅ΠΆΠ΄Ρƒ ΠΊΠΈΡ€ΠΏΠΈΡ‡ΠΎΠΌ ΠΈ Π΄Π΅Ρ€Π΅Π²ΠΎΠΌ.

alexxlab

Π”ΠΎΠ±Π°Π²ΠΈΡ‚ΡŒ ΠΊΠΎΠΌΠΌΠ΅Π½Ρ‚Π°Ρ€ΠΈΠΉ

Π’Π°Ρˆ адрСс email Π½Π΅ Π±ΡƒΠ΄Π΅Ρ‚ ΠΎΠΏΡƒΠ±Π»ΠΈΠΊΠΎΠ²Π°Π½. ΠžΠ±ΡΠ·Π°Ρ‚Π΅Π»ΡŒΠ½Ρ‹Π΅ поля ΠΏΠΎΠΌΠ΅Ρ‡Π΅Π½Ρ‹ *